0% found this document useful (0 votes)
314 views211 pages

Full Chapter 1 PDF

This volume contains 14 chapters that aim to prove the Andrica Conjecture using elementary number theory. Chapter 1 introduces key concepts like totatives, nuclei, and nucleic sets that will be used to arrive at general counting formulas for numbers coprime to sets of primes. It also presents the Strong Andrica Conjecture and Surjection Theorem, which states there is a surjection between the gaps of consecutive integers coprime to consecutive primes and those coprime to any set of primes. Chapter 2 explores mappings between the gaps of different prime sets. Chapter 3 formally presents the Strong Andrica, Cramer, and Firoozbakht Conjectures, which are strengthened versions of existing conjectures about prime numbers and their distributions.

Uploaded by

Edward Solomon
Copyright
© © All Rights Reserved
We take content rights seriously. If you suspect this is your content, claim it here.
Available Formats
Download as PDF, TXT or read online on Scribd
0% found this document useful (0 votes)
314 views211 pages

Full Chapter 1 PDF

This volume contains 14 chapters that aim to prove the Andrica Conjecture using elementary number theory. Chapter 1 introduces key concepts like totatives, nuclei, and nucleic sets that will be used to arrive at general counting formulas for numbers coprime to sets of primes. It also presents the Strong Andrica Conjecture and Surjection Theorem, which states there is a surjection between the gaps of consecutive integers coprime to consecutive primes and those coprime to any set of primes. Chapter 2 explores mappings between the gaps of different prime sets. Chapter 3 formally presents the Strong Andrica, Cramer, and Firoozbakht Conjectures, which are strengthened versions of existing conjectures about prime numbers and their distributions.

Uploaded by

Edward Solomon
Copyright
© © All Rights Reserved
We take content rights seriously. If you suspect this is your content, claim it here.
Available Formats
Download as PDF, TXT or read online on Scribd
You are on page 1/ 211

Elementary Behaviors of

Consecutive Totatives,
and the Andrica Conjecture
Volume One

Edward K. Solomon
April 4th, 2020

Abstract
This volume contains a series of fourteen chapters that will use elementary number theory and
counting arguments to arrive at a proof of the Andrica Conjecture. The methods used also have great
potential towards solving other well known problems concerning prime numbers.

The first chapter shall strengthen the Andrica Conjecture by transforming the original conjecture
into a statement concerning Chinese Remainder Theorem, that given a set of n consecutive primes, Pn,
the largest difference between two consecutive integers pairwise coprime to Pn is precisely equal to 2pn−1
.
Furthermore, that given any general set of n primes, Sn, that the limit between two consecutive
integers coprime to Sn is less than or equal to 2pn−1 , as Chinese Remainder will be used to demonstrate
that there exists a surjection for the lengths of every gap generated by Sn between all consecutive
integers pairwise coprime to Sn, unto the gaps generated by Pn between all consecutive integers pairwise
coprime to Pn. This result shall be known as the Surjection Theorem, and operates under the assumption
that the Strong Andrica Conjecture has been proven by brute force for Pn.
Introduction
Prime numbers behave in a chaotic manner, which often makes it difficult to establish rigid rules
governing their distribution, especially their distribution within quadratic intervals. In this paper a quadratic
interval is the distance between the squares of two consecutive primes. All composite numbers within a
quadratic interval are divisible by some prime number less than or equal to the two squared consecutive
primes bounding the interval. This was a well known theorem to the ancient Greeks, who used it to
construct a prime number finding algorithm titled the Sieve of Eratosthenes.

This ancient algorithm sieves all integers that are divisible by primes less than or equal to the
primes bounding the local (quadratic) interval, leaving only prime numbers. At this point one may ask: Is
there a well-defined limit to how many consecutive integers can be sieved using a finite set of prime
numbers? It is this question that forces us to examine a much larger question: What is the largest
possible difference between consecutive totatives of some integer k? Since all prime numbers in a local
interval are totatives of the product of the sieving set of prime numbers, an answer to the latter question
also answers the original question.
Master Table of Contents
Volume One
Part One, The Strong Andrica Conjecture
❖ Page 5…...: Chapter 1; Totatives, Centrum, Nucleic Sets, Strong Andrica Conjecture, Surjection Theorem
❖ Page 211….: Chapter 2; Belligerence Mandate, Proving the Belligerence Mandate
❖ Page 254...: Chapter 3; Strong Andrica-Bertrand Postulate, Quartic Bracket Theorem; Quartic Sovereignty
❖ Page 314...: Chapter 4; Strong Legendre-Bertrand Postulate, Quartic Growth, Quadratic Growth
❖ Page 491...: Chapter 5; Initializing the Proof of the Andrica Conjecture by Induction
❖ Page 623...: Chapter 6; Numerical Data Concerning Lengths of Gaps with Increasing Belligerence.
❖ Page 871...: Chapter 7; Cascade Failure, Proving the Cascade Failure by Infinite Descent
❖ Page 925...: Chapter 8; The Belligerence Cascade, Proving the Belligerence Cascade
❖ Page 990...: Chapter 9; Returning to the Proof of the Andrica Conjecture by Induction
❖ Page 997...: Chapter 10; Tension, J - Order Tension; Tension Theory
❖ Page 1165...: Chapter 11; Restating the Strong Andrica Conjecture in terms of Third-Order Tension
❖ Page 1175...: Chapter 12; Applying the Surjection Theorem to Third-Order Tension
❖ Page 1253...: Chapter 13; Applying the Belligerence Mandate and the Belligerence Cascade to Third-Order Tension.
❖ Page 1379...: Chapter 14; Completing the Proof of the Strong Andrica, Cramer and Firoozbakht Conjectures.
Part Two, Proving the Proper Cramer and Firoozbakht Conjectures
❖ Page 1580...: Chapter 15; Quartic Sovereignty and Celestial Dance
❖ Page 1677...: Chapter 16; Common Barycenters
❖ Page 1691...: Chapter 17; The Activity Limit within the Eratosthenes Strip; Quadratic Strip; Upsilon
❖ Page 1951...: Chapter 18; Completing the Proofs of the Proper Cramer and Firoozbakht Conjectures
❖ Page 2132-2168...: Glossary
Table of Contents
Chapter 1,
Totatives, Polynomic Counting Functions,
Strong Andrica Conjecture, Surjection Theorem

Part One, The Andrica Conjecture


❖ Section ɪ: Polynomic Counting Functions
➢ 1.1; Page 5: Totatives
➢ 1.2; Page 6: Definitions of Particular Sets: S, P and W; and T.
■ Page 7: Omega function, the product of a set.
➢ 1.3; Page 8: Euler Totient Function
➢ 1.4; Page 9: Chinese Remainder Theorem
➢ 1.5; Page 10-14: Geometrically Counting Totatives using rectangular rank- n tensors
■ Page 15: On Allusions to Fermat’s Last Theorem
➢ 1.6; Page 17: Centrum, Star
■ Page 18: Nucleus , Exclusive and Inclusive
■ Page 18: How to Identify a Centrum by its System of Congruences.
➢ 1.7; Page 19-22: Elements of a Nucleus; Zeta Function
➢ 1.8; Page 23: Nucleic Sets
■ Pages 24-28: Example of the Nucleic Sets in respect to S = {2,3,5,7}
➢ 1.9.1; Page 28-61: Geometrically Counting the Size of Nucleic Sets
➢ 1.9.2; Page 61: Using Set Theory to Derive the General Counting Formula (Class B2)
■ Initial Cases of ( n − 2 ) = 3 and 4
■ Page 71: For all n .
➢ 1.10; Page 72: General Counting Formulas. Classes: A0, A1, A2, A3, B1, B2
■ Page 72: General Formula Class A0, s ≥ (n + 1), ∀s ∈ S, k > n +1, The Null Count
■ Page 73: General Formula Class A1, s ≥ (n + 1), ∀s ∈ S, k = n +1, The Factorial Count
■ Page 74: General Formula Class A2, s ≥ (n + 1), ∀s ∈ S, k < n +1, The Permutative Count
■ Page 77: Primorial Product to Sum Formula.
■ Page 78: Euler Phi Function Polynomic Identity
■ Page 78: Immutable Preset, Absolute Belligerence Counting Function
■ Page 79: Absolute Bellligerence Counting Theorem.
■ Page 80-81 :General Formula Class B1; Mutable Cyclic Preset (Primorial)
■ Page 82-83: General Formula Class B2: Repeating Mutable Cyclic Preset
■ Page 84: Senary Boolean Theorem, Page 100: Augmented Senary Class B3 Formula.
■ Page 85: Specialized Formula Class C1: Complete or Partial Cyclic Senary Presets
● All Gaps
■ Page 86: Specialized Formula Class C2: Partial Trigesimal Cyclic Presets, Radix 30
● Page 90: Radix, Frequency, Isolation definitions
● Gaps of 2, 4, 6, 8
● Page 95-98: 12, 14, 16, 18, 22, 24, 26, 28
● Page 94: General Class C2 Formula, Gaps of h , Page 99: Repeated C2
● Page 101: Gaps of 10, 20, 30, General Formula Class D1 for Gap Ω
■ Page 103: Rank- h Euler Polynomial Identities (moved to Appendix)
❖ Section II: Mapping Gaps between distinct Primordial Sets
➢ Section II: Mapping Gaps between distinct Primordial Sets
■ Abstract: Page 102
■ 2.1; Page 103: Preliminary Definitions and Lemmas
● Page 103: Superset Span Theorem
● Page 105: Logical Equivalents of the Superset Span Theorem.
● Page 105: Primordial Set, the Union of all Nucleic Sets.
● Page 106: Principal Dirichlet Filter Function
■ 2.2; Page 107: Maps Between |S| = |T| and δ [S] = δ [T]
● Page 107: Large Prime Set, Surjection Lemma.
● Pages: 108 - 118: Numerical Example, Mapping S = {2,7,67,71} to T = {2,11,13,
19}
■ 2.3; page 120
● Page 120: Types of Subsets and Intersections, Exchanges
● Foreign Sets, Divorced Sets, Familial Sets, Estranged Sets, Exchange Function
● Familial Surjection, Bijection, Injection Lemmas,.
● Absolute Orthogonal Surjection, Bijection, Injection
❖ Section III: Strong Andrica Conjecture
➢ 3.1; Page 129: Strong Andrica Conjecture
■ Page 129: Strong Andrica Conjecture Base Case
■ Page 131: The Celestial Definitions
■ Page 132: The Quartic Set
■ Page 133: Bracketed Primes (meson); Anchor
■ Page 134: Perturbed Goldbach Star; Page 135: Barycenter; Leptons and Hadrons
➢ 3.2; Page 137: Strong Cramer Conjecture; Page 140: Gluon Set
■ Page 152: Firoozbakht’s Conjecture
■ Page 156: The Strong and Proper Forms of the Andrica, Cramer and Firoozbakht
Conjectures;
■ Page 157 The Activity Limit (Upsilon)

❖ Section IV: Surjection Theorem


➢ Page 158: Surjection Theorem

❖ Section V: Local Appendices for Chapter 1.


➢ Page 181
■ Page 181, Appendix A: Computer Script to verify Strong Andrica Conjecture Base Case
■ Page 183, Appendix B: Class A - D Formulas, Counting Gaps of length k
■ Page 190: Appendix C: Counting Surjections
■ Page 201: Appendix D: Rank Euler Polynomic Identities
■ Page 205-210: Appendix E: Extended Rigorous Treatment of Case 3a, Surjection Theorem.
Section I

Section I Abstract

In Section 1 we will learn how to explicitly count the numbers of gaps of a specific length that
exist between consecutive totative pairs. The methods used to derive these counting formulas will
illuminate why they take on the form of polynomial identities; initially we will begin our derivations through
rather crude means on a case-by-case basis, however this initial approach is organic, visceral and
intuitive; afterwards, we shall derive the formulas again, but in rigorous fashion, and in such a
comprehensive manner that a general counting formula is made manifest.

We shall then proceed to formulate specialized counting functions by learning how to identify and
isolate the general polynomic elements peculiar to certain gaps, and then recombine their general
elements into a specialized counting function.

Totatives

1.1 Preliminary Definitions

A totative is a special case of two relatively prime numbers. If x is less than y, and x and y share
no common factors, then x is a totative of y. This definition includes x = 1 as being relatively prime to y. In
this paper the term coprime is preferred to relatively prime when the term totative cannot be used.

Definition 1.1.1, Totative


A totative is a positive integer less than or equal to a number n which is also relatively prime to n, where 1 is counted
as being relatively prime to all numbers. The number of totatives of n is the value of the totient function φ(n) .

The term was popularized by Sylvester (1879; Dickson 2005, p. 124), who spelled it "totitive."
http://mathworld.wolfram.com/Totative.html
1.2 Definitions of Particular Sets

Definition 1.2.1 General Set of Primes and cardinality.


Let Sn be a set of prime numbers, and n be the cardinality of S, such that |S| = n

Let Sn be ordered from least to greatest.

Definition 1.2.2 The Set of Consecutive Primes; {2,3,5,7,11…}

b
Let Pn denote a set of the first n consecutive primes, and let P a denote a set of consecutive
th
primes starting from the ath prime and ending on the b prime.

Let Pn-k denote the set of the first (n − k) consecutive primes.

b
Pn , P a and Pn-k shall always be ordered from least to greatest.

Definition 1.2.3 Fractured Set of Primes; n = 8; W 4


n−4 = {2,3,5,7,11, 29,59,101,127}

Given the set Pn-k , then:


Let si ∈ Sc, such that ∀si ∈ Sc, si > pn
c
Let W n−k denote the union of the sets Pn-k ⋃ Sc.

c
By definition Pn-k ⋂ Sc = ∅ , therefore |W n−k | = (n − k + c)

Definition 1.2.4a Regularized Fractured Set of Primes

Given the set Pn-k , then:


Let si ∈ Sc, such that ∀si ∈ Sc, si > pn and c = k
c
Let W n−k denote the union of the sets Pn-k ⋃ Sc.

c
By definition Pn-k ⋂ Sc = ∅ , therefore |W n−k | = (n − k + c) = n , since c = k

k
The regularized set will often be written simply as W n−k . This set is the primary focus of
establishing the Base Case for the proof of Andrica Conjecture, which shall be a proof by induction on the
sets Pn+1 and Sn+1 . Showing that the Base Case holds for Pn alone is not sufficient! The Base Case must
be shown for all Sn before one can proceed to the inductive case of (n + 1) .
Definition 1.2.4b Diminished Fractured Set of Primes
Given the set Pn-k , then:
Let si ∈ Sc, such that ∀si ∈ Sc, si > pn and c < k
c
Let W n−k denote the union of the sets Pn-k ⋃ Sc.

c
By definition Pn-k ⋂ Sc = ∅ , therefore |W n−k | = (n − k + c) < n , since c < k

Definition 1.2.4c Amplified Fractured Set of Primes


Given the set Pn-k , then:
Let si ∈ Sc, such that ∀si ∈ Sc, si > pn and c > k
c
Let W n−k denote the union of the sets Pn-k ⋃ Sc.

c
By definition Pn-k ⋂ Sc = ∅ , therefore |W n−k | = (n − k + c) > n , since c > k

Definition 1.2.5 Perforated Set of Primes; {2,3,5,59,11,83,17,19,23}


7 , 13
Given the sets Pk and Sk, then:
Pk ⋂ Sk = Tm; m ≥ 1

Then let:
Sk - Tm = Vr ; r ≥ (k − m)

= Tm ⋃ Vr is said to be a Perforated Set against Pk.


r
Then the set T m

This set was titled “Perforated,” since it is akin to taking the set Pk and randomly removing some
primes from it and then replacing the missing primes with some set of arbitrarily large primes (since by
definition all primes in Vr are greater than pk ).

Also bear in mind that a Fractured Set of Primes is also a Perforated Set of Primes, but a
Perforated Set of Primes is not a Fractured Set of Primes; as such, a Perforated Set can also be
diminished or amplified, but the universal notation system for dealing with such entities has yet to be
introduced, and will be addressed later in this chapter.

The laws governing the maximum difference between consecutive totatives of Fractured and
Perforated sets are the primary focus of this entire volume. We will first encounter these sets in the Fourth
Section of Chapter 1 (The Surjection Theorem).

Definition 1.2.6 The Omega Function, product of the set of primes.


i= n
Let Ω[Sn] = ∏ si
i =1

This function may act on any of the aforementioned sets.


Definition 1.2.7 Principal Dirichlet Character, λ

Given Sn, let λ(k) = 0 if k divides Ω[Sn], and let λ(k) = 1 if k does not divide Ω[Sn].

Although the Dirichlet Characters are mostly a relic from previous iterations of this paper over the
last decade, they are still quite useful, but no longer dominate the main body of the paper as they did in
the past. An evolution of the concept in the form of “Dirichlet Filters” arises when mapping the count of
particular gaps generated by some set onto another set, which will be seen later in this chapter.

1.3 The Euler Totient Function, counting totatives


The Euler Totient Function yields the exact number of totatives of any positive integer. So if we
wanted to count how many totatives there are to the number Ω[Sn], we would perform the Euler Totient
Function.

Definition 1.3.1 Formula for the Euler Totient Function for a set of distinct primes.

i= n
Let Φ (Ω[Sn]) = ∏ si − 1
i =1
1.4 An Alternate Method of Counting Totatives,
Chinese Remainder Theorem

Chinese Remainder Theorem and Totatives go hand-in-hand. Chinese Remainder Theorem


establishes that every totative t of some some integer k has a unique system of congruences modulo k,
and every valid system of congruences that allows some integer t to be a totative to k is exhausted for
1 ≤ t ≤ k.

Although Chinese Remainder Theorem is an essential argument often used to derive the formula
of the Euler Totient Function and to prove that the Euler Totient Function is multiplicative, there is actually
a more simplistic and geometric manner in which Chinese Remainder Theorem can be used to count the
total numbers of totatives to an integer.

Furthermore, this geometric method can not only be used to obtain the precise number of
totatives to some integer, but in fact count the existence of how many pairs of consecutive totatives are
separated by a specified distance, such as twin and cousin totatives.

Had these geometric proofs been both common knowledge and in common use by the
mathematical community, I have no doubt that the Andrica Conjecture would have already been solved
decades (or perhaps even centuries) prior to this date. None of the work in this volume is particularly
difficult, rather it’s just long and tedious.

Definition 1.4.1 The Totative Set

Let TotS be a set the totatives to Ω[S].

Now we wish to determine the cardinality of TotS. Each totative has a unique system of
congruences, and each system of congruences that permits a solution that is a totative must be
exhausted between 1 and Ω[S].

Each system of congruences that permits a solution that is totative has the following form:
ti ≡ a1 mod s1 ≡ a2 mod s2 ≡ a3 mod s3 ... ≡ an mod sn ∣ ∀a, a =/ 0

Then it follows that for any prime si in S, there are (si − 1) non-zero remainders that satisfy the above
form. Thus ai = 1 or 2 or 3 ... or si − 1

Thus the total count of system of congruences whose solutions yield a totative is simply the
i=n
product of each prime in S subtracted by one, therefore |TotS| = ∏ (si − 1) , this is the exact same formula
i=1
yielded by the Euler Totient Function.
1.5 The Geometrical Interpretation of the Totative Counting Method

To begin this section, we will first examine a set containing two primes and count their totatives.

Let S = {7,11} and let us compile all possible systems of congruences in respect to 7 and 11 into a
matrix whose dimensions are 7x11.

k ≡ a mod 7 ≡ b mod 11 ; − 3 ≤ a ≤+ 3, − 5 ≤ b ≤+ 5, a, b ∈ Z (the directions of positive and


negative entries are reversed in these matrices, the reason for this shall be revealed later).

+5, -3 +4, -3 +3, -3 +2, -3 +1, -3 0, -3 -1, -3 -2, -3 -3, -3 -4, -3 -5, -3

+5, -2 +4, -2 +3, -2 +2, -2 +1, -2 0, -2 -1, -2 -2, -2 -3, -2 -4, -2 -5, -2

+5, -1 +4, -1 +3, -1 +2, -1 +1, -1 0, -1 -1, -1 -2, -1 -3, -1 -4, -1 -5, -1

+5, 0 +4, 0 +3, 0 +2, 0 +1, 0 0, 0 -1, 0 -2, 0 -2, 0 -4, 0 -5, 0

+5, +1 +4, +1 +3, +1 +2, +1 +1, +1 0, +1 -1, +1 -2, +1 -2, +1 -4, +1 -5, +1

+5, +2 +4, +2 +3, +2 +2, +2 +1, +2 0, +2 -1, +2 -2, +2 -2, +2 -4, +2 -5, +2

+5, +3 +4, +3 +3, +3 +2, +3 +1, +3 0, +3 -1, +3 -2, +3 -2, +3 -4, +3 -5, +3

All of the above 77 entries containing a zero represent a number that can be divided by either 7
or 11 or both. Those numbers are not totatives (highlighted in yellow), and therefore must be subtracted
from the count of 77. We shall rearrange the matrix to place all the entries containing zeros on the edges
of the matrix.

k ≡ a mod 7 ≡ b mod 11 ; − 3 ≤ a ≤+ 3, − 5 ≤ b ≤+ 5, a, b ∈ Z
0, 0 -1, 0 -2, 0 -2, 0 -4, 0 -5, 0 +5, 0 +4, 0 +3, 0 +2, 0 +1, 0

0, +1 -1, +1 -2, +1 -2, +1 -4, +1 -5, +1 +5, +1 +4, +1 +3, +1 +2, +1 +1, +1

0, +2 -1, +2 -2, +2 -2, +2 -4, +2 -5, +2 +5, +2 +4, +2 +3, +2 +2, +2 +1, +2

0, +3 -1, +3 -2, +3 -2, +3 -4, +3 -5, +3 +5, +3 +4, +3 +3, +3 +2, +3 +1, +3

0, -3 -1, -3 -2, -3 -3, -3 -4, -3 -5, -3 +5, -3 +4, -3 +3, -3 +2, -3 +1, -3

0, -2 -1, -2 -2, -2 -3, -2 -4, -2 -5, -2 +5, -2 +4, -2 +3, -2 +2, -2 +1, -2

0, -1 -1, -1 -2, -1 -3, -1 -4, -1 -5, -1 +5, -1 +4, -1 +3, -1 +2, -1 +1, -1

The remaining entries are highlighted in blue, and they represent all of the totatives in respect to
the number 77. They form a submatrix of dimensions 6x10 = (7-1)(11-1), which is in agreement with the
Euler Totient Function. This matrix maneuver could have been performed on any set of two primes.

Now we shall examine a set of three primes, {5,7,11}, and represent all possible systems of
congruences as a rank-3 tensor. Since I can only write in two dimensions, I will split this rank-3 tenor into
five slates of 77, with each slate representing a fixed remainder in respect to the modulus 5.

/
k ≡ − 2 mod 5 ≡ a mod 7 ≡ b mod 11 ; − 3 ≤ a ≤+ 3, − 5 ≤ b ≤+ 5, a, b ∈ Z
+5, -3 +4, -3 +3, -3 +2, -3 +1, -3 0, -3 -1, -3 -2, -3 -3, -3 -4, -3 -5, -3

+5, -2 +4, -2 +3, -2 +2, -2 +1, -2 0, -2 -1, -2 -2, -2 -3, -2 -4, -2 -5, -2

+5, -1 +4, -1 +3, -1 +2, -1 +1, -1 0, -1 -1, -1 -2, -1 -3, -1 -4, -1 -5, -1

+5, 0 +4, 0 +3, 0 +2, 0 +1, 0 0, 0 -1, 0 -2, 0 -2, 0 -4, 0 -5, 0

+5, +1 +4, +1 +3, +1 +2, +1 +1, +1 0, +1 -1, +1 -2, +1 -2, +1 -4, +1 -5, +1

+5, +2 +4, +2 +3, +2 +2, +2 +1, +2 0, +2 -1, +2 -2, +2 -2, +2 -4, +2 -5, +2

+5, +3 +4, +3 +3, +3 +2, +3 +1, +3 0, +3 -1, +3 -2, +3 -2, +3 -4, +3 -5, +3

k ≡ − 1 mod 5 ≡ a mod 7 ≡ b mod 11 ; − 3 ≤ a ≤+ 3, − 5 ≤ b ≤+ 5, a, b ∈ Z


+5, -3 +4, -3 +3, -3 +2, -3 +1, -3 0, -3 -1, -3 -2, -3 -3, -3 -4, -3 -5, -3

+5, -2 +4, -2 +3, -2 +2, -2 +1, -2 0, -2 -1, -2 -2, -2 -3, -2 -4, -2 -5, -2

+5, -1 +4, -1 +3, -1 +2, -1 +1, -1 0, -1 -1, -1 -2, -1 -3, -1 -4, -1 -5, -1

+5, 0 +4, 0 +3, 0 +2, 0 +1, 0 0, 0 -1, 0 -2, 0 -2, 0 -4, 0 -5, 0

+5, +1 +4, +1 +3, +1 +2, +1 +1, +1 0, +1 -1, +1 -2, +1 -2, +1 -4, +1 -5, +1

+5, +2 +4, +2 +3, +2 +2, +2 +1, +2 0, +2 -1, +2 -2, +2 -2, +2 -4, +2 -5, +2

+5, +3 +4, +3 +3, +3 +2, +3 +1, +3 0, +3 -1, +3 -2, +3 -2, +3 -4, +3 -5, +3

k ≡ 0 mod 5 ≡ a mod 7 ≡ b mod 11 ; − 3 ≤ a ≤+ 3, − 5 ≤ b ≤+ 5, a, b ∈ Z


+5, -3 +4, -3 +3, -3 +2, -3 +1, -3 0, -3 -1, -3 -2, -3 -3, -3 -4, -3 -5, -3

+5, -2 +4, -2 +3, -2 +2, -2 +1, -2 0, -2 -1, -2 -2, -2 -3, -2 -4, -2 -5, -2

+5, -1 +4, -1 +3, -1 +2, -1 +1, -1 0, -1 -1, -1 -2, -1 -3, -1 -4, -1 -5, -1

+5, 0 +4, 0 +3, 0 +2, 0 +1, 0 0, 0 -1, 0 -2, 0 -2, 0 -4, 0 -5, 0

+5, +1 +4, +1 +3, +1 +2, +1 +1, +1 0, +1 -1, +1 -2, +1 -2, +1 -4, +1 -5, +1

+5, +2 +4, +2 +3, +2 +2, +2 +1, +2 0, +2 -1, +2 -2, +2 -2, +2 -4, +2 -5, +2

+5, +3 +4, +3 +3, +3 +2, +3 +1, +3 0, +3 -1, +3 -2, +3 -2, +3 -4, +3 -5, +3

k ≡ + 1 mod 5 ≡ a mod 7 ≡ b mod 11 ; − 3 ≤ a ≤+ 3, − 5 ≤ b ≤+ 5, a, b ∈ Z


+5, -3 +4, -3 +3, -3 +2, -3 +1, -3 0, -3 -1, -3 -2, -3 -3, -3 -4, -3 -5, -3

+5, -2 +4, -2 +3, -2 +2, -2 +1, -2 0, -2 -1, -2 -2, -2 -3, -2 -4, -2 -5, -2

+5, -1 +4, -1 +3, -1 +2, -1 +1, -1 0, -1 -1, -1 -2, -1 -3, -1 -4, -1 -5, -1

+5, 0 +4, 0 +3, 0 +2, 0 +1, 0 0, 0 -1, 0 -2, 0 -2, 0 -4, 0 -5, 0

+5, +1 +4, +1 +3, +1 +2, +1 +1, +1 0, +1 -1, +1 -2, +1 -2, +1 -4, +1 -5, +1

+5, +2 +4, +2 +3, +2 +2, +2 +1, +2 0, +2 -1, +2 -2, +2 -2, +2 -4, +2 -5, +2

+5, +3 +4, +3 +3, +3 +2, +3 +1, +3 0, +3 -1, +3 -2, +3 -2, +3 -4, +3 -5, +3

k ≡ + 2 mod 5 ≡ a mod 7 ≡ b mod 11 ; − 3 ≤ a ≤+ 3, − 5 ≤ b ≤+ 5, a, b ∈ Z


+5, -3 +4, -3 +3, -3 +2, -3 +1, -3 0, -3 -1, -3 -2, -3 -3, -3 -4, -3 -5, -3

+5, -2 +4, -2 +3, -2 +2, -2 +1, -2 0, -2 -1, -2 -2, -2 -3, -2 -4, -2 -5, -2

+5, -1 +4, -1 +3, -1 +2, -1 +1, -1 0, -1 -1, -1 -2, -1 -3, -1 -4, -1 -5, -1

+5, 0 +4, 0 +3, 0 +2, 0 +1, 0 0, 0 -1, 0 -2, 0 -2, 0 -4, 0 -5, 0

+5, +1 +4, +1 +3, +1 +2, +1 +1, +1 0, +1 -1, +1 -2, +1 -2, +1 -4, +1 -5, +1

+5, +2 +4, +2 +3, +2 +2, +2 +1, +2 0, +2 -1, +2 -2, +2 -2, +2 -4, +2 -5, +2

+5, +3 +4, +3 +3, +3 +2, +3 +1, +3 0, +3 -1, +3 -2, +3 -2, +3 -4, +3 -5, +3

/
Here is an image of the entire tensor, both separated into slates and as a single entity; the yellow
coloring represents entries in the tensor containing zeros:

As in the previous two dimensional matrix (rank-2 tensor), we shall now proceed to shuffle the
slates to place all the zero (yellow) entries on the exterior of the rectangular prism yielding a half-shell
pattern.

/
k ≡ 0 mod 5 ≡ a mod 7 ≡ b mod 11 ; − 3 ≤ a ≤+ 3, − 5 ≤ b ≤+ 5, a, b ∈ Z
0, 0 -1, 0 -2, 0 -2, 0 -4, 0 -5, 0 +5, 0 +4, 0 +3, 0 +2, 0 +1, 0

0, +1 -1, +1 -2, +1 -2, +1 -4, +1 -5, +1 +5, +1 +4, +1 +3, +1 +2, +1 +1, +1

0, +2 -1, +2 -2, +2 -2, +2 -4, +2 -5, +2 +5, +2 +4, +2 +3, +2 +2, +2 +1, +2

0, +3 -1, +3 -2, +3 -2, +3 -4, +3 -5, +3 +5, +3 +4, +3 +3, +3 +2, +3 +1, +3

0, -3 -1, -3 -2, -3 -3, -3 -4, -3 -5, -3 +5, -3 +4, -3 +3, -3 +2, -3 +1, -3

0, -2 -1, -2 -2, -2 -3, -2 -4, -2 -5, -2 +5, -2 +4, -2 +3, -2 +2, -2 +1, -2

0, -1 -1, -1 -2, -1 -3, -1 -4, -1 -5, -1 +5, -1 +4, -1 +3, -1 +2, -1 +1, -1

k ≡− 2 mod 5 ≡ a mod 7 ≡ b mod 11 ; − 3 ≤ a ≤+ 3, − 5 ≤ b ≤+ 5, a, b ∈ Z


0, 0 -1, 0 -2, 0 -2, 0 -4, 0 -5, 0 +5, 0 +4, 0 +3, 0 +2, 0 +1, 0

0, +1 -1, +1 -2, +1 -2, +1 -4, +1 -5, +1 +5, +1 +4, +1 +3, +1 +2, +1 +1, +1

0, +2 -1, +2 -2, +2 -2, +2 -4, +2 -5, +2 +5, +2 +4, +2 +3, +2 +2, +2 +1, +2

0, +3 -1, +3 -2, +3 -2, +3 -4, +3 -5, +3 +5, +3 +4, +3 +3, +3 +2, +3 +1, +3

0, -3 -1, -3 -2, -3 -3, -3 -4, -3 -5, -3 +5, -3 +4, -3 +3, -3 +2, -3 +1, -3

0, -2 -1, -2 -2, -2 -3, -2 -4, -2 -5, -2 +5, -2 +4, -2 +3, -2 +2, -2 +1, -2

0, -1 -1, -1 -2, -1 -3, -1 -4, -1 -5, -1 +5, -1 +4, -1 +3, -1 +2, -1 +1, -1

k ≡− 1 mod 5 ≡ a mod 7 ≡ b mod 11 ; − 3 ≤ a ≤+ 3, − 5 ≤ b ≤+ 5, a, b ∈ Z


0, 0 -1, 0 -2, 0 -2, 0 -4, 0 -5, 0 +5, 0 +4, 0 +3, 0 +2, 0 +1, 0

0, +1 -1, +1 -2, +1 -2, +1 -4, +1 -5, +1 +5, +1 +4, +1 +3, +1 +2, +1 +1, +1

0, +2 -1, +2 -2, +2 -2, +2 -4, +2 -5, +2 +5, +2 +4, +2 +3, +2 +2, +2 +1, +2

0, +3 -1, +3 -2, +3 -2, +3 -4, +3 -5, +3 +5, +3 +4, +3 +3, +3 +2, +3 +1, +3

0, -3 -1, -3 -2, -3 -3, -3 -4, -3 -5, -3 +5, -3 +4, -3 +3, -3 +2, -3 +1, -3

0, -2 -1, -2 -2, -2 -3, -2 -4, -2 -5, -2 +5, -2 +4, -2 +3, -2 +2, -2 +1, -2

0, -1 -1, -1 -2, -1 -3, -1 -4, -1 -5, -1 +5, -1 +4, -1 +3, -1 +2, -1 +1, -1

k ≡+ 1 mod 5 ≡ a mod 7 ≡ b mod 11 ; − 3 ≤ a ≤+ 3, − 5 ≤ b ≤+ 5, a, b ∈ Z


0, 0 -1, 0 -2, 0 -2, 0 -4, 0 -5, 0 +5, 0 +4, 0 +3, 0 +2, 0 +1, 0

0, +1 -1, +1 -2, +1 -2, +1 -4, +1 -5, +1 +5, +1 +4, +1 +3, +1 +2, +1 +1, +1

0, +2 -1, +2 -2, +2 -2, +2 -4, +2 -5, +2 +5, +2 +4, +2 +3, +2 +2, +2 +1, +2

0, +3 -1, +3 -2, +3 -2, +3 -4, +3 -5, +3 +5, +3 +4, +3 +3, +3 +2, +3 +1, +3

0, -3 -1, -3 -2, -3 -3, -3 -4, -3 -5, -3 +5, -3 +4, -3 +3, -3 +2, -3 +1, -3

0, -2 -1, -2 -2, -2 -3, -2 -4, -2 -5, -2 +5, -2 +4, -2 +3, -2 +2, -2 +1, -2

0, -1 -1, -1 -2, -1 -3, -1 -4, -1 -5, -1 +5, -1 +4, -1 +3, -1 +2, -1 +1, -1

k ≡+ 2 mod 5 ≡ a mod 7 ≡ b mod 11 ; − 3 ≤ a ≤+ 3, − 5 ≤ b ≤+ 5, a, b ∈ Z


0, 0 -1, 0 -2, 0 -2, 0 -4, 0 -5, 0 +5, 0 +4, 0 +3, 0 +2, 0 +1, 0

0, +1 -1, +1 -2, +1 -2, +1 -4, +1 -5, +1 +5, +1 +4, +1 +3, +1 +2, +1 +1, +1

0, +2 -1, +2 -2, +2 -2, +2 -4, +2 -5, +2 +5, +2 +4, +2 +3, +2 +2, +2 +1, +2

0, +3 -1, +3 -2, +3 -2, +3 -4, +3 -5, +3 +5, +3 +4, +3 +3, +3 +2, +3 +1, +3

0, -3 -1, -3 -2, -3 -3, -3 -4, -3 -5, -3 +5, -3 +4, -3 +3, -3 +2, -3 +1, -3

0, -2 -1, -2 -2, -2 -3, -2 -4, -2 -5, -2 +5, -2 +4, -2 +3, -2 +2, -2 +1, -2

0, -1 -1, -1 -2, -1 -3, -1 -4, -1 -5, -1 +5, -1 +4, -1 +3, -1 +2, -1 +1, -1

/
Here is another image of the entire tensor, with the slates having been arranged, with the zero
(yellow) entries forming an exterior half-shell on the top-posterior sides.

The remaining volume of this rank-3 tensor is 4*6*10 = (5-1)(7-1)(11-1) = 240, which again is in
agreement with the Euler Totient Function, as there exists 240 totatives to the number 5*7*11 = 385.

These tensor manipulations would apply to sets of greater size as well, for instance the set
S = {29, 31,43,61} would produce a rank-4 tensor with dimensions 29*31*43*61, and after the 3-D slates
are shuffled in four dimensional space, a rank-4 tensor of dimensions 28*30*42*60 would exist within it,
containing all solutions to the systems of congruences permitting a totative, with the discarded half-shell
(in four dimensions) containing all the zero entries.

The count of discarded solutions (volume of the n - dimensional half-shell) is given by the formula
i=n i=n
∏ si − ∏ (si − 1) . If we prohibit 2 from being an element of S, then the first product will always be odd and
i=1 i=1
possess exactly n factors with sn being the greatest factor; the second product will always be even
n
(being congruent to 0 mod 2 ) and always possess at least n + 1 factors (non-distinct factors) with its
greatest factor being less than sn . Thus, the difference of the products cannot be divided by sn .

/
Since both products have a different number of non-distinct factors, it is most likely not possible
for the difference of the products to be represented as a lesser rectangular rank- n tensor, such that:

i=n i=n i=n


∏ si − ∏ (si − 1 ) =/ ∏ (si − b), b ≥ 2 ,
i=1 i=1 i=1

and perhaps more generally,

i=n i=n i=n


∏ si − ∏ (si − a ) =/ ∏ (si − b), a ≥ 1, b ≥ 2
i=1 i=1 i=1

If this is true then the difference of two rank- n tensors, whose respective dimensions each differ
by some constant, cannot be another rank- n tensor whose dimensions are each shortened by another
constant when compared against the greater of the original two tensors, whose dimensions were distinct
prime numbers.

Thankfully the veracity of the above statements has no effect on our quest to solve the Andrica
Conjecture. Anyone who attempts to prove (or disprove) the above suppositions will immediately be
confronted by a generalized form of Fermat’s Last Theorem, involving the divisions of rank- n tensors into
smaller rank-n tensors.

Later on this in this section we shall see other glaring allusions to Fermat’s Last Theorem (such as the above equation),
and given Fermat’s extensive study of prime numbers, I have no doubt that this is how Fermat’s came to realize and perhaps even
prove his Last Theorem; however, most unfortunately, Fermat's Last Theorem is not the subject of this discourse, and must wait for
another time (and nor shall I further illuminate such allusions to Fermat’s Last Theorem).

On a final note concerning this subject, (since many who reviewed this work had asked me the same question) I do
sincerely believe that Fermat came to realize and also prove his Last Theorem using similar methods to what we shall explore in this
chapter, since Fermat himself extensively studied and published results on prime numbers and elementary number theory. It is very
likely that he attempted to count the number of prime gaps in specific integer sequences and came across the proposition:

i=n i=n i=n


∏ si − ∏ (si − a ) =/ ∏ (si − b), a ≥ 1, b ≥ 2 , and then proceeded to prove it. How he did so, I have no idea, nor do I care at
i=1 i=1 i=1
this time.

Due to the generic polynomic identities of these counting formulas, he then applied his proof to include to rank- n
rectangular tensors of any starting dimensions (composite numbers), instead of rectangular prismatic tensors with only prime
dimensions, and from there he examined the special case of rank- n cubic tensors. If there exists any “simple (elementary) and
marvelous proof” of his Last Theorem, this would be the path to navigate. That is the most I will entertain this subject in the main
body of this volume and I will not answer any correspondence concerning the issue until I release a separate paper on the subject.

I have had more than one person who began to review this work become completely subsumed by Fermat’s Last
Theorem , (one of whom called my cell phone past midnight on multiple occasions), and instead of focusing on Andrica Conjecture
and this paper, they came up empty-handed on their inquiries into Fermat’s Last Theorem and then abandoned the reading of this
paper due to their own self-inflicted exhaustion. Fermat’s Last Theorem has already been proven by Andrew Wiles (1995) and
requires no further attention.

/
1.6 Centrum ,The Midpoint between two consecutive totatives.

Centrum are the most crucial entities when attempting to understand the distribution of totatives.
For each pair of consecutive totatives, there exists a midpoint betwixt them, such that there exists
precisely as many centra as there also exists totatives.

Definition 1.5.1 Centrum, The Midpoint Between two consecutive Totatives, μ

Let μi = 12 (ti + ti+1 ) ∀t ∈ TotS.

If i = Φ (Ω[S]) then μ = 0 or Ω[S], depending on the context. It will most often be referred to as μ0
instead of μΦ[Ω(S)]

The plural form of centrum is “centra.”

Let us examine a set of eight centra generated by S = {2,3,5}, Ω (S) = 30

k = 10 (i − 1) + j
i, j 1 2 3 4 5 6 7 8 9 10

1 1 2 3 4 5 6 7 8 9 10

2 11 12 13 14 15 16 17 18 19 20

3 21 22 23 24 25 26 27 28 29 30

t ≡ a mod 2 ≡ b mod 3 ≡ c mod 5, a, b, c =/ 0 , TotS = {1, 7, 11, 13, 17, 19, 23, 29}

i, j 1 2 3 4 5 6 7 8 9 10

1 1 7

2 11 13 17 19

3 23 29

μi = 12 (ti + ti+1 ) ∀t; μ8 = 30 = 1


2
((1 + 30) − 29) , Set of μ = 4, 9, 12, 15, 18, 21, 26, 30

i, j 1 2 3 4 5 6 7 8 9 10

1 1 4 7 9

2 11 12 13 15 17 18 19

3 21 23 26 29 30

/
Definition 1.6.2a The Distance Function, Ψ

Ψ(μi ) = (ti+1 − ti )

This function acts on a centrum (the midpoint of two consecutive totatives), returning the difference
(distance) between the consecutive totatives.

Definition 1.6.2b Half-Distance Function, γ


The half-distance is usually more important than the full distance between totatives, in fact, it appears so
often that it deserves its own function: γ (μi ) = 12 (Ψ(μi )) .

Let M be the set of centra, and let us again examine the cetra for the set {2,3,5}:

TotS = {1, 7, 11, 13, 17, 19, 23, 29} M = {4, 9, 12, 15, 18, 21, 26, 30}
i, j 1 2 3 4 5 6 7 8 9 10

1 1 4 7 9

2 11 12 13 15 17 18 19

3 21 23 26 29 30

Ψ(μ1 ) = 6 , Ψ(μ2 ) = 4 . Ψ(μ3 ) = 2 , Ψ(μ4 ) = 4 , Ψ(μ5 ) = 2 , Ψ(μ6 ) = 4 , Ψ(μ7 ) = 6 , Ψ(μ8 or μ0 ) = 2


5
Definition 1.6.6a Exclusive Nucleus

An Exclusive Nucleus is a sequence of integers that exists between two consecutive totatives, which
excludes the pair of totatives bounding it. It is referenced by the system of congruences in respect to its
centrum.

An “Exclusive Nucleus” must always be stated in order to avoid confusion with an Inclusive Nucleus.

The length of an Exclusive Nucleus is equal to (Ψ(μ) − 2) when 2 ∈ S, with μ being its respective
centrum.

The length of an Exclusive Nucleus is equal to (Ψ(μ) − 1) when 2 ∈/ S.

/
Definition 1.6.6b Nucleus (Inclusive); Star

A Nucleus is a sequence of integers that exists between two consecutive totatives, and it includes the pair
of totatives bounding it. It is also referenced by the system of congruences in respect to its centrum.

The Inclusive Nucleus shall generally be called a “Nucleus,” and shall only be titled an “Inclusive
Nucleus,” when the context demands specificity.

The length of a Nucleus is equal to Ψ(μ) when 2 ∈ S, with μ being its respective centrum.

The length of a Nucleus is also equal to Ψ(μ) when 2 ∈/ S.

Later in this volume we shall define a Star and a Constellation, where an Inclusive Nucleus is
merely a type of Star. A centrum will be shown to be a type of Anchor, and an anchor will be shown to
exist inside the Stellar Core, a part of Star that cannot be altered without destroying the entity (leading to
direct numerical contradictions and violations of Fundamental Theorem of Arithmetic). These anchor
points are critical integers within a nucleus from which we write our system of congruences and alter our
system of congruences to yield and transition between different nuclei.

Definition 1.6.3 The System of Congruences for a Centrum

Let γ be equal to half the distance between two consecutive totatives (definition 1.5.2b). The system of
congruences for each centrum (with a difference of 2γ between its bounding totatives) takes the following
form:

μ ≡ a1 mod s1 ≡ a2 mod s2 ... ≡ an mod sn ∣ ∀a, − 12 (si − 1) ≤ ai ≤ + 1


2
(si − 1) and ∀a, ai =/ − γ or + γ

Notice that this definition restricts all remainders to being a distance within one-half of its
respective prime modulus from the centrum. So, if pi = 17 and there is difference between two
consecutive totatives equal to 10 (and thus γ = 5) , the choices of remainders are {-8, - 7, -6, -4, -3, -2, -1,
0, 1, 2, 3, 4, 6, 7, 8}. Notice that positive and negative 5 are excluded, since no prime in S is allowed to
divide the totatives located at μ − 5 and μ + 5 (otherwise they wouldn't be totatives!)

In this paper a system of congruences shall always adhere to this form unless expressly stated
otherwise.

/
1.7 The Elements of a Nucleus

The internal structure of a nucleus is the primary concern of this entire volume. It is from the
inspection of their forms that allows us to count and obtain the limits of the length of a nucleus.

Definition 1.7.0; Zeta Function

Let S be a set of primes.


Let k be an integer, and is bounded as follows: 1 ≤ k ≤ Ω (S)

Let si be the least prime in S that divides k , such that:

ξ (k) = si , or, ξ (k) = ∅ if no prime in S divides k .

Definition 1.7.1 Absent Primes

Absent Primes:
Let μ ≡ a1 mod s1 ≡ a2 mod s2 ... ≡ an mod sn ∣ ∀a, − 12 (si − 1) ≤ ai ≤ + 1
2
(si − 1) and ∀a, ai =/ − γ or + γ ,

If there exists a prime, si , such that the absolute value of its remainder, ai , is greater than γ (μ) , then si is
said to be absent, as this prime is not sieving any integer between the corresponding pair of consecutive
totatives that bind μ .

Let A be the set of Absent Primes.

Example (nucleus highlighted in blue)


Let S= {2,3,5,13} and let μ ≡ 0 mod 2, − 1 mod 3 ≡ 1 mod 5 ≡− 4 mod 13, Ψ(μ) = 6, μ = 386 ; A = {13}

k 79 80 81 82 83 84 85 86 87 88 89 90 91 92 93

ti . . μi . . ti+1

ξ (k) ∅ 2 5 2 3 2 ∅ 13

Since the prime number 13 sieves an integer outside the nucleus, it is said to be Absent.

/
Definition 1.7.2 Redundant Primes
Redundant Primes:
Let μ ≡ a1 mod s1 ≡ a2 mod s2 ... ≡ an mod sn ∣ ∀a, − 12 (si − 1) ≤ ai ≤ + 1
2
(si − 1) and ∀a, ai =/ − γ or + γ ,

Let K be a set of consecutive integers equal to some nucleus


Let C be a subset of K containing all integers divisible by some prime si .
Let |C| = h , h ≥ 1 ; if C is empty (h = 0) , then si is Absent (this is the formal definition of Absent).

If ξ (cx ) < si ∀x, 1 ≤ x ≤ h , then si is said to be Redundant.

Let R be the set of all redundant primes in S in respect to some centrum.

Example (nucleus highlighted in black)


Let S= {2,3,5, 7, 13} and let μ ≡ − 1 mod 2, − 1 mod 3 ≡ 2 mod 5 ≡ 0 mod 7 ≡− 2 mod 13,
Ψ(μ) = 8, μ = 1337
K = {1133, 1134, 1135, 1136, 1137 = μ , 1138, 1139, 1140, 1141}
C ⊂ K, C = {1135, 1140}, ξ (1135) = 3 < 5; ξ(1140) = 2 < 5 ∴ 5 ∈ R.

μ−4 μ−3 μ−2 μ−1 μ+1 μ+2 μ+3 μ+4 μ+8


k μ

ti . . . μi . . . ti

ξ (k) ∅ 2 3 2 7 2 13 2 ∅ 13 5

Since the prime number 5 fails to sieve any integers inside the nucleus that were not already sieved by a
lesser prime (2 or 3), 5 is redundant.

A more human definition (less technical) of Redundant Primes:

If there exists a prime, si , such that the absolute value of its remainder, ai , is less than γ (μ) , such that if
si were removed from S (and therefore the system of congruences for μ ), and the gap between the pair
of totatives bounding μ was not affected, then si is said to be redundant, since it did not sieve any
integers between (μ − γ + 1) and (μ + γ − 1) that the lesser primes in S had not already sieved.

Definition 1.7.3 Nugatory Primes


Nugatory Primes:
Let the union of the sets A and R be named the Nugatory Set, F.

F = A ∪ R.

These primes are said to be nugatory because they are without force, as they had no effect on the length
of the gap (length of the nucleus) between the two consecutive totatives.

/
Definition 1.7.4 Active Primes
Active Primes:
Let the difference of the sets S and F be named the Active Set, D.
D = S - F.

No prime from the active set can be removed from S without compromising the integrity of the gap
between the pair of consecutive totatives bounding the midpoint.

Definition 1.7.5 Belligerent Primes (this is the most important definition in this volume)
Belligerent Primes:
If si ∈ D, and si only sieves one integer between (μ − γ + 1) and (μ + γ − 1) then, by definition, si is the
only prime that divides that integer and therefore cannot be removed from S without forfeiting the integrity
of the gap.

Such a prime is said to be belligerent, let B be the set of belligerent primes.


B ⊆ D.

Let S= {2,3,5, 7, 13} and let μ ≡ − 1 mod 2, − 1 mod 3 ≡ 2 mod 5 ≡ 0 mod 7 ≡− 2 mod 13,
Ψ(μ) = 8, μ = 1337 , B = {7, 13} since 7 and 13 only sieve one integer each within the nucleus.

k μ−4 μ−3 μ−2 μ−1 μ μ+1 μ+2 μ+3 μ+4 μ+8

ti . . . μi . . . ti

ξ (k) ∅ 2 3 2 7 2 13 2 ∅ 13 5

Belligerent Primes are virtually the sole focus of this entire volume; as such, the formal definition shall
now be provided:

Let μ ≡ a1 mod s1 ≡ a2 mod s2 ... ≡ an mod sn ∣ ∀a, − 12 (si − 1) ≤ ai ≤ + 1


2
(si − 1) and ∀a, ai =/ − γ or + γ ,

Let K be a set of consecutive integers equal to some nucleus


Let C be a subset of K containing all integers divisible by some prime si .
Let |C| = h , h ≥ 1 ; if C is empty (h = 0) , then si is Absent.

If ∃x, 1 ≤ x ≤ h such that ξ (cx ) = si , and that...


ξ (cy ) < si ∀y, 1 ≤ y ≤ h, y =/ x , then si is said to be Belligerent.

Definition 1.7.6 Harmonious Primes


If si ∈ D, but si ∈/ B , then si sieves more than one integer between (μ − γ + 1) and (μ + γ − 1) , and by
definition, si is the only prime such that ξ (k) = si for at least two (or more) integers within the nucleus
therefore cannot be removed from S with forfeiting the integrity of the gap.

Such a prime is said to be Harmonious, let H be the set of harmonious primes.


B ⊆ D.
H=D-B

/
Here’s an example of all the primes in S being herded into their respective sets for some gap of
14 between two totatives. The first row contains the integer μ and all consecutive integers to the left and
right of μ, and the bottom row contains the least prime in S that divides each integer in the top row (zeta
function). I allowed the prime number 101 to divide the midpoint so it could be placed in the redundant
set.

S = {2, 3, 5, 17, 43, 101, 113, 127},


A = {127}, R = {101}, F = {101,127}, D = {2, 3, 5, 17, 43, 113}, B = {5, 17, 43, 113}, H = {2, 3}
μ ≡ 0 mod 2 ≡ 0 mod 3 ≡ − 1mod 5 ≡ 5 mod 17 ≡ 1 mod 43 ≡− 5 mod 113
μ ≡ 0 mod 101 ≡ f mod 127, 8 ≤ ∣f ∣ ≤ 50 , μ = m(182, 047, 854); 1 ≤ m ≤ 127

-8 -7 -6 -5 -4 -3 -2 -1 μ 1 +2 +3 +4 +5 +6 7 +8 +9 +10

2 ∅ 2 17 2 3 2 43 101 5 2 3 2 113 2 ∅ 2 3 2

Since μ ≡ f mod 127, 8 ≤ ∣f ∣ ≤ 62 , there are 110 variations in which this gap can occur using these
systems of congruences with the prime number 127 remaining absent, and a total of 125 variations
where 127 is either absent or redundant without impairing the integrity of the gap (not sieving μ ± 7 ).

The above table is shortened version of a Dirichlet-Resonance Array, which will be introduced later in this
volume. For now these shortened versions will be used in their place until the full version is required.

/
1.8 Nucleic Set

Definition 1.8.1a Nucleic Set; 1.8.1.b Galaxy

A nucleic set is a collection of all nuclei of the same length between 1 and Ω (S). The length of
the nuclei in question is given by the superscript, k , following 𝛭 (capitol Mu). Here is the formal definition:

Let Sn be a set of primes and let Φ(Ω (Sn)) be the count of totatives to Ω (Sn ).

Let Φ(Ω (Sn)) be represented by the variable c , such that c = Φ(Ω (Sn)).

Let k be a positive integer.

For each ui , 1 ≤ i ≤ c , such that Ψ(μi ) = k , let it be placed in the set 𝛭k.

Similar to the Totative Set, the union of all nucleic sets forms the Galaxy of S.

Definition 1.8.2 Counting Function for a Nucleic Set

Let ϖ (𝛭k) be the counting function that determines the size of 𝛭k, such that ϖ (𝛭k) = |𝛭k|

See next page for a detailed example of nucleic sets for the set {2,3,5,7}.

/
Example of Nucleic Sets
n = 4 ; Pn = {2,3,5,7}
Ω(Pn) = 210
Φ Ω(Pn) = 48

Principal Dirichlet Representation of Ω(Pn) (totatives = 1, non-totatives = 0)


x = (j − 1) + i , where j is the vertical index and i is the horizontal index.
The Galaxy of Pn.
i, j 1 2 3 4 5 6 7 8 9 10 11 12 13 14 15

1 1 0 0 0 0 0 0 0 0 0 1 0 1 0 0

2 0 1 0 1 0 0 0 1 0 0 0 0 0 1 0

3 1 0 0 0 0 0 1 0 0 0 1 0 1 0 0

4 0 1 0 0 0 0 0 1 0 0 0 0 0 1 0

5 1 0 0 0 0 0 1 0 0 0 1 0 1 0 0

6 0 0 0 1 0 0 0 1 0 0 0 0 0 1 0

7 0 0 0 0 0 0 1 0 0 0 1 0 1 0 0

8 0 1 0 1 0 0 0 1 0 0 0 0 0 0 0

9 1 0 0 0 0 0 1 0 0 0 1 0 0 0 0

10 0 1 0 1 0 0 0 1 0 0 0 0 0 1 0

11 1 0 0 0 0 0 1 0 0 0 0 0 1 0 0

12 0 1 0 1 0 0 0 1 0 0 0 0 0 1 0

13 1 0 0 0 0 0 1 0 0 0 1 0 1 0 0

14 0 1 0 1 0 0 0 0 0 0 0 0 0 1 0

Count of gaps = 48 = ϕ Ω(Pn)


ϖ (M2) = 15
ϖ (M4) = 15
ϖ (M6) = 14
ϖ (M8) = 2
ϖ (M10) = 2

k=5
∑ ϖ(M ) = 15 + 15 + 14 + 2 + 2 = 48 = ϕ Ω(Pn)
2k
k=1

k=5
( 2k
)
∑ (2k)ϖ(M ) = (2)(15) + (4)(15) + (6)(14) + (8)(2) + (10(2) = 30 + 60 + 84 + 16 + 20 = 210 = Ω(Pn)
k=1

/
There are fifteen nuclei with a length of 4; the fifteen nuclei of length 4 are highlighted below (the
highlight alternates between orange and blue for ease of viewing):

Pn = {2,3,5,7}
Ω(Pn) = 210
Φ Ω(Pn) = 48

M4 = {15, 21, 39, 45, 69, 81, 99, 105, 111, 129, 141, 165, 171, 189, 195}

Principal Dirichlet Representation of Ω(Pn) (totatives = 1, non-totatives = 0)

x = (j − 1) + i , where j is the vertical index and i is the horizontal index.


i, j 1 2 3 4 5 6 7 8 9 10 11 12 13 14 15

1 1 0 0 0 0 0 0 0 0 0 1 0 1 0 0

2 0 1 0 1 0 0 0 1 0 0 0 0 0 1 0

3 1 0 0 0 0 0 1 0 0 0 1 0 1 0 0

4 0 1 0 0 0 0 0 1 0 0 0 0 0 1 0

5 1 0 0 0 0 0 1 0 0 0 1 0 1 0 0

6 0 0 0 1 0 0 0 1 0 0 0 0 0 1 0

7 0 0 0 0 0 0 1 0 0 0 1 0 1 0 0

8 0 1 0 1 0 0 0 1 0 0 0 0 0 0 0

9 1 0 0 0 0 0 1 0 0 0 1 0 0 0 0

10 0 1 0 1 0 0 0 1 0 0 0 0 0 1 0

11 1 0 0 0 0 0 1 0 0 0 0 0 1 0 0

12 0 1 0 1 0 0 0 1 0 0 0 0 0 1 0

13 1 0 0 0 0 0 1 0 0 0 1 0 1 0 0

14 0 1 0 1 0 0 0 0 0 0 0 0 0 1 0

/
There are fourteen nuclei with a length of 6, the fourteen nuclei of length 6 are highlighted below (the
highlight alternates between orange and blue for ease of viewing):

Pn = {2,3,5,7}
Ω(Pn) = 210
Φ Ω(Pn) = 48

M6 = {26, 34, 50, 56, 64, 76, 86, 124, 134, 146, 154, 160, 176, 184}

Principal Dirichlet Representation of Ω(Pn) (totatives = 1, non-totatives = 0)

x = (j − 1) + i , where j is the vertical index and i is the horizontal index.


i, j 1 2 3 4 5 6 7 8 9 10 11 12 13 14 15

1 1 0 0 0 0 0 0 0 0 0 1 0 1 0 0

2 0 1 0 1 0 0 0 1 0 0 0 0 0 1 0

3 1 0 0 0 0 0 1 0 0 0 1 0 1 0 0

4 0 1 0 0 0 0 0 1 0 0 0 0 0 1 0

5 1 0 0 0 0 0 1 0 0 0 1 0 1 0 0

6 0 0 0 1 0 0 0 1 0 0 0 0 0 1 0

7 0 0 0 0 0 0 1 0 0 0 1 0 1 0 0

8 0 1 0 1 0 0 0 1 0 0 0 0 0 0 0

9 1 0 0 0 0 0 1 0 0 0 1 0 0 0 0

10 0 1 0 1 0 0 0 1 0 0 0 0 0 1 0

11 1 0 0 0 0 0 1 0 0 0 0 0 1 0 0

12 0 1 0 1 0 0 0 1 0 0 0 0 0 1 0

13 1 0 0 0 0 0 1 0 0 0 1 0 1 0 0

14 0 1 0 1 0 0 0 0 0 0 0 0 0 1 0

/
There are two nuclei with a length of 8 and 10, the four nuclei of length 8 and 10 are highlighted below.
(the blue highlight is length of 8 and the red highlight is length of 10):

Pn = {2,3,5,7}
Ω(Pn) = 210
Φ Ω(Pn) = 48

M8 = {93, 117} (additive inverses modulo 210)


M10 = {6, 204} (additive inverses modulo 210)

Principal Dirichlet Representation of Ω(Pn) (totatives = 1, non-totatives = 0)

x = (j − 1) + i , where j is the vertical index and i is the horizontal index.


i, j 1 2 3 4 5 6 7 8 9 10 11 12 13 14 15

1 1 0 0 0 0 0 0 0 0 0 1 0 1 0 0

2 0 1 0 1 0 0 0 1 0 0 0 0 0 1 0

3 1 0 0 0 0 0 1 0 0 0 1 0 1 0 0

4 0 1 0 0 0 0 0 1 0 0 0 0 0 1 0

5 1 0 0 0 0 0 1 0 0 0 1 0 1 0 0

6 0 0 0 1 0 0 0 1 0 0 0 0 0 1 0

7 0 0 0 0 0 0 1 0 0 0 1 0 1 0 0

8 0 1 0 1 0 0 0 1 0 0 0 0 0 0 0

9 1 0 0 0 0 0 1 0 0 0 1 0 0 0 0

10 0 1 0 1 0 0 0 1 0 0 0 0 0 1 0

11 1 0 0 0 0 0 1 0 0 0 0 0 1 0 0

12 0 1 0 1 0 0 0 1 0 0 0 0 0 1 0

13 1 0 0 0 0 0 1 0 0 0 1 0 1 0 0

14 0 1 0 1 0 0 0 0 0 0 0 0 0 1 0

/
There are fifteen nuclei with a length of 2, the fifteen nuclei of length 2 are highlighted below:

Pn = {2,3,5,7}
Ω(Pn) = 210
Φ Ω(Pn) = 48

M2 = {0 or 210, 12, 18, 30, 42, 60, 72, 102, 108, 138, 150, 168, 180, 192, 198}

Principal Dirichlet Representation of Ω(Pn) (totatives = 1, non-totatives = 0)

x = (j − 1) + i , where j is the vertical index and i is the horizontal index.


i, j 1 2 3 4 5 6 7 8 9 10 11 12 13 14 15

1 1 0 0 0 0 0 0 0 0 0 1 0 1 0 0

2 0 1 0 1 0 0 0 1 0 0 0 0 0 1 0

3 1 0 0 0 0 0 1 0 0 0 1 0 1 0 0

4 0 1 0 0 0 0 0 1 0 0 0 0 0 1 0

5 1 0 0 0 0 0 1 0 0 0 1 0 1 0 0

6 0 0 0 1 0 0 0 1 0 0 0 0 0 1 0

7 0 0 0 0 0 0 1 0 0 0 1 0 1 0 0

8 0 1 0 1 0 0 0 1 0 0 0 0 0 0 0

9 1 0 0 0 0 0 1 0 0 0 1 0 0 0 0

10 0 1 0 1 0 0 0 1 0 0 0 0 0 1 0

11 1 0 0 0 0 0 1 0 0 0 0 0 1 0 0

12 0 1 0 1 0 0 0 1 0 0 0 0 0 1 0

13 1 0 0 0 0 0 1 0 0 0 1 0 1 0 0

14 0 1 0 1 0 0 0 0 0 0 0 0 0 1 0

/
1.9 Using Geometry to Count the Size of Nucleic Set
General Formula

In the same manner we used the geometry of rank- n tensors to count the number of totatives to
an integer, we shall count the size of Nucleic sets of a predetermined length, k . It will revealed that the
general counting formula for counting the size of nucleic sets is simply an exercise in standard binomial
expansion, albeit in disguise! This is because the size of nucleic set is simply the result of adding and
subtracting the successive exterior shells of a tensor.

The set of sieving primes used to generate the nucleic sets shall always contain the prime
numbers {2,3}; however, no other prime in the sieving set shall be smaller than 12 k . Under these
conditions we shall yield the general formula for counting the size of a nucleic set (when primes smaller
than 12 k are in the sieving set, irregularities occur).

First we shall obtain the formula for counting the amount of nuclei of length 2. The centrum of
such a nucleus is always divisible by 6, such that μ ≡ 0 mod 2 ≡ 0 mod 3 , assuming that {2,3} ∈ S.

μ ≡ 0 mod 2 ≡ 0 mod 3 ≡ a i mod si ∀i, 3 ≤ i ≤ n, ai =/ ± 1

k μ−1 6m μ+1 μ+2

ti μ ti+1

ξ (k) ∅ 2 ∅

/
Lemma 1.9.1.1; The size of M2 , S = {2, 3, 7, 11} and |S| = 4, {2,3} ∈ S

Let S = {2, 3, 7, 11}

∀μ ∈ M2, μ ≡ 0 mod 2 ≡ 0 mod 3 ≡ a1 mod 7 ≡ a2 mod 11; a1 , a2 =/ ± 1

i=n
ϖ (M2) = ∏ (si − 2) = (7 − 2)(11 − 2) = (5)(9) = 45
i=3

Proof:

Let us compile all possible systems of congruences in respect to 7 and 11 into a matrix whose
dimensions are 7x11, Ω (S) = 77 = (7)(11)

k ≡ a mod 7 ≡ b mod 11 ; − 3 ≤ a ≤+ 3, − 5 ≤ b ≤+ 5, a, b ∈ Z (the directions of positive and


negative entries are reversed in these matrices, the reason for this shall be revealed later).

k ≡ 0 mod 6
+5, -3 +4, -3 +3, -3 +2, -3 +1, -3 0, -3 -1, -3 -2, -3 -3, -3 -4, -3 -5, -3

+5, -2 +4, -2 +3, -2 +2, -2 +1, -2 0, -2 -1, -2 -2, -2 -3, -2 -4, -2 -5, -2

+5, -1 +4, -1 +3, -1 +2, -1 +1, -1 0, -1 -1, -1 -2, -1 -3, -1 -4, -1 -5, -1

+5, 0 +4, 0 +3, 0 +2, 0 +1, 0 0, 0 -1, 0 -2, 0 -2, 0 -4, 0 -5, 0

+5, +1 +4, +1 +3, +1 +2, +1 +1, +1 0, +1 -1, +1 -2, +1 -2, +1 -4, +1 -5, +1

+5, +2 +4, +2 +3, +2 +2, +2 +1, +2 0, +2 -1, +2 -2, +2 -2, +2 -4, +2 -5, +2

+5, +3 +4, +3 +3, +3 +2, +3 +1, +3 0, +3 -1, +3 -2, +3 -2, +3 -4, +3 -5, +3

All of the above 77 entries containing a positive or negative 1 represent systems of congruences
which shall not yield a nucleus of length 2, they are highlighted in yellow. Let us rearrange them to place
them on the margins of the matrix, which yields an interior sub-matrix of dimensions 5x9 = (7-2)(11-2).

/
k ≡ a mod 7 ≡ b mod 11 ; − 3 ≤ a ≤+ 3, − 5 ≤ b ≤+ 5, a, b ∈ Z
+1, -1 +4, -1 +3, -1 +2, -1 +5, -1 0, -1 -5, -1 -2, -1 -3, -1 -4, -1 -1, -1

+1, -2 +4, -2 +3, -2 +2, -2 +5, -2 0, -2 -5, -2 -2, -2 -3, -2 -4, -2 -1, -2

+1, -3 +4, -3 +3, -3 +2, -3 +5, -3 0, -3 -5, -3 -2, -3 -3, -3 -4, -3 -1, -3

+1, 0 +4, 0 +3, 0 +2, 0 +5, 0 0, 0 -5, 0 -2, 0 -3, 0 -4, 0 -1, 0

+1, +3 +4, +3 +3, +3 +2, +3 +5, +3 0, +3 -5, +3 -2, +3 -3, +3 -4, +3 -1, +3

+1, +2 +4, +2 +3, +2 +2, +2 +5, +2 0, +2 -5, +2 -2, +2 -3, +2 -4, +2 -1, +2

+1, +1 +4, +1 +3, +1 +2, +1 +5, +1 0, +1 -5, +1 -2, +1 -2, +1 -4, +1 -1, +1

This maneuver could have been performed for any of four primes containing {2,3}, therefore the
i=4
size of M2 equals ∏ (si − 2) .
i=3
Q.E.D

Lemma 1.8.1.2; The size of M2 , S = {2, 3, 5, 7, 11} and |S| ≥ 5, {2,3} ∈ S

Let S = {2, 3, 5, 7, 11}

∀μ ∈ M2, μ ≡ 0 mod 2 ≡ 0 mod 3 ≡ a1 mod 7 ≡ a2 mod 11 ≡ a3 mod 5; a1 , a2 , a3 =/ ± 1

i=5
ϖ (M2) = ∏ (si − 2) = (5 − 2)(7 − 2)(11 − 2) = (3)(5)(9) = 45 = 135
i=3

Proof:

Let us compile all possible systems of congruences in respect to 7 and 11 into a matrix whose
dimensions are 7x11, and then make 5 slates of that matrix to form a rank-3 tensor.

/
k ≡ − 2 mod 5 ≡ a mod 7 ≡ b mod 11 ; − 3 ≤ a ≤+ 3, − 5 ≤ b ≤+ 5, a, b ∈ Z
+1, -1 +4, -1 +3, -1 +2, -1 +5, -1 0, -1 -5, -1 -2, -1 -3, -1 -4, -1 -1, -1

+1, -2 +4, -2 +3, -2 +2, -2 +5, -2 0, -2 -5, -2 -2, -2 -3, -2 -4, -2 -1, -2

+1, -3 +4, -3 +3, -3 +2, -3 +5, -3 0, -3 -5, -3 -2, -3 -3, -3 -4, -3 -1, -3

+1, 0 +4, 0 +3, 0 +2, 0 +5, 0 0, 0 -5, 0 -2, 0 -3, 0 -4, 0 -1, 0

+1, +3 +4, +3 +3, +3 +2, +3 +5, +3 0, +3 -5, +3 -2, +3 -3, +3 -4, +3 -1, +3

+1, +2 +4, +2 +3, +2 +2, +2 +5, +2 0, +2 -5, +2 -2, +2 -3, +2 -4, +2 -1, +2

+1, +1 +4, +1 +3, +1 +2, +1 +5, +1 0, +1 -5, +1 -2, +1 -2, +1 -4, +1 -1, +1

k ≡ − 1 mod 5 ≡ a mod 7 ≡ b mod 11 ; − 3 ≤ a ≤+ 3, − 5 ≤ b ≤+ 5, a, b ∈ Z


+1, -1 +4, -1 +3, -1 +2, -1 +5, -1 0, -1 -5, -1 -2, -1 -3, -1 -4, -1 -1, -1

+1, -2 +4, -2 +3, -2 +2, -2 +5, -2 0, -2 -5, -2 -2, -2 -3, -2 -4, -2 -1, -2

+1, -3 +4, -3 +3, -3 +2, -3 +5, -3 0, -3 -5, -3 -2, -3 -3, -3 -4, -3 -1, -3

+1, 0 +4, 0 +3, 0 +2, 0 +5, 0 0, 0 -5, 0 -2, 0 -3, 0 -4, 0 -1, 0

+1, +3 +4, +3 +3, +3 +2, +3 +5, +3 0, +3 -5, +3 -2, +3 -3, +3 -4, +3 -1, +3

+1, +2 +4, +2 +3, +2 +2, +2 +5, +2 0, +2 -5, +2 -2, +2 -3, +2 -4, +2 -1, +2

+1, +1 +4, +1 +3, +1 +2, +1 +5, +1 0, +1 -5, +1 -2, +1 -2, +1 -4, +1 -1, +1

k ≡ 0 mod 5 ≡ a mod 7 ≡ b mod 11 ; − 3 ≤ a ≤+ 3, − 5 ≤ b ≤+ 5, a, b ∈ Z


+1, -1 +4, -1 +3, -1 +2, -1 +5, -1 0, -1 -5, -1 -2, -1 -3, -1 -4, -1 -1, -1

+1, -2 +4, -2 +3, -2 +2, -2 +5, -2 0, -2 -5, -2 -2, -2 -3, -2 -4, -2 -1, -2

+1, -3 +4, -3 +3, -3 +2, -3 +5, -3 0, -3 -5, -3 -2, -3 -3, -3 -4, -3 -1, -3

+1, 0 +4, 0 +3, 0 +2, 0 +5, 0 0, 0 -5, 0 -2, 0 -3, 0 -4, 0 -1, 0

+1, +3 +4, +3 +3, +3 +2, +3 +5, +3 0, +3 -5, +3 -2, +3 -3, +3 -4, +3 -1, +3

+1, +2 +4, +2 +3, +2 +2, +2 +5, +2 0, +2 -5, +2 -2, +2 -3, +2 -4, +2 -1, +2

+1, +1 +4, +1 +3, +1 +2, +1 +5, +1 0, +1 -5, +1 -2, +1 -2, +1 -4, +1 -1, +1

k ≡+ 1 mod 5 ≡ a mod 7 ≡ b mod 11 ; − 3 ≤ a ≤+ 3, − 5 ≤ b ≤+ 5, a, b ∈ Z


+1, -1 +4, -1 +3, -1 +2, -1 +5, -1 0, -1 -5, -1 -2, -1 -3, -1 -4, -1 -1, -1

+1, -2 +4, -2 +3, -2 +2, -2 +5, -2 0, -2 -5, -2 -2, -2 -3, -2 -4, -2 -1, -2

+1, -3 +4, -3 +3, -3 +2, -3 +5, -3 0, -3 -5, -3 -2, -3 -3, -3 -4, -3 -1, -3

+1, 0 +4, 0 +3, 0 +2, 0 +5, 0 0, 0 -5, 0 -2, 0 -3, 0 -4, 0 -1, 0

+1, +3 +4, +3 +3, +3 +2, +3 +5, +3 0, +3 -5, +3 -2, +3 -3, +3 -4, +3 -1, +3

+1, +2 +4, +2 +3, +2 +2, +2 +5, +2 0, +2 -5, +2 -2, +2 -3, +2 -4, +2 -1, +2

+1, +1 +4, +1 +3, +1 +2, +1 +5, +1 0, +1 -5, +1 -2, +1 -2, +1 -4, +1 -1, +1

k ≡+ 2 mod 5 ≡ a mod 7 ≡ b mod 11 ; − 3 ≤ a ≤+ 3, − 5 ≤ b ≤+ 5, a, b ∈ Z


+1, -1 +4, -1 +3, -1 +2, -1 +5, -1 0, -1 -5, -1 -2, -1 -3, -1 -4, -1 -1, -1

+1, -2 +4, -2 +3, -2 +2, -2 +5, -2 0, -2 -5, -2 -2, -2 -3, -2 -4, -2 -1, -2

+1, -3 +4, -3 +3, -3 +2, -3 +5, -3 0, -3 -5, -3 -2, -3 -3, -3 -4, -3 -1, -3

+1, 0 +4, 0 +3, 0 +2, 0 +5, 0 0, 0 -5, 0 -2, 0 -3, 0 -4, 0 -1, 0

+1, +3 +4, +3 +3, +3 +2, +3 +5, +3 0, +3 -5, +3 -2, +3 -3, +3 -4, +3 -1, +3

+1, +2 +4, +2 +3, +2 +2, +2 +5, +2 0, +2 -5, +2 -2, +2 -3, +2 -4, +2 -1, +2

+1, +1 +4, +1 +3, +1 +2, +1 +5, +1 0, +1 -5, +1 -2, +1 -2, +1 -4, +1 -1, +1

/
Here is an image of the rank 3 tensor:

As we did earlier when counting totatives, we shall arrange the slates to produce a
continuous interior of valid systems of congruences, with the rejected solutions forming an exterior shell.

/
k ≡ − 1 mod 5 ≡ a mod 7 ≡ b mod 11 ; − 3 ≤ a ≤+ 3, − 5 ≤ b ≤+ 5, a, b ∈ Z
+1, -1 +4, -1 +3, -1 +2, -1 +5, -1 0, -1 -5, -1 -2, -1 -3, -1 -4, -1 -1, -1

+1, -2 +4, -2 +3, -2 +2, -2 +5, -2 0, -2 -5, -2 -2, -2 -3, -2 -4, -2 -1, -2

+1, -3 +4, -3 +3, -3 +2, -3 +5, -3 0, -3 -5, -3 -2, -3 -3, -3 -4, -3 -1, -3

+1, 0 +4, 0 +3, 0 +2, 0 +5, 0 0, 0 -5, 0 -2, 0 -3, 0 -4, 0 -1, 0

+1, +3 +4, +3 +3, +3 +2, +3 +5, +3 0, +3 -5, +3 -2, +3 -3, +3 -4, +3 -1, +3

+1, +2 +4, +2 +3, +2 +2, +2 +5, +2 0, +2 -5, +2 -2, +2 -3, +2 -4, +2 -1, +2

+1, +1 +4, +1 +3, +1 +2, +1 +5, +1 0, +1 -5, +1 -2, +1 -2, +1 -4, +1 -1, +1

k ≡ − 2 mod 5 ≡ a mod 7 ≡ b mod 11 ; − 3 ≤ a ≤+ 3, − 5 ≤ b ≤+ 5, a, b ∈ Z


+1, -1 +4, -1 +3, -1 +2, -1 +5, -1 0, -1 -5, -1 -2, -1 -3, -1 -4, -1 -1, -1

+1, -2 +4, -2 +3, -2 +2, -2 +5, -2 0, -2 -5, -2 -2, -2 -3, -2 -4, -2 -1, -2

+1, -3 +4, -3 +3, -3 +2, -3 +5, -3 0, -3 -5, -3 -2, -3 -3, -3 -4, -3 -1, -3

+1, 0 +4, 0 +3, 0 +2, 0 +5, 0 0, 0 -5, 0 -2, 0 -3, 0 -4, 0 -1, 0

+1, +3 +4, +3 +3, +3 +2, +3 +5, +3 0, +3 -5, +3 -2, +3 -3, +3 -4, +3 -1, +3

+1, +2 +4, +2 +3, +2 +2, +2 +5, +2 0, +2 -5, +2 -2, +2 -3, +2 -4, +2 -1, +2

+1, +1 +4, +1 +3, +1 +2, +1 +5, +1 0, +1 -5, +1 -2, +1 -2, +1 -4, +1 -1, +1

k ≡ 0 mod 5 ≡ a mod 7 ≡ b mod 11 ; − 3 ≤ a ≤+ 3, − 5 ≤ b ≤+ 5, a, b ∈ Z


+1, -1 +4, -1 +3, -1 +2, -1 +5, -1 0, -1 -5, -1 -2, -1 -3, -1 -4, -1 -1, -1

+1, -2 +4, -2 +3, -2 +2, -2 +5, -2 0, -2 -5, -2 -2, -2 -3, -2 -4, -2 -1, -2

+1, -3 +4, -3 +3, -3 +2, -3 +5, -3 0, -3 -5, -3 -2, -3 -3, -3 -4, -3 -1, -3

+1, 0 +4, 0 +3, 0 +2, 0 +5, 0 0, 0 -5, 0 -2, 0 -3, 0 -4, 0 -1, 0

+1, +3 +4, +3 +3, +3 +2, +3 +5, +3 0, +3 -5, +3 -2, +3 -3, +3 -4, +3 -1, +3

+1, +2 +4, +2 +3, +2 +2, +2 +5, +2 0, +2 -5, +2 -2, +2 -3, +2 -4, +2 -1, +2

+1, +1 +4, +1 +3, +1 +2, +1 +5, +1 0, +1 -5, +1 -2, +1 -2, +1 -4, +1 -1, +1

k ≡+ 2 mod 5 ≡ a mod 7 ≡ b mod 11 ; − 3 ≤ a ≤+ 3, − 5 ≤ b ≤+ 5, a, b ∈ Z


+1, -1 +4, -1 +3, -1 +2, -1 +5, -1 0, -1 -5, -1 -2, -1 -3, -1 -4, -1 -1, -1

+1, -2 +4, -2 +3, -2 +2, -2 +5, -2 0, -2 -5, -2 -2, -2 -3, -2 -4, -2 -1, -2

+1, -3 +4, -3 +3, -3 +2, -3 +5, -3 0, -3 -5, -3 -2, -3 -3, -3 -4, -3 -1, -3

+1, 0 +4, 0 +3, 0 +2, 0 +5, 0 0, 0 -5, 0 -2, 0 -3, 0 -4, 0 -1, 0

+1, +3 +4, +3 +3, +3 +2, +3 +5, +3 0, +3 -5, +3 -2, +3 -3, +3 -4, +3 -1, +3

+1, +2 +4, +2 +3, +2 +2, +2 +5, +2 0, +2 -5, +2 -2, +2 -3, +2 -4, +2 -1, +2

+1, +1 +4, +1 +3, +1 +2, +1 +5, +1 0, +1 -5, +1 -2, +1 -2, +1 -4, +1 -1, +1

k ≡+ 1 mod 5 ≡ a mod 7 ≡ b mod 11 ; − 3 ≤ a ≤+ 3, − 5 ≤ b ≤+ 5, a, b ∈ Z


+1, -1 +4, -1 +3, -1 +2, -1 +5, -1 0, -1 -5, -1 -2, -1 -3, -1 -4, -1 -1, -1

+1, -2 +4, -2 +3, -2 +2, -2 +5, -2 0, -2 -5, -2 -2, -2 -3, -2 -4, -2 -1, -2

+1, -3 +4, -3 +3, -3 +2, -3 +5, -3 0, -3 -5, -3 -2, -3 -3, -3 -4, -3 -1, -3

+1, 0 +4, 0 +3, 0 +2, 0 +5, 0 0, 0 -5, 0 -2, 0 -3, 0 -4, 0 -1, 0

+1, +3 +4, +3 +3, +3 +2, +3 +5, +3 0, +3 -5, +3 -2, +3 -3, +3 -4, +3 -1, +3

+1, +2 +4, +2 +3, +2 +2, +2 +5, +2 0, +2 -5, +2 -2, +2 -3, +2 -4, +2 -1, +2

+1, +1 +4, +1 +3, +1 +2, +1 +5, +1 0, +1 -5, +1 -2, +1 -2, +1 -4, +1 -1, +1

/
A pictorial image showing the rearranged slates, containing a smaller rank-3 tensor of dimensions 3*5*9

The resulting volume of the interior rank-3 tensor is 3*5*9 = (5-2)(7-2)(11-2). Again we could
performed this manipulation of the slates from any set of 5 primes, with {2,3} ∈ S, or more generally for
any set of primes whatsoever, since the solution set to the systems of congruences yielding a nucleus of
length 2 for a set of n primes always producing a rank- (n − 2) tensor whose respective dimensions are
two less than each of the primes in S.
i=n
Thus the size of M2 = ∏ (si − 2) assuming that {2,3} ∈ S.
i=3
QED

/
Lemma 1.9.1.2; The size of M4 , S = {2, 3, 7, 11} and |S| = n , {2,3} ∈ S

Let S = {2, 3, 7, 11}

∀μ ∈ M4, μ ≡ 1 mod 2 ≡ 0 mod 3 ≡ a1 mod 7 ≡ a2 mod 11; a1 , a2 =/ ± 2


The centrum of a Nucleus of length 4 must be congruent to 3 mod 6.

i=n
ϖ (M4) = ∏ (si − 2) = (7 − 2)(11 − 2) = (5)(9) = 45
i=3

Proof:

Let us compile all possible systems of congruences in respect to 7 and 11 into a matrix whose
dimensions are 7x11.

k ≡ a mod 7 ≡ b mod 11 ; − 3 ≤ a ≤+ 3, − 5 ≤ b ≤+ 5, a, b ∈ Z (the directions of positive and


negative entries are reversed in these matrices, the reason for this shall be revealed later).

k ≡ 0 mod 6
+5, -3 +4, -3 +3, -3 +2, -3 +1, -3 0, -3 -1, -3 -2, -3 -3, -3 -4, -3 -5, -3

+5, -2 +4, -2 +3, -2 +2, -2 +1, -2 0, -2 -1, -2 -2, -2 -3, -2 -4, -2 -5, -2

+5, -1 +4, -1 +3, -1 +2, -1 +1, -1 0, -1 -1, -1 -2, -1 -3, -1 -4, -1 -5, -1

+5, 0 +4, 0 +3, 0 +2, 0 +1, 0 0, 0 -1, 0 -2, 0 -2, 0 -4, 0 -5, 0

+5, +1 +4, +1 +3, +1 +2, +1 +1, +1 0, +1 -1, +1 -2, +1 -2, +1 -4, +1 -5, +1

+5, +2 +4, +2 +3, +2 +2, +2 +1, +2 0, +2 -1, +2 -2, +2 -2, +2 -4, +2 -5, +2

+5, +3 +4, +3 +3, +3 +2, +3 +1, +3 0, +3 -1, +3 -2, +3 -2, +3 -4, +3 -5, +3

All of the above 77 entries containing a positive or negative 2 represent systems of congruences
which shall not yield a nucleus of length 4, they are highlighted in yellow. They can also be arranged to
place them on the margins of the matrix, which yields an interior sub-matrix of dimensions 5x9 =
(7-2)(11-2).

In an identical manner to the previous lemma, this would also apply to rank- (n − 2) tensors, such
that:

i=n
ϖ (M4) = ∏ (si − 2)
i=3

Therefore there exists a bijection between Nucleic Sets of lengths 2 and 4, which supports the Hardy and
Littlewood Conjecture on the distribution of twin and cousin primes.

/
Lemma 1.9.1.3; The size of M6 , S = {2, 3, 7, 11} and |S| = 4 , {2,3} ∈ S

Let S = {2, 3, 7, 11}

∀μ ∈ M6, μ ≡ 0 mod 2 ≡− 1mod 3 ≡ a1 mod 7 ≡ a2 mod 11; a1 , a2 =/ ± 3

Either 7 or 11 must be belligerent, such that it sieves (μ − 1)

k μ−3 μ−1 μ μ+3

ti ui ti+1

ξ (k) ∅ 2 bx 2 3 2 ∅

( )
i=n i=n
ϖ (M6) = 2 ∏ (si − 2) − ∏(si − 3)
i=3 i=3

Proof:

Let us compile all possible systems of congruences in respect to 7 and 11 into a matrix whose
dimensions are 7x11.

k ≡ a mod 7 ≡ b mod 11 ; − 3 ≤ a ≤+ 3, − 5 ≤ b ≤+ 5, a, b ∈ Z (the directions of positive and


negative entries are reversed in these matrices, the reason for this shall be revealed later).

k ≡ 2 mod 6
+5, -3 +4, -3 +3, -3 +2, -3 +1, -3 0, -3 -1, -3 -2, -3 -3, -3 -4, -3 -5, -3

+5, -2 +4, -2 +3, -2 +2, -2 +1, -2 0, -2 -1, -2 -2, -2 -3, -2 -4, -2 -5, -2

+5, -1 +4, -1 +3, -1 +2, -1 +1, -1 0, -1 -1, -1 -2, -1 -3, -1 -4, -1 -5, -1

+5, 0 +4, 0 +3, 0 +2, 0 +1, 0 0, 0 -1, 0 -2, 0 -2, 0 -4, 0 -5, 0

+5, +1 +4, +1 +3, +1 +2, +1 +1, +1 0, +1 -1, +1 -2, +1 -2, +1 -4, +1 -5, +1

+5, +2 +4, +2 +3, +2 +2, +2 +1, +2 0, +2 -1, +2 -2, +2 -2, +2 -4, +2 -5, +2

+5, +3 +4, +3 +3, +3 +2, +3 +1, +3 0, +3 -1, +3 -2, +3 -2, +3 -4, +3 -5, +3

All of the above 77 entries containing a positive or negative 3 represent systems of congruences
which shall not yield a nucleus of length 4, they are highlighted in yellow. They can also be arranged to
place them on the margins of the matrix, which yields an interior sub-matrix of dimensions 5x9 =
(7-2)(11-2).

Let us arrange them so that solutions that divide the endpoints of the nucleus are discarded.

/
k ≡ 2 mod 6
+3, -3 +4, -3 +5, -3 +2, -3 +1, -3 0, -3 -1, -3 -2, -3 -5, -3 -4, -3 -3, -3

+3, -2 +4, -2 +5, -2 +2, -2 +1, -2 0, -2 -1, -2 -2, -2 -5, -2 -4, -2 -3, -2

+3, -1 +4, -1 +5, -1 +2, -1 +1, -1 0, -1 -1, -1 -2, -1 -5, -1 -4, -1 -3, -1

+3, 0 +4, 0 +5, 0 +2, 0 +1, 0 0, 0 -1, 0 -2, 0 -5, 0 -4, 0 -2, 0

+3, +1 +4, +1 +5, +1 +2, +1 +1, +1 0, +1 -1, +1 -2, +1 -5, +1 -4, +1 -2, +1

+3, +2 +4, +2 +5, +2 +2, +2 +1, +2 0, +2 -1, +2 -2, +2 -5, +2 -4, +2 -2, +2

+3, +3 +4, +3 +5, +3 +2, +3 +1, +3 0, +3 -1, +3 -2, +3 -5, +3 -4, +3 -2, +3

i=n
Thus the initial set of possible solutions has a size of ∏ (si − 2)
i=3
However, within the remaining solutions, there exists yet another subset that does not permit a
nucleus of length 6 to occur. These would be ALL solutions not containing a +1 in them, since there must
exist at least prime, either 7 or 11, that divides μ − 1 , such that μ ≡ 1 mod 7 or 1 mod 11.

The remaining solutions not containing a +1 shall be highlighted in blue. The viable solutions will
highlighted in light red.
+3, -3 +4, -3 +5, -3 +2, -3 +1, -3 0, -3 -1, -3 -2, -3 -5, -3 -4, -3 -3, -3

+3, -2 +4, -2 +5, -2 +2, -2 +1, -2 0, -2 -1, -2 -2, -2 -5, -2 -4, -2 -3, -2

+3, -1 +4, -1 +5, -1 +2, -1 +1, -1 0, -1 -1, -1 -2, -1 -5, -1 -4, -1 -3, -1

+3, 0 +4, 0 +5, 0 +2, 0 +1, 0 0, 0 -1, 0 -2, 0 -5, 0 -4, 0 -2, 0

+3, +1 +4, +1 +5, +1 +2, +1 +1, +1 0, +1 -1, +1 -2, +1 -5, +1 -4, +1 -2, +1

+3, +2 +4, +2 +5, +2 +2, +2 +1, +2 0, +2 -1, +2 -2, +2 -5, +2 -4, +2 -2, +2

+3, +3 +4, +3 +5, +3 +2, +3 +1, +3 0, +3 -1, +3 -2, +3 -5, +3 -4, +3 -2, +3

Now we shall rearrange the above matrix one more time:

+3, -3 +1, -3 +5, -3 +2, -3 +4, -3 0, -3 -1, -3 -2, -3 -5, -3 -4, -3 -3, -3

+3, -2 +1, -2 +5, -2 +2, -2 +4, -2 0, -2 -1, -2 -2, -2 -5, -2 -4, -2 -3, -2

+3, -1 +1, -1 +5, -1 +2, -1 +4, -1 0, -1 -1, -1 -2, -1 -5, -1 -4, -1 -3, -1

+3, 0 +1, 0 +5, 0 +2, 0 +4, 0 0, 0 -1, 0 -2, 0 -5, 0 -4, 0 -2, 0

+3, +2 +1, +2 +5, +2 +2, +2 +4, +2 0, +2 -1, +2 -2, +2 -5, +2 -4, +2 -2, +2

+3, +1 +1, +1 +5, +1 +2, +1 +4, +1 0, +1 -1, +1 -2, +1 -5, +1 -4, +1 -2, +1

+3, +3 +1, +3 +5, +3 +2, +3 +4, +3 0, +3 -1, +3 -2, +3 -5, +3 -4, +3 -2, +3

The set of invalid solutions has a size of 4*8 Thus the initial set of possible solutions has a size of
i=n
∏ (si − 3) , as this would apply to any primes numbers in place of 7 and 11 in S.
i=3
QED

/
We shall now extend the size of S, and see if applies for rank-3 tensors.

Lemma 1.9.1.4; The size of M6 , S = {2, 3, 5, 7, 11} and |S| = n , {2,3} ∈ S

Let S = {2, 3, 7, 11}

∀μ ∈ M6, μ ≡ 0 mod 2 ≡− 1mod 3 ≡ a1 mod 7 ≡ a2 mod 11; a1 , a2 =/ ± 3

Either 5 or 7 or 11 must be belligerent, such that it sieves (μ − 1).

We will immediately start with the previous matrix, and then stack this matrix 5 times. If μ ≡ ± 3 mod 5 ,
then the entire slate is discarded, and therefore highlighted in yellow.

k ≡ − 2 mod 5 = 3 mod 5 , all solutions are discarded since 5 divides the endpoint of the nucleus.
+3, -3 +1, -3 +5, -3 +2, -3 +4, -3 0, -3 -1, -3 -2, -3 -5, -3 -4, -3 -3, -3

+3, -2 +1, -2 +5, -2 +2, -2 +4, -2 0, -2 -1, -2 -2, -2 -5, -2 -4, -2 -3, -2

+3, -1 +1, -1 +5, -1 +2, -1 +4, -1 0, -1 -1, -1 -2, -1 -5, -1 -4, -1 -3, -1

+3, 0 +1, 0 +5, 0 +2, 0 +4, 0 0, 0 -1, 0 -2, 0 -5, 0 -4, 0 -2, 0

+3, +2 +1, +2 +5, +2 +2, +2 +4, +2 0, +2 -1, +2 -2, +2 -5, +2 -4, +2 -2, +2

+3, +1 +1, +1 +5, +1 +2, +1 +4, +1 0, +1 -1, +1 -2, +1 -5, +1 -4, +1 -2, +1

+3, +3 +1, +3 +5, +3 +2, +3 +4, +3 0, +3 -1, +3 -2, +3 -5, +3 -4, +3 -2, +3

k ≡ − 1 mod 5 , since all of these solutions are congruent to -1 mod 5, only the light red entries produce a
nucleus of length 6.
+3, -3 +1, -3 +5, -3 +2, -3 +4, -3 0, -3 -1, -3 -2, -3 -5, -3 -4, -3 -3, -3

+3, -2 +1, -2 +5, -2 +2, -2 +4, -2 0, -2 -1, -2 -2, -2 -5, -2 -4, -2 -3, -2

+3, -1 +1, -1 +5, -1 +2, -1 +4, -1 0, -1 -1, -1 -2, -1 -5, -1 -4, -1 -3, -1

+3, 0 +1, 0 +5, 0 +2, 0 +4, 0 0, 0 -1, 0 -2, 0 -5, 0 -4, 0 -2, 0

+3, +2 +1, +2 +5, +2 +2, +2 +4, +2 0, +2 -1, +2 -2, +2 -5, +2 -4, +2 -2, +2

+3, +1 +1, +1 +5, +1 +2, +1 +4, +1 0, +1 -1, +1 -2, +1 -5, +1 -4, +1 -2, +1

+3, +3 +1, +3 +5, +3 +2, +3 +4, +3 0, +3 -1, +3 -2, +3 -5, +3 -4, +3 -2, +3

/
k ≡ 0 mod 5 , since all of these solutions are congruent to 0 mod 5, only the light red entries produce a
nucleus of length 6.
+3, -3 +1, -3 +5, -3 +2, -3 +4, -3 0, -3 -1, -3 -2, -3 -5, -3 -4, -3 -3, -3

+3, -2 +1, -2 +5, -2 +2, -2 +4, -2 0, -2 -1, -2 -2, -2 -5, -2 -4, -2 -3, -2

+3, -1 +1, -1 +5, -1 +2, -1 +4, -1 0, -1 -1, -1 -2, -1 -5, -1 -4, -1 -3, -1

+3, 0 +1, 0 +5, 0 +2, 0 +4, 0 0, 0 -1, 0 -2, 0 -5, 0 -4, 0 -2, 0

+3, +2 +1, +2 +5, +2 +2, +2 +4, +2 0, +2 -1, +2 -2, +2 -5, +2 -4, +2 -2, +2

+3, +1 +1, +1 +5, +1 +2, +1 +4, +1 0, +1 -1, +1 -2, +1 -5, +1 -4, +1 -2, +1

+3, +3 +1, +3 +5, +3 +2, +3 +4, +3 0, +3 -1, +3 -2, +3 -5, +3 -4, +3 -2, +3

k ≡ 1 mod 5 , since all of these solutions are congruent to 1 mod 5, all of the discarded solutions in blue
are valid in this slate, since 5 divides (μ − 1) in each of the systems of congruences represented.
+3, -3 +1, -3 +5, -3 +2, -3 +4, -3 0, -3 -1, -3 -2, -3 -5, -3 -4, -3 -3, -3

+3, -2 +1, -2 +5, -2 +2, -2 +4, -2 0, -2 -1, -2 -2, -2 -5, -2 -4, -2 -3, -2

+3, -1 +1, -1 +5, -1 +2, -1 +4, -1 0, -1 -1, -1 -2, -1 -5, -1 -4, -1 -3, -1

+3, 0 +1, 0 +5, 0 +2, 0 +4, 0 0, 0 -1, 0 -2, 0 -5, 0 -4, 0 -2, 0

+3, +2 +1, +2 +5, +2 +2, +2 +4, +2 0, +2 -1, +2 -2, +2 -5, +2 -4, +2 -2, +2

+3, +1 +1, +1 +5, +1 +2, +1 +4, +1 0, +1 -1, +1 -2, +1 -5, +1 -4, +1 -2, +1

+3, +3 +1, +3 +5, +3 +2, +3 +4, +3 0, +3 -1, +3 -2, +3 -5, +3 -4, +3 -2, +3

k ≡ 2 mod 5 = − 3 mod 5 , all solutions are discarded since 5 divides the endpoint of the nucleus.
+3, -3 +1, -3 +5, -3 +2, -3 +4, -3 0, -3 -1, -3 -2, -3 -5, -3 -4, -3 -3, -3

+3, -2 +1, -2 +5, -2 +2, -2 +4, -2 0, -2 -1, -2 -2, -2 -5, -2 -4, -2 -3, -2

+3, -1 +1, -1 +5, -1 +2, -1 +4, -1 0, -1 -1, -1 -2, -1 -5, -1 -4, -1 -3, -1

+3, 0 +1, 0 +5, 0 +2, 0 +4, 0 0, 0 -1, 0 -2, 0 -5, 0 -4, 0 -2, 0

+3, +2 +1, +2 +5, +2 +2, +2 +4, +2 0, +2 -1, +2 -2, +2 -5, +2 -4, +2 -2, +2

+3, +1 +1, +1 +5, +1 +2, +1 +4, +1 0, +1 -1, +1 -2, +1 -5, +1 -4, +1 -2, +1

+3, +3 +1, +3 +5, +3 +2, +3 +4, +3 0, +3 -1, +3 -2, +3 -5, +3 -4, +3 -2, +3

If were to stack these slates, the discarded blue solutions would form a rank-3 tensor of
dimensions 2*4*8 = (5-3)(7-3)(11-3) = 64, while the discarded yellow solutions form a solid shell with a
depth of one unit encapsulating the blue tensor and the remaining valid solutions. This would apply to
any set of primes of larger size as the same geometrical manipulations could be performed on

(∏ )
i=n i=n
rank- (n − 2) tensors, thus the total count of viable solutions equals (si − 2) − ∏(si − 3)
i=3 i=3

/
The shape of the valid solutions in light pink takes on the form of 3 orthogonal walls joining at
single corner (+1,+1, +1). Face XZ = 3*9; Face XY = 5*9; Face YZ = 3*5

( )
i=n i=n
However, the Lemma states that ϖ (M6) = 2 ∏ (si − 2) − ∏(si − 3) , but the work we’ve done
i=3 i=3

thus far has only yielded half that value. Where does this factor of two come from? Let us look again at a
nucleus of length 6:

k ≡ 0 mod 2 ≡ − 1 mod 3 ≡ 2 mod 6


k μ−3 μ−1 μ μ+3

ti ui ti+1

ξ (k) ∅ 2 bx 2 3 2 ∅

For each of the valid systems of congruences highlighted in pink, there exists an additive inverse that also
yields a nucleus of length 6.

k ≡ 0 mod 2 ≡ + 1 mod 3 ≡ 4 mod 6


k μ−3 μ−1 μ μ+3

ti ui ti+1

ξ (k) ∅ 2 3 2 bx 2 ∅

As such there exists a bijection between the valid solutions congruent to 2 mod 6 and 4 mod 6. Thus the

(∏ )
i=n i=n
total count of nuclei of length 6 is precisely equal ϖ (M6) = 2 (si − 2) − ∏(si − 3) .
i=3 i=3

QED

/
Lemma 1.9.1.4; The size of M8 , S = {2, 3, 11, 13} and |S| = 4 , {2,3} ∈ S

Let S = {2, 3,11, 13}

5 and 7 cannot be in S due to counting irregularities; sieving sets including 5 and/or 7 require
specialized formulas that must be derived either geometrically or through the Set Theory Interpretation
which will be introduced fairly soon, due to the fact that we will soon be forced into 4 or more spatial
dimensions to yield counting formulas for larger nuclei, which is beyond human perception.

The Set Theory Interpretation will provide us with a concrete method of obtaining all the
specialized formulas, and will also illuminate why I have thus far reversed the negative and positive
directions in the matrices. In this lemma we will see the initial workings of the Set Theory Interpretation.

∀μ ∈ M8, μ ≡ − 1 mod 2 ≡ 0 mod 3 ≡ a1 mod 11 ≡ a2 mod 13; a1 , a2 =/ ± 4

Both 11 or 13 must be belligerent, such that they sieve (μ − 2) and (μ + 2) , {11, 13} ∈ B.

k μ−4 μ−2 μ μ+2 μ+4

ti ui ti+1

ξ (k) ∅ 2 bx 2 3 2 by 2 ∅

i=n i=n i=n


ϖ (M8) = ∏ (si − 2) − 2 ∏ (si − 3) + ∏ (si − 4) , n ≥ 4
i=3 i=3 i=3

Proof:

Let us compile all possible systems of congruences in respect to 11 and 13 into a matrix whose
dimensions are 11x13 = 143.

/
k ≡ a mod 11 ≡ b mod 13 ; − 5 ≤ a ≤+ 5, − 6 ≤ b ≤+ 6, a, b ∈ Z .

k ≡ 3 mod 6
+5, -6 +4, -6 +3, -6 +2, -6 +1, -6 0, -6 -1, -6 -2, -6 -3, -6 -4, -6 -5, -6

+5, -5 +4, -5 +3, -5 +2, -5 +1, -5 0, -5 -1, -5 -2, -5 -3, -5 -4, -5 -5, -5

+5, -4 +4, -4 +3, -4 +2, -4 +1, -4 0, -4 -1, -4 -2, -4 -3, -4 -4, -4 -5, -4

+5, -3 +4, -3 +3, -3 +2, -3 +1, -3 0, -3 -1, -3 -2, -3 -3, -3 -4, -3 -5, -3

+5, -2 +4, -2 +3, -2 +2, -2 +1, -2 0, -2 -1, -2 -2, -2 -3, -2 -4, -2 -5, -2

+5, -1 +4, -1 +3, -1 +2, -1 +1, -1 0, -1 -1, -1 -2, -1 -3, -1 -4, -1 -5, -1

+5, 0 +4, 0 +3, 0 +2, 0 +1, 0 0, 0 -1, 0 -2, 0 -2, 0 -4, 0 -5, 0

+5, +1 +4, +1 +3, +1 +2, +1 +1, +1 0, +1 -1, +1 -2, +1 -2, +1 -4, +1 -5, +1

+5, +2 +4, +2 +3, +2 +2, +2 +1, +2 0, +2 -1, +2 -2, +2 -2, +2 -4, +2 -5, +2

+5, +3 +4, +3 +3, +3 +2, +3 +1, +3 0, +3 -1, +3 -2, +3 -2, +3 -4, +3 -5, +3

+5, +4 +4, +4 +3, +4 +2, +4 +1, +4 0, +4 -1, +4 -2, +4 -3, +4 -4, +4 -5, +4

+5, +5 +4, +5 +3, +5 +2, +5 +1, +5 0, +5 -1, +5 -2, +5 -3, +5 -4, +5 -5, +5

+5, +6 +4, +6 +3, +6 +2, +6 +1, +6 0, +6 -1, +6 -2, +6 -3, +6 -4, +6 -5, +6

All of the above 143 entries containing a positive or negative 4 represent systems of congruences
which shall not yield a nucleus of length 4, they are highlighted in yellow. Bear in mind that +3 = -4 mod 7,
and -3 = 4 mod 7, and 1 = -4 mod 5 and -1 = +4 mod 5, which is why they are excluded from the general
counting formulas.

No matter how long the length of a nucleus may be, no prime in S is allowed to divide its
endpoints (later on the endpoints will be formally defined as terminals), thus the first term in the General
−=n
Nucleic Counting Formulas will always start with ∏ (si − 2) , since these solutions can always be arranged
i=3
to form the exterior shell of a rank- (n − 2) tensor.
.
Also note that nuclei of length eight require a minimum of two belligerent primes.
The set S = {2,3, p } can never sieve 8 consecutive integers. This observation is the primary focus of
solving the Andrica Conjecture, since we must determine both the precise minimum quantity and the
special quality of the primes required to sieve a sequence of integers. It is of my opinion that
mathematicians have so far failed to solve the Andrica Conjecture because they never considered the
quality of the primes , and rather focused solely on the quantity and therefore always defaulted to
analytical arguments instead of elementary and geometrical ones.

For now let us continue with this two-dimension example of {11, 13} and rearrange the perimeter.

/
k ≡ 3 mod 6
+4, -6 +5, -4 +3, -4 +2, -4 +1, -4 0, -4 -1, -4 -2, -4 -3, -4 -5, -4 -4, -6

+4, -5 +5, -6 +3, -6 +2, -6 +1, -6 0, -6 -1, -6 -2, -6 -3, -6 -5, -6 -4, -5

+4, -4 +5, -5 +3, -5 +2, -5 +1, -5 0, -5 -1, -5 -2, -5 -3, -5 -5, -5 -4, -4

+4, -3 +5, -3 +3, -3 +2, -3 +1, -3 0, -3 -1, -3 -2, -3 -3, -3 -5, -3 -4, -3

+4, -2 +5, -2 +3, -2 +2, -2 +1, -2 0, -2 -1, -2 -2, -2 -3, -2 -5, -2 -4, -2

+4, -1 +5, -1 +3, -1 +2, -1 +1, -1 0, -1 -1, -1 -2, -1 -3, -1 -5, -1 -4, -1

+4, 0 +5, 0 +3, 0 +2, 0 +1, 0 0, 0 -1, 0 -2, 0 -2, 0 -5, 0 -4, 0

+4, +1 +5, +1 +3, +1 +2, +1 +1, +1 0, +1 -1, +1 -2, +1 -2, +1 -5, +1 -4, +1

+4, +2 +5, +2 +3, +2 +2, +2 +1, +2 0, +2 -1, +2 -2, +2 -2, +2 -5, +2 -4, +2

+4, +3 +5, +3 +3, +3 +2, +3 +1, +3 0, +3 -1, +3 -2, +3 -2, +3 -5, +3 -4, +3

+4, +4 +5, +5 +3, +5 +2, +5 +1, +5 0, +5 -1, +5 -2, +5 -3, +5 -5, +5 -4, +4

+4, +5 +5, +6 +3, +6 +2, +6 +1, +6 0, +6 -1, +6 -2, +6 -3, +6 -5, +6 -4, +5

+4, +6 +45 +4 +3, +4 +2, +4 +1, +4 0, +4 -1, +4 -2, +4 -3, +4 -5, +4 -4, +6

i=n
Thus the initial set of possible solutions has a size of ∏ (si − 2) . Call this Set A.
i=3
However, within the remaining solutions, there exists yet another subset that does not permit a
nucleus of length 8 to occur. These would be ALL solutions not containing a +2 in them, since there must
exist at least prime, either 11 or 13, that divides μ − 2 , such that μ ≡ + 2 mod 11 or + 2 mod 13

The remaining solutions not containing a +2 shall be highlighted in blue. The viable solutions will
highlighted in light red. k ≡ 3 mod 6
+4, -6 +5, -4 +3, -4 +2, -4 +1, -4 0, -4 -1, -4 -2, -4 -3, -4 -5, -4 -4, -6

+4, -5 +5, -6 +3, -6 +2, -6 +1, -6 0, -6 -1, -6 -2, -6 -3, -6 -5, -6 -4, -5

+4, -4 +5, -5 +3, -5 +2, -5 +1, -5 0, -5 -1, -5 -2, -5 -3, -5 -5, -5 -4, -4

+4, -3 +5, -3 +3, -3 +2, -3 +1, -3 0, -3 -1, -3 -2, -3 -3, -3 -5, -3 -4, -3

+4, -2 +5, -2 +3, -2 +2, -2 +1, -2 0, -2 -1, -2 -2, -2 -3, -2 -5, -2 -4, -2

+4, -1 +5, -1 +3, -1 +2, -1 +1, -1 0, -1 -1, -1 -2, -1 -3, -1 -5, -1 -4, -1

+4, 0 +5, 0 +3, 0 +2, 0 +1, 0 0, 0 -1, 0 -2, 0 -2, 0 -5, 0 -4, 0

+4, +1 +5, +1 +3, +1 +2, +1 +1, +1 0, +1 -1, +1 -2, +1 -2, +1 -5, +1 -4, +1

+4, +2 +5, +2 +3, +2 +2, +2 +1, +2 0, +2 -1, +2 -2, +2 -2, +2 -5, +2 -4, +2

+4, +3 +5, +3 +3, +3 +2, +3 +1, +3 0, +3 -1, +3 -2, +3 -2, +3 -5, +3 -4, +3

+4, +4 +5, +5 +3, +5 +2, +5 +1, +5 0, +5 -1, +5 -2, +5 -3, +5 -5, +5 -4, +4

+4, +5 +5, +6 +3, +6 +2, +6 +1, +6 0, +6 -1, +6 -2, +6 -3, +6 -5, +6 -4, +5

+4, +6 +45 +4 +3, +4 +2, +4 +1, +4 0, +4 -1, +4 -2, +4 -3, +4 -5, +4 -4, +6

/
Now we shall rearrange the above matrix one more time:

+4, -6 +5, -4 +3, -4 +2, -4 +1, -4 0, -4 -1, -4 -2, -4 -3, -4 -5, -4 -4, -6

+4, -5 +2, +3 +1, +3 0, +3 -1, +3 -2, +3 -2, +3 -5, +3 +5, +3 +3, +3 -4, -5

+4, -4 +2, +5 +1, +5 0, +5 -1, +5 -2, +5 -3, +5 -5, +5 +5, +5 +3, +5 -4, -4

+4, -3 +2, +6 +1, +6 0, +6 -1, +6 -2, +6 -3, +6 -5, +6 +5, +6 +3, +6 -4, -3

+4, -2 +2, -6 +1, -6 0, -6 -1, -6 -2, -6 -3, -6 -5, -6 +5, -6 +3, -6 -4, -2

+4, -1 +2, -5 +1, -5 0, -5 -1, -5 -2, -5 -3, -5 -5, -5 +5, -5 +3, -5 -4, -1

+4, 0 +2, -3 +1, -3 0, -3 -1, -3 -2, -3 -3, -3 -5, -3 +5, -3 +3, -3 -4, 0

+4, +1 +2, -2 +1, -2 0, -2 -1, -2 -2, -2 -3, -2 -5, -2 +5, -2 +3, -2 -4, +1

+4, +2 +2, -1 +1, -1 0, -1 -1, -1 -2, -1 -3, -1 -5, -1 +5, -1 +3, -1 -4, +2

+4, +3 +2, 0 +1, 0 0, 0 -1, 0 -2, 0 -2, 0 -5, 0 +5, 0 +3, 0 -4, +3

+4, +4 +2, +1 +1, +1 0, +1 -1, +1 -2, +1 -2, +1 -5, +1 +5, +1 +3, +1 -4, +4

+4, +5 +2, +2 +1, +2 0, +2 -1, +2 -2, +2 -2, +2 -5, +2 +5, +2 +3, +2 -4, +5

+4, +6 +45 +4 +3, +4 +2, +4 +1, +4 0, +4 -1, +4 -2, +4 -3, +4 -5, +4 -4, +6

The set of invalid solutions has a size of 8*10. Thus the initial set of possible solutions has a size
i=n
of ∏ (si − 3) , as this would apply to any primes numbers in place of 11 and 13 in S (aside from 5 and 7).
i=3

Call this set of discard solutions (in blue) Set C.

/
However, there exists STILL yet another subset of EQUAL size that does not permit a nucleus of
length 8 to occur. These would be ALL solutions not containing a -2 in them, since there must exist at
least prime, either 11 or 13, that divides μ + 2 , such that μ ≡ − 2 mod 11 or − 2 mod 13

The remaining solutions not containing a -2 shall be highlighted in blue. The viable solutions will
highlighted in light red.

k ≡ 3 mod 6
+4, -6 +5, -4 +3, -4 +2, -4 +1, -4 0, -4 -1, -4 -2, -4 -3, -4 -5, -4 -4, -6

+4, -5 +5, -6 +3, -6 +2, -6 +1, -6 0, -6 -1, -6 -2, -6 -3, -6 -5, -6 -4, -5

+4, -4 +5, -5 +3, -5 +2, -5 +1, -5 0, -5 -1, -5 -2, -5 -3, -5 -5, -5 -4, -4

+4, -3 +5, -3 +3, -3 +2, -3 +1, -3 0, -3 -1, -3 -2, -3 -3, -3 -5, -3 -4, -3

+4, -2 +5, -2 +3, -2 +2, -2 +1, -2 0, -2 -1, -2 -2, -2 -3, -2 -5, -2 -4, -2

+4, -1 +5, -1 +3, -1 +2, -1 +1, -1 0, -1 -1, -1 -2, -1 -3, -1 -5, -1 -4, -1

+4, 0 +5, 0 +3, 0 +2, 0 +1, 0 0, 0 -1, 0 -2, 0 -2, 0 -5, 0 -4, 0

+4, +1 +5, +1 +3, +1 +2, +1 +1, +1 0, +1 -1, +1 -2, +1 -2, +1 -5, +1 -4, +1

+4, +2 +5, +2 +3, +2 +2, +2 +1, +2 0, +2 -1, +2 -2, +2 -2, +2 -5, +2 -4, +2

+4, +3 +5, +3 +3, +3 +2, +3 +1, +3 0, +3 -1, +3 -2, +3 -2, +3 -5, +3 -4, +3

+4, +4 +5, +5 +3, +5 +2, +5 +1, +5 0, +5 -1, +5 -2, +5 -3, +5 -5, +5 -4, +4

+4, +5 +5, +6 +3, +6 +2, +6 +1, +6 0, +6 -1, +6 -2, +6 -3, +6 -5, +6 -4, +5

+4, +6 +45 +4 +3, +4 +2, +4 +1, +4 0, +4 -1, +4 -2, +4 -3, +4 -5, +4 -4, +6

Rearranged, call this set of discard solutions (in blue) Set E.


+4, -6 +5, -4 +3, -4 +2, -4 +1, -4 0, -4 -1, -4 -2, -4 -3, -4 -5, -4 -4, -6

+4, -5 -3, -2 -5, -2 +5, -2 +3, -2 +2, -2 +1, -2 0, -2 -1, -2 -2, -2 -4, -5

+4, -4 -3, -1 -5, -1 +5, -1 +3, -1 +2, -1 +1, -1 0, -1 -1, -1 -2, -1 -4, -4

+4, -3 -2, 0 -5, 0 +5, 0 +3, 0 +2, 0 +1, 0 0, 0 -1, 0 -2, 0 -4, -3

+4, -2 -2, +1 -5, +1 +5, +1 +3, +1 +2, +1 +1, +1 0, +1 -1, +1 -2, +1 -4, -2

+4, -1 -2, +2 -5, +2 +5, +2 +3, +2 +2, +2 +1, +2 0, +2 -1, +2 -2, +2 -4, -1

+4, 0 -2, +3 -5, +3 +5, +3 +3, +3 +2, +3 +1, +3 0, +3 -1, +3 -2, +3 -4, 0

+4, +1 -3, +5 -5, +5 +5, +5 +3, +5 +2, +5 +1, +5 0, +5 -1, +5 -2, +5 -4, +1

+4, +2 -3, +6 -5, +6 +5, +6 +3, +6 +2, +6 +1, +6 0, +6 -1, +6 -2, +6 -4, +2

+4, +3 -3, -6 -5, -6 +5, -6 +3, -6 +2, -6 +1, -6 0, -6 -1, -6 -2, -6 -4, +3

+4, +4 -3, -5 -5, -5 +5, -5 +3, -5 +2, -5 +1, -5 0, -5 -1, -5 -2, -5 -4, +4

+4, +5 -3, -3 -5, -3 +5, -3 +3, -3 +2, -3 +1, -3 0, -3 -1, -3 -2, -3 -4, +5

+4, +6 +45 +4 +3, +4 +2, +4 +1, +4 0, +4 -1, +4 -2, +4 -3, +4 -5, +4 -4, +6

/
Let us combine the matrices of the discarded solutions for both +2 and -2 (notice that I never
rearrange the yellow entries, since they act as indices for the matrices). The solutions that were discarded
by both of the previous matrices are highlighted in purple, and the solutions discarded only by one of the
matrices are highlighted in green. We result with only TWO viable solutions.

Recall that set A is the initial set of all possible solutions, Set C is the set of discarded solutions
from the first matrix, and Set E is the set of discarded solutions from the third matrix.

Let L be the solutions in purple; L represents the intersection of C and E.


L = C ⋂ E.

Let G be the solutions in green; G represents the symmetric difference of C an E.


G = C △ E.

+4, -6 +5, -4 +3, -4 +2, -4 +1, -4 0, -4 -1, -4 -2, -4 -3, -4 -5, -4 -4, -6

+4, -5 +2, -2 +1, -2 0, -2 -1, -2 +3, -2 -3, -2 -5, -2 +5, -2 -2, -2 -4, -5

+4, -4 +2, +5 +1, +5 0, +5 -1, +5 +3, +5 -3, +5 -5, +5 +5, +5 -2, +5 -4, -4

+4, -3 +2, +6 +1, +6 0, +6 -1, +6 +3, +6 -3, +6 -5, +6 +5, +6 -2, +6 -4, -3

+4, -2 +2, -6 +1, -6 0, -6 -1, -6 +3, -6 -3, -6 -5, -6 +5, -6 -2, -6 -4, -2

+4, -1 +2, -5 +1, -5 0, -5 -1, -5 +3, -5 -3, -5 -5, -5 +5, -5 -2, -5 -4, -1

+4, 0 +2, -3 +1, -3 0, -3 -1, -3 +3, -3 -3, -3 -5, -3 +5, -3 -2, -3 -4, 0

+4, +1 +2, +3 +1, +3 0, +3 -1, +3 +3, +3 -3, +3 -5, +3 +5, +3 -2, +3 -4, +1

+4, +2 +2, -1 +1, -1 0, -1 -1, -1 +3, -1 -3, -1 -5, -1 +5, -1 -2, -1 -4, +2

+4, +3 +2, 0 +1, 0 0, 0 -1, 0 +3, 0 -3, 0 -5, 0 +5, 0 -2, 0 -4, +3

+4, +4 +2, +1 +1, +1 0, +1 -1, +1 +3, +1 -3, +1 -5, +1 +5, +1 -2, +1 -4, +4

+4, +5 +2, +2 +1, +2 0, +2 -1, +2 +3, +2 -2, +2 -5, +2 +5, +2 -2, +2 -4, +5

+4, +6 +45 +4 +3, +4 +2, +4 +1, +4 0, +4 -1, +4 -2, +4 -3, +4 -5, +4 -4, +6

We must now find a way to numerically subtract all of green and purple solutions. The answer is
simple: Subtract both C and E, and then add back G, since G was subtracted twice, as G is a subset of
both C and E.

We know that:
i=4
|A| = 9*11 = (11-2)(13-2) = ∏(si − 2)
i=3
i=4
|C| and |E| = 8*10 = (11-3)(13-3) = ∏(si − 3)
i=3
i=4
|G| = 7*9 = (11-4)(13 - 4) = ∏(si − 4)
i=3

i=n i=n i=n


Thus |A| - |C| - |E| + |G| = ∏ (si − 2) − 2 ∏ (si − 3) + ∏ (si − 4)
i=3 i=3 i=3

/
Now let us generalize this for larger sets of primes, and therefore for rank-( n − 2 ) tensors. We will
use 11*13 as the slate of a rank-3 tensor, and add a fifth prime to S. S = {2,3,11,13, 127}.

For μ ≡ ± 4 mod 127 , we would get two completely eliminated slates. We would place these two slates at
the top and bottom faces of the rank-3 tensor (rectangular prism of height 127).

+4, -6 +5, -4 +3, -4 +2, -4 +1, -4 0, -4 -1, -4 -2, -4 -3, -4 -5, -4 -4, -6

+4, -5 +2, -2 +1, -2 0, -2 -1, -2 +3, -2 -3, -2 -5, -2 +5, -2 -2, -2 -4, -5

+4, -4 +2, +5 +1, +5 0, +5 -1, +5 +3, +5 -3, +5 -5, +5 +5, +5 -2, +5 -4, -4

+4, -3 +2, +6 +1, +6 0, +6 -1, +6 +3, +6 -3, +6 -5, +6 +5, +6 -2, +6 -4, -3

+4, -2 +2, -6 +1, -6 0, -6 -1, -6 +3, -6 -3, -6 -5, -6 +5, -6 -2, -6 -4, -2

+4, -1 +2, -5 +1, -5 0, -5 -1, -5 +3, -5 -3, -5 -5, -5 +5, -5 -2, -5 -4, -1

+4, 0 +2, -3 +1, -3 0, -3 -1, -3 +3, -3 -3, -3 -5, -3 +5, -3 -2, -3 -4, 0

+4, +1 +2, +3 +1, +3 0, +3 -1, +3 +3, +3 -3, +3 -5, +3 +5, +3 -2, +3 -4, +1

+4, +2 +2, -1 +1, -1 0, -1 -1, -1 +3, -1 -3, -1 -5, -1 +5, -1 -2, -1 -4, +2

+4, +3 +2, 0 +1, 0 0, 0 -1, 0 +3, 0 -3, 0 -5, 0 +5, 0 -2, 0 -4, +3

+4, +4 +2, +1 +1, +1 0, +1 -1, +1 +3, +1 -3, +1 -5, +1 +5, +1 -2, +1 -4, +4

+4, +5 +2, +2 +1, +2 0, +2 -1, +2 +3, +2 -2, +2 -5, +2 +5, +2 -2, +2 -4, +5

+4, +6 +45 +4 +3, +4 +2, +4 +1, +4 0, +4 -1, +4 -2, +4 -3, +4 -5, +4 -4, +6

For μ ≡ x mod 127, x =/ + 4, + 2, − 2, − 4 , we would result with 123 consecutive slates identical to the
2-D solutions.

+4, -6 +5, -4 +3, -4 +2, -4 +1, -4 0, -4 -1, -4 -2, -4 -3, -4 -5, -4 -4, -6

+4, -5 +2, -2 +1, -2 0, -2 -1, -2 +3, -2 -3, -2 -5, -2 +5, -2 -2, -2 -4, -5

+4, -4 +2, +5 +1, +5 0, +5 -1, +5 +3, +5 -3, +5 -5, +5 +5, +5 -2, +5 -4, -4

+4, -3 +2, +6 +1, +6 0, +6 -1, +6 +3, +6 -3, +6 -5, +6 +5, +6 -2, +6 -4, -3

+4, -2 +2, -6 +1, -6 0, -6 -1, -6 +3, -6 -3, -6 -5, -6 +5, -6 -2, -6 -4, -2

+4, -1 +2, -5 +1, -5 0, -5 -1, -5 +3, -5 -3, -5 -5, -5 +5, -5 -2, -5 -4, -1

+4, 0 +2, -3 +1, -3 0, -3 -1, -3 +3, -3 -3, -3 -5, -3 +5, -3 -2, -3 -4, 0

+4, +1 +2, +3 +1, +3 0, +3 -1, +3 +3, +3 -3, +3 -5, +3 +5, +3 -2, +3 -4, +1

+4, +2 +2, -1 +1, -1 0, -1 -1, -1 +3, -1 -3, -1 -5, -1 +5, -1 -2, -1 -4, +2

+4, +3 +2, 0 +1, 0 0, 0 -1, 0 +3, 0 -3, 0 -5, 0 +5, 0 -2, 0 -4, +3

+4, +4 +2, +1 +1, +1 0, +1 -1, +1 +3, +1 -3, +1 -5, +1 +5, +1 -2, +1 -4, +4

+4, +5 +2, +2 +1, +2 0, +2 -1, +2 +3, +2 -2, +2 -5, +2 +5, +2 -2, +2 -4, +5

+4, +6 +45 +4 +3, +4 +2, +4 +1, +4 0, +4 -1, +4 -2, +4 -3, +4 -5, +4 -4, +6

/
For μ ≡ + 2 mod 127 , we would result with a single slate that enables all discarded solutions containing a
-2, since the nucleus would contain a belligerent prime at both of the required locations. These
reactivated solutions are highlighted in blue. We would place this above the bottom slate.

+4, -6 +5, -4 +3, -4 +2, -4 +1, -4 0, -4 -1, -4 -2, -4 -3, -4 -5, -4 -4, -6

+4, -5 +2, -2 +1, -2 0, -2 -1, -2 +3, -2 -3, -2 -5, -2 +5, -2 -2, -2 -4, -5

+4, -4 +2, +5 +1, +5 0, +5 -1, +5 +3, +5 -3, +5 -5, +5 +5, +5 -2, +5 -4, -4

+4, -3 +2, +6 +1, +6 0, +6 -1, +6 +3, +6 -3, +6 -5, +6 +5, +6 -2, +6 -4, -3

+4, -2 +2, -6 +1, -6 0, -6 -1, -6 +3, -6 -3, -6 -5, -6 +5, -6 -2, -6 -4, -2

+4, -1 +2, -5 +1, -5 0, -5 -1, -5 +3, -5 -3, -5 -5, -5 +5, -5 -2, -5 -4, -1

+4, 0 +2, -3 +1, -3 0, -3 -1, -3 +3, -3 -3, -3 -5, -3 +5, -3 -2, -3 -4, 0

+4, +1 +2, +3 +1, +3 0, +3 -1, +3 +3, +3 -3, +3 -5, +3 +5, +3 -2, +3 -4, +1

+4, +2 +2, -1 +1, -1 0, -1 -1, -1 +3, -1 -3, -1 -5, -1 +5, -1 -2, -1 -4, +2

+4, +3 +2, 0 +1, 0 0, 0 -1, 0 +3, 0 -3, 0 -5, 0 +5, 0 -2, 0 -4, +3

+4, +4 +2, +1 +1, +1 0, +1 -1, +1 +3, +1 -3, +1 -5, +1 +5, +1 -2, +1 -4, +4

+4, +5 +2, +2 +1, +2 0, +2 -1, +2 +3, +2 -2, +2 -5, +2 +5, +2 -2, +2 -4, +5

+4, +6 +45 +4 +3, +4 +2, +4 +1, +4 0, +4 -1, +4 -2, +4 -3, +4 -5, +4 -4, +6

For μ ≡ − 2 mod 127 , we would result with a single slate that enables all discarded solutions containing a
+2, since the nucleus would contain a belligerent prime at both of the required locations. These
reactivated solutions are highlighted in blue. We would place this below the top slate.

+4, -6 +5, -4 +3, -4 +2, -4 +1, -4 0, -4 -1, -4 -2, -4 -3, -4 -5, -4 -4, -6

+4, -5 +2, -2 +1, -2 0, -2 -1, -2 +3, -2 -3, -2 -5, -2 +5, -2 -2, -2 -4, -5

+4, -4 +2, +5 +1, +5 0, +5 -1, +5 +3, +5 -3, +5 -5, +5 +5, +5 -2, +5 -4, -4

+4, -3 +2, +6 +1, +6 0, +6 -1, +6 +3, +6 -3, +6 -5, +6 +5, +6 -2, +6 -4, -3

+4, -2 +2, -6 +1, -6 0, -6 -1, -6 +3, -6 -3, -6 -5, -6 +5, -6 -2, -6 -4, -2

+4, -1 +2, -5 +1, -5 0, -5 -1, -5 +3, -5 -3, -5 -5, -5 +5, -5 -2, -5 -4, -1

+4, 0 +2, -3 +1, -3 0, -3 -1, -3 +3, -3 -3, -3 -5, -3 +5, -3 -2, -3 -4, 0

+4, +1 +2, +3 +1, +3 0, +3 -1, +3 +3, +3 -3, +3 -5, +3 +5, +3 -2, +3 -4, +1

+4, +2 +2, -1 +1, -1 0, -1 -1, -1 +3, -1 -3, -1 -5, -1 +5, -1 -2, -1 -4, +2

+4, +3 +2, 0 +1, 0 0, 0 -1, 0 +3, 0 -3, 0 -5, 0 +5, 0 -2, 0 -4, +3

+4, +4 +2, +1 +1, +1 0, +1 -1, +1 +3, +1 -3, +1 -5, +1 +5, +1 -2, +1 -4, +4

+4, +5 +2, +2 +1, +2 0, +2 -1, +2 +3, +2 -2, +2 -5, +2 +5, +2 -2, +2 -4, +5

+4, +6 +45 +4 +3, +4 +2, +4 +1, +4 0, +4 -1, +4 -2, +4 -3, +4 -5, +4 -4, +6

/
The resulting shape of the valid solutions would contain two red columns protruding upwards from
the opposing corners for 125 consecutive slates (all slates except the exterior shell in yellow). The bottom
slate (second from bottom) would have a horizontal and vertical row starting from the bottom-left corner
(in blue) connecting the two columns, and the top slate (second from top) would contain a mirror image of
the same (hence the true geometrical meaning of symmetrical difference in set theory).

In the picture below the red and blue lines represent the viable solutions to systems of
congruences yielding a nucleus of length 8 (all six lines represent a series orthogonal projections through
successive cells).

/
If we attempt to visualize the occurrences inside a rank-4 tensor by examine the (x,y,z) space
moving through w-space, we see that the yellow shell remains intact, and the when the prism passes
through the w = +2 and w = -2 planes, it reactivates some of the rejected solutions (any solution that has
at least one complimentary -2 or +2 remainder).

Likewise when moving through 5 dimensions we get a very similar result to the 2D case. As you can see if we were to
translate this entire figure upwards (sixth dimension), we would get another prism with two red columns, but connected by four blue
walls instead of blue lines (closer examination would of course reveal that the walls are very perforated). The red prisms represent
the intersection of the complimentary (+2,-2) and (-2, +2), activating all non-yellow solutions (no perforations), and the 6-D version of
the structure would be capped on both sides with pure yellow prisms. Ironically the 5-D representation is easier to understand than
the above 4-D representation, since the 5-D representation closely resembles the original 2-D matrix and there are both 10 distinct
2D subplanes and 10 distinct 3D subspaces in 5-dimensional space).

Each of the (+2,-2) and (-2,+2) vectors in each of the ten possible planes of (x,y,z,w,t), are all being orthogonally projected
through the ten distinct 3-D spaces of (x,y,z,w,t), whose bounds only restrict the yellow regions. Since (w,t) = (+2, -2) is a solution, it
is orthogonally projected unto the entire (x,y,z) prism, which is one of the ten possible 3D subspaces of (x,y,z,w,t), and is why we
see the entire prism colored red (except the shell) at (w,t) = (+2,-2) and (w,t) = (-2+2).

/
Lemma 1.9.1.5; The size of M10 , S = {2, 3, 11, 13} and |S| = 4 , {2,3} ∈ S

Let S = {2, 3,11, 13}

∀μ ∈ M10, μ ≡ 0 mod 2 ≡ 0 mod 3 ≡ a1 mod 11 ≡ a2 mod 13; a1 , a2 =/ ± 5

Both 11 or 13 must be belligerent, such that they sieve (μ − 1) and (μ + 1) , {11, 13} ∈ B.

k μ−5 μ−1 μ μ+1 μ+5

ti ui ti+1

ξ (k) ∅ 2 3 2 bx 2 by 2 3 2 ∅

i=n i=n i=n


ϖ (M10) = ∏ (si − 2) − 2 ∏ (si − 3) + ∏ (si − 4) , n ≥ 4
i=3 i=3 i=3

Proof:

Let us compile all possible systems of congruences in respect to 11 and 13 into a matrix whose
dimensions are 11x13 = 143.

/
k ≡ a mod 11 ≡ b mod 13 ; − 5 ≤ a ≤+ 5, − 6 ≤ b ≤+ 6, a, b ∈ Z .

k ≡ 0 mod 6
+5, -6 +4, -6 +3, -6 +2, -6 +1, -6 0, -6 -1, -6 -2, -6 -3, -6 -4, -6 -5, -6

+5, -5 +4, -5 +3, -5 +2, -5 +1, -5 0, -5 -1, -5 -2, -5 -3, -5 -4, -5 -5, -5

+5, -4 +4, -4 +3, -4 +2, -4 +1, -4 0, -4 -1, -4 -2, -4 -3, -4 -4, -4 -5, -4

+5, -3 +4, -3 +3, -3 +2, -3 +1, -3 0, -3 -1, -3 -2, -3 -3, -3 -4, -3 -5, -3

+5, -2 +4, -2 +3, -2 +2, -2 +1, -2 0, -2 -1, -2 -2, -2 -3, -2 -4, -2 -5, -2

+5, -1 +4, -1 +3, -1 +2, -1 +1, -1 0, -1 -1, -1 -2, -1 -3, -1 -4, -1 -5, -1

+5, 0 +4, 0 +3, 0 +2, 0 +1, 0 0, 0 -1, 0 -2, 0 -2, 0 -4, 0 -5, 0

+5, +1 +4, +1 +3, +1 +2, +1 +1, +1 0, +1 -1, +1 -2, +1 -2, +1 -4, +1 -5, +1

+5, +2 +4, +2 +3, +2 +2, +2 +1, +2 0, +2 -1, +2 -2, +2 -2, +2 -4, +2 -5, +2

+5, +3 +4, +3 +3, +3 +2, +3 +1, +3 0, +3 -1, +3 -2, +3 -2, +3 -4, +3 -5, +3

+5, +4 +4, +4 +3, +4 +2, +4 +1, +4 0, +4 -1, +4 -2, +4 -3, +4 -4, +4 -5, +4

+5, +5 +4, +5 +3, +5 +2, +5 +1, +5 0, +5 -1, +5 -2, +5 -3, +5 -4, +5 -5, +5

+5, +6 +4, +6 +3, +6 +2, +6 +1, +6 0, +6 -1, +6 -2, +6 -3, +6 -4, +6 -5, +6

All of the above 143 entries containing a positive or negative 5 represent systems of congruences
which shall not yield a nucleus of length 5, they are highlighted in yellow. Bear in mind that +5 = -2 mod 7,
and -5 = +2 mod 7, and 0 = -5 mod 5 and 0 = +5 mod 5, which is why they are excluded from the general
counting formulas.

.
Also note that nuclei of length ten require a minimum of two belligerent primes.
The set S = {2,3, p } can never sieve 10 consecutive integers.

For now let us continue with this two-dimensional example of {11, 13} and rearrange the
perimeter.

/
k ≡ 0 mod 6
+5, -5 +4, -5 +3, -5 +2, -5 +1, -5 0, -5 -1, -5 -2, -5 -3, -5 -4, -5 -5, -5

+5, -6 +4, -6 +3, -6 +2, -6 +1, -6 0, -6 -1, -6 -2, -6 -3, -6 -4, -6 -5, -6

+5, -4 +4, -4 +3, -4 +2, -4 +1, -4 0, -4 -1, -4 -2, -4 -3, -4 -4, -4 -5, -4

+5, -3 +4, -3 +3, -3 +2, -3 +1, -3 0, -3 -1, -3 -2, -3 -3, -3 -4, -3 -5, -3

+5, -2 +4, -2 +3, -2 +2, -2 +1, -2 0, -2 -1, -2 -2, -2 -3, -2 -4, -2 -5, -2

+5, -1 +4, -1 +3, -1 +2, -1 +1, -1 0, -1 -1, -1 -2, -1 -3, -1 -4, -1 -5, -1

+5, 0 +4, 0 +3, 0 +2, 0 +1, 0 0, 0 -1, 0 -2, 0 -2, 0 -4, 0 -5, 0

+5, +1 +4, +1 +3, +1 +2, +1 +1, +1 0, +1 -1, +1 -2, +1 -2, +1 -4, +1 -5, +1

+5, +2 +4, +2 +3, +2 +2, +2 +1, +2 0, +2 -1, +2 -2, +2 -2, +2 -4, +2 -5, +2

+5, +3 +4, +3 +3, +3 +2, +3 +1, +3 0, +3 -1, +3 -2, +3 -2, +3 -4, +3 -5, +3

+5, +4 +4, +4 +3, +4 +2, +4 +1, +4 0, +4 -1, +4 -2, +4 -3, +4 -4, +4 -5, +4

+5, +6 +4, +6 +3, +6 +2, +6 +1, +6 0, +6 -1, +6 -2, +6 -3, +6 -4, +6 -5, +6

+5, +5 +4, +5 +3, +5 +2, +5 +1, +5 0, +5 -1, +5 -2, +5 -3, +5 -4, +5 -5, +5

i=n
Thus the initial set of possible solutions has a size of ∏ (si − 2) . Call this Set A.
i=3
However, within the remaining solutions, there exists yet another subset that does not permit a
nucleus of length 10 to occur. These would be ALL solutions not containing a +1 in them, since there
must exist at least prime, either 11 or 13, that divides μ − 1 , such that μ ≡ + 1 mod 11 or + 1 mod 13

The remaining solutions not containing a +1 shall be highlighted in blue. The viable solutions will
highlighted in light red.

k ≡ 0 mod 6
+5, -5 +4, -5 +3, -5 +2, -5 +1, -5 0, -5 -1, -5 -2, -5 -3, -5 -4, -5 -5, -5

+5, -6 +4, -6 +3, -6 +2, -6 +1, -6 0, -6 -1, -6 -2, -6 -3, -6 -4, -6 -5, -6

+5, -4 +4, -4 +3, -4 +2, -4 +1, -4 0, -4 -1, -4 -2, -4 -3, -4 -4, -4 -5, -4

+5, -3 +4, -3 +3, -3 +2, -3 +1, -3 0, -3 -1, -3 -2, -3 -3, -3 -4, -3 -5, -3

+5, -2 +4, -2 +3, -2 +2, -2 +1, -2 0, -2 -1, -2 -2, -2 -3, -2 -4, -2 -5, -2

+5, -1 +4, -1 +3, -1 +2, -1 +1, -1 0, -1 -1, -1 -2, -1 -3, -1 -4, -1 -5, -1

+5, 0 +4, 0 +3, 0 +2, 0 +1, 0 0, 0 -1, 0 -2, 0 -2, 0 -4, 0 -5, 0

+5, +1 +4, +1 +3, +1 +2, +1 +1, +1 0, +1 -1, +1 -2, +1 -2, +1 -4, +1 -5, +1

+5, +2 +4, +2 +3, +2 +2, +2 +1, +2 0, +2 -1, +2 -2, +2 -2, +2 -4, +2 -5, +2

+5, +3 +4, +3 +3, +3 +2, +3 +1, +3 0, +3 -1, +3 -2, +3 -2, +3 -4, +3 -5, +3

+5, +4 +4, +4 +3, +4 +2, +4 +1, +4 0, +4 -1, +4 -2, +4 -3, +4 -4, +4 -5, +4

+5, +6 +4, +6 +3, +6 +2, +6 +1, +6 0, +6 -1, +6 -2, +6 -3, +6 -4, +6 -5, +6

+5, +5 +4, +5 +3, +5 +2, +5 +1, +5 0, +5 -1, +5 -2, +5 -3, +5 -4, +5 -5, +5

/
Now we shall rearrange the above matrix one more time:

+5, -5 +1, -5 0, -5 -1, -5 -2, -5 -3, -5 -4, -5 +4, -5 +3, -5 +2, -5 -5, -5

+5, -6 +1, +2 0, +2 -1, +2 -2, +2 -3, +2 -4, +2 +4, +2 +3, +2 +2, +2 -5, -6

+5, -4 +1, +3 0, +3 -1, +3 -2, +3 -3, +3 -4, +3 +4, +3 +3, +3 +2, +3 -5, -4

+5, -3 +1, +4 0, +4 -1, +4 -2, +4 -3, +4 -4, +4 +4, +4 +3, +4 +2, +4 -5, -3

+5, -2 +1, +6 0, +6 -1, +6 -2, +6 -3, +6 -4, +6 +4, +6 +3, +6 +2, +6 -5, -2

+5, -1 +1, -6 0, -6 -1, -6 -2, -6 -3, -6 -4, -6 +4, -6 +3, -6 +2, -6 -5, -1

+5, 0 +1, -4 0, -4 -1, -4 -2, -4 -3, -4 -4, -4 +4, -4 +3, -4 +2, -4 -5, 0

+5, +1 +1, -3 0, -3 -1, -3 -2, -3 -3, -3 -4, -3 +4, -3 +3, -3 +2, -3 -5, +1

+5, +2 +1, -2 0, -2 -1, -2 -2, -2 -3, -2 -4, -2 +4, -2 +3, -2 +2, -2 -5, +2

+5, +3 +1, -1 0, -1 -1, -1 -2, -1 -3, -1 -4, -1 +4, -1 +3, -1 +2, -1 -5, +3

+5, +4 +1, 0 0, 0 -1, 0 -2, 0 -3, 0 -4, 0 +4, 0 +3, 0 +2, 0 -5, +4

+5, +6 +1, +1 0, +1 -1, +1 -2, +1 -3, +1 -4, +1 +4, +1 +3, +1 +2, +1 -5, +6

+5, +5 +1, +5 0, +5 -1, +5 -2, +5 -3, +5 -4, +5 +4, +5 -3, +5 +2, +5 -5, +5

The set of invalid solutions has a size of 8*10. Thus the initial set of possible solutions has a size
i=n
of ∏ (si − 3) , as this would apply to any primes numbers in place of 11 and 13 in S (aside from 5 and 7).
i=3

Call this set of discard solutions (in blue) Set C. As you can see these rearranged matrices are
taking on the same form of the previous matrices for counting gaps of 8, which is why they produce the
same formula.

/
However,, there exists STILL yet another subset of EQUAL size that does not permit a nucleus of
length 10 to occur. These would be ALL solutions not containing a -1 in them, since there must exist at
least prime, either 11 or 13, that divides μ + 1 , such that μ ≡ − 1 mod 11 or − 1 mod 13

The remaining solutions not containing a -2 shall be highlighted in blue. The viable solutions will
highlighted in light red.

+5, -5 +1, -5 0, -5 -1, -5 -2, -5 -3, -5 -4, -5 +4, -5 +3, -5 +2, -5 -5, -5

+5, -6 +1, +2 0, +2 -1, +2 -2, +2 -3, +2 -4, +2 +4, +2 +3, +2 +2, +2 -5, -6

+5, -4 +1, +3 0, +3 -1, +3 -2, +3 -3, +3 -4, +3 +4, +3 +3, +3 +2, +3 -5, -4

+5, -3 +1, +4 0, +4 -1, +4 -2, +4 -3, +4 -4, +4 +4, +4 +3, +4 +2, +4 -5, -3

+5, -2 +1, +6 0, +6 -1, +6 -2, +6 -3, +6 -4, +6 +4, +6 +3, +6 +2, +6 -5, -2

+5, -1 +1, -6 0, -6 -1, -6 -2, -6 -3, -6 -4, -6 +4, -6 +3, -6 +2, -6 -5, -1

+5, 0 +1, -4 0, -4 -1, -4 -2, -4 -3, -4 -4, -4 +4, -4 +3, -4 +2, -4 -5, 0

+5, +1 +1, -3 0, -3 -1, -3 -2, -3 -3, -3 -4, -3 +4, -3 +3, -3 +2, -3 -5, +1

+5, +2 +1, -2 0, -2 -1, -2 -2, -2 -3, -2 -4, -2 +4, -2 +3, -2 +2, -2 -5, +2

+5, +3 +1, -1 0, -1 -1, -1 -2, -1 -3, -1 -4, -1 +4, -1 +3, -1 +2, -1 -5, +3

+5, +4 +1, 0 0, 0 -1, 0 -2, 0 -3, 0 -4, 0 +4, 0 +3, 0 +2, 0 -5, +4

+5, +6 +1, +1 0, +1 -1, +1 -2, +1 -3, +1 -4, +1 +4, +1 +3, +1 +2, +1 -5, +6

+5, +5 +1, +5 0, +5 -1, +5 -2, +5 -3, +5 -4, +5 +4, +5 -3, +5 +2, +5 -5, +5

Rearranged, call this set of discarded solutions (in blue) Set E (row -/+ 6 swapped with -/+ 1)
+5, -5 +1, -5 0, -5 +2, -5 -2, -5 -3, -5 -4, -5 +4, -5 +3, -5 -1, -5 -5, -5

+5, -1 +1, -1 0, -1 +2, -1 -2, -1 +3, -1 -4, -1 +4, -1 +3, -1 -1, +1 -5, -1

+5, -4 +1, +3 0, +3 +2, +2 -2, +3 -3, +3 -4, +3 +4, +3 +3, +3 -1, +3 -5, -4

+5, -3 +1, +4 0, +4 +2, +3 -2, +4 -3, +4 -4, +4 +4, +4 +3, +4 -1, +4 -5, -3

+5, -2 +1, +6 0, +6 +2, +4 -2, +6 -3, +6 -4, +6 +4, +6 +3, +6 -1, +6 -5, -2

+5, -6 +1, -6 0, -6 +2, +6 -2, -6 -3, -6 -4, -6 +4, -6 +3, -6 -1, -6 -5, -6

+5, 0 +1, -4 0, -4 +2, -6 -2, -4 -3, -4 -4, -4 +4, -4 +3, -4 -1, -4 -5, 0

+5, +6 +1, -3 0, -3 +2, -4 -2, -3 -3, -3 -4, -3 +4, -3 +3, -3 -1, -3 -5, +6

+5, +2 +1, -2 0, -2 +2, -3 -2, -2 -3, -2 -4, -2 +4, -2 +3, -2 -1, -2 -5, +2

+5, +3 +1, -1 0, -1 +2, -2 -2, -1 -3, -1 -4, -1 +4, -1 +3, -1 -1, -1 -5, +3

+5, +4 +1, 0 0, 0 -2, 0 -2, 0 -3, 0 -4, 0 +4, 0 +3, 0 -1, 0 -5, +4

+5, +1 +1, +1 0, +1 -2, +1 -2, +1 -3, +1 -4, +1 +4, +1 +3, +1 -1, +1 -5, +1

+5, +5 +1, +5 0, +5 +2, +5 -2, +5 -3, +5 -4, +5 +4, +5 -3, +5 -1, +5 -5, +5

/
Let us combine the matrices of the discarded solutions for both +2 and -2 (notice that I never
rearrange the yellow entries, since they act as indices for the matrices). The solutions that were discarded
by both of the previous matrices are highlighted in purple, and the solutions discarded only by one of the
matrices are highlighted in green. We result with only TWO viable solutions.

Recall that set A is the initial set of all possible solutions, Set C is the set of discarded solutions
from the first matrix, and Set E is the set of discarded solutions from the third matrix.

Let L be the solutions in purple; F represents the intersection of C and E.


L = C ⋂ E.

Let G be the solutions in green; G represents the symmetric difference of C an E.


G = C △ E.

+5, -5 +1, -5 0, -5 +2, -5 -2, -5 -3, -5 -4, -5 +4, -5 +3, -5 -1, -5 -5, -5

+5, -1 +1, -1 0, -1 +2, -1 -2, -1 +3, -1 -4, -1 +4, -1 +3, -1 -1, -1 -5, -1

+5, -4 +1, +3 0, +3 +2, +2 -2, +3 -3, +3 -4, +3 +4, +3 +3, +3 -1, +3 -5, -4

+5, -3 +1, +4 0, +4 +2, +3 -2, +4 -3, +4 -4, +4 +4, +4 +3, +4 -1, +4 -5, -3

+5, -2 +1, +6 0, +6 +2, +4 -2, +6 -3, +6 -4, +6 +4, +6 +3, +6 -1, +6 -5, -2

+5, -6 +1, -6 0, -6 +2, +6 -2, -6 -3, -6 -4, -6 +4, -6 +3, -6 -1, -6 -5, -6

+5, 0 +1, -4 0, -4 +2, -6 -2, -4 -3, -4 -4, -4 +4, -4 +3, -4 -1, -4 -5, 0

+5, +6 +1, -3 0, -3 +2, -4 -2, -3 -3, -3 -4, -3 +4, -3 +3, -3 -1, -3 -5, +6

+5, +2 +1, -2 0, -2 +2, -3 -2, -2 -3, -2 -4, -2 +4, -2 +3, -2 -1, -2 -5, +2

+5, +3 +1, -1 0, -1 +2, -2 -2, -1 -3, -1 -4, -1 +4, -1 +3, -1 -1, -1 -5, +3

+5, +4 +1, 0 0, 0 -2, 0 -2, 0 -3, 0 -4, 0 +4, 0 +3, 0 -1, 0 -5, +4

+5, +1 +1, +1 0, +1 -2, +1 -2, +1 -3, +1 -4, +1 +4, +1 +3, +1 -1, +1 -5, +1

+5, +5 +1, +5 0, +5 +2, +5 -2, +5 -3, +5 -4, +5 +4, +5 -3, +5 -1, +5 -5, +5

We must now find a way to numerically subtract all of green and purple solutions. The answer is
the exact same as before: Subtract both C and E, and then add back G, since G was subtracted twice, as
G is a subset of both C and E.

We know that:
i=4
|A| = 9*11 = (11-2)(13-2) = ∏(si − 2)
i=3
i=4
|C| and |E| = 8*10 = (11-3)(13-3) = ∏(si − 3)
i=3
i=4
|G| = 7*9 = (11-4)(13 - 4) = ∏(si − 4)
i=3

i=n i=n i=n


Thus |A| - |C| - |E| + |G| = ∏ (si − 2) − 2 ∏ (si − 3) + ∏ (si − 4)
i=3 i=3 i=3

/
Now let us generalize this for larger sets of primes, and therefore for rank-( n − 2 ) tensors. We will
use 11*13 as the slate of a rank-3 tensor, and add a fifth prime to S. S = {2,3,11,13, 127}.

For μ ≡ ± 5 mod 127 , we would get two completely eliminated slates. We would place these two slates at
the top and bottom faces of the rank-3 tensor (rectangular prism of height 127).

+5, -5 +1, -5 0, -5 +2, -5 -2, -5 -3, -5 -4, -5 +4, -5 +3, -5 -1, -5 -5, -5

+5, -1 +1, -1 0, -1 +2, -1 -2, -1 +3, -1 -4, -1 +4, -1 +3, -1 -1, -1 -5, -1

+5, -4 +1, +3 0, +3 +2, +2 -2, +3 -3, +3 -4, +3 +4, +3 +3, +3 -1, +3 -5, -4

+5, -3 +1, +4 0, +4 +2, +3 -2, +4 -3, +4 -4, +4 +4, +4 +3, +4 -1, +4 -5, -3

+5, -2 +1, +6 0, +6 +2, +4 -2, +6 -3, +6 -4, +6 +4, +6 +3, +6 -1, +6 -5, -2

+5, -6 +1, -6 0, -6 +2, +6 -2, -6 -3, -6 -4, -6 +4, -6 +3, -6 -1, -6 -5, -6

+5, 0 +1, -4 0, -4 +2, -6 -2, -4 -3, -4 -4, -4 +4, -4 +3, -4 -1, -4 -5, 0

+5, +6 +1, -3 0, -3 +2, -4 -2, -3 -3, -3 -4, -3 +4, -3 +3, -3 -1, -3 -5, +6

+5, +2 +1, -2 0, -2 +2, -3 -2, -2 -3, -2 -4, -2 +4, -2 +3, -2 -1, -2 -5, +2

+5, +3 +1, -1 0, -1 +2, -2 -2, -1 -3, -1 -4, -1 +4, -1 +3, -1 -1, -1 -5, +3

+5, +4 +1, 0 0, 0 -2, 0 -2, 0 -3, 0 -4, 0 +4, 0 +3, 0 -1, 0 -5, +4

+5, +1 +1, +1 0, +1 -2, +1 -2, +1 -3, +1 -4, +1 +4, +1 +3, +1 -1, +1 -5, +1

+5, +5 +1, +5 0, +5 +2, +5 -2, +5 -3, +5 -4, +5 +4, +5 -3, +5 -1, +5 -5, +5

For μ ≡ x mod 127, x =/ + 5, + 1, − 1, − 5 , we would result with 123 consecutive slates identical to the
2-D solutions.

+5, -5 +1, -5 0, -5 +2, -5 -2, -5 -3, -5 -4, -5 +4, -5 +3, -5 -1, -5 -5, -5

+5, -1 +1, -1 0, -1 +2, -1 -2, -1 +3, -1 -4, -1 +4, -1 +3, -1 -1, -1 -5, -1

+5, -4 +1, +3 0, +3 +2, +2 -2, +3 -3, +3 -4, +3 +4, +3 +3, +3 -1, +3 -5, -4

+5, -3 +1, +4 0, +4 +2, +3 -2, +4 -3, +4 -4, +4 +4, +4 +3, +4 -1, +4 -5, -3

+5, -2 +1, +6 0, +6 +2, +4 -2, +6 -3, +6 -4, +6 +4, +6 +3, +6 -1, +6 -5, -2

+5, -6 +1, -6 0, -6 +2, +6 -2, -6 -3, -6 -4, -6 +4, -6 +3, -6 -1, -6 -5, -6

+5, 0 +1, -4 0, -4 +2, -6 -2, -4 -3, -4 -4, -4 +4, -4 +3, -4 -1, -4 -5, 0

+5, +6 +1, -3 0, -3 +2, -4 -2, -3 -3, -3 -4, -3 +4, -3 +3, -3 -1, -3 -5, +6

+5, +2 +1, -2 0, -2 +2, -3 -2, -2 -3, -2 -4, -2 +4, -2 +3, -2 -1, -2 -5, +2

+5, +3 +1, -1 0, -1 +2, -2 -2, -1 -3, -1 -4, -1 +4, -1 +3, -1 -1, -1 -5, +3

+5, +4 +1, 0 0, 0 -2, 0 -2, 0 -3, 0 -4, 0 +4, 0 +3, 0 -1, 0 -5, +4

+5, +1 +1, +1 0, +1 -2, +1 -2, +1 -3, +1 -4, +1 +4, +1 +3, +1 -1, +1 -5, +1

+5, +5 +1, +5 0, +5 +2, +5 -2, +5 -3, +5 -4, +5 +4, +5 -3, +5 -1, +5 -5, +5

/
For μ ≡ + 1 mod 127 , we would result with a single slate that enables all discarded solutions containing a
-1, since the nucleus would contain a belligerent prime at both of the required locations. These
reactivated solutions are highlighted in blue. We would place this above the bottom slate.

+5, -5 +1, -5 0, -5 +2, -5 -2, -5 -3, -5 -4, -5 +4, -5 +3, -5 -1, -5 -5, -5

+5, -1 +1, -1 0, -1 +2, -1 -2, -1 +3, -1 -4, -1 +4, -1 +3, -1 -1, -1 -5, -1

+5, -4 +1, +3 0, +3 +2, +2 -2, +3 -3, +3 -4, +3 +4, +3 +3, +3 -1, +3 -5, -4

+5, -3 +1, +4 0, +4 +2, +3 -2, +4 -3, +4 -4, +4 +4, +4 +3, +4 -1, +4 -5, -3

+5, -2 +1, +6 0, +6 +2, +4 -2, +6 -3, +6 -4, +6 +4, +6 +3, +6 -1, +6 -5, -2

+5, -6 +1, -6 0, -6 +2, +6 -2, -6 -3, -6 -4, -6 +4, -6 +3, -6 -1, -6 -5, -6

+5, 0 +1, -4 0, -4 +2, -6 -2, -4 -3, -4 -4, -4 +4, -4 +3, -4 -1, -4 -5, 0

+5, +6 +1, -3 0, -3 +2, -4 -2, -3 -3, -3 -4, -3 +4, -3 +3, -3 -1, -3 -5, +6

+5, +2 +1, -2 0, -2 +2, -3 -2, -2 -3, -2 -4, -2 +4, -2 +3, -2 -1, -2 -5, +2

+5, +3 +1, -1 0, -1 +2, -2 -2, -1 -3, -1 -4, -1 +4, -1 +3, -1 -1, -1 -5, +3

+5, +4 +1, 0 0, 0 -2, 0 -2, 0 -3, 0 -4, 0 +4, 0 +3, 0 -1, 0 -5, +4

+5, +1 +1, +1 0, +1 -2, +1 -2, +1 -3, +1 -4, +1 +4, +1 +3, +1 -1, +1 -5, +1

+5, +5 +1, +5 0, +5 +2, +5 -2, +5 -3, +5 -4, +5 +4, +5 -3, +5 -1, +5 -5, +5

For μ ≡ − 1 mod 127 , we would result with a single slate that enables all discarded solutions containing a
+1, since the nucleus would contain a belligerent prime at both of the required locations. These
reactivated solutions are highlighted in blue. We would place this below the top slate.

+5, -5 +1, -5 0, -5 +2, -5 -2, -5 -3, -5 -4, -5 +4, -5 +3, -5 -1, -5 -5, -5

+5, -1 +1, -1 0, -1 +2, -1 -2, -1 +3, -1 -4, -1 +4, -1 +3, -1 -1, -1 -5, -1

+5, -4 +1, +3 0, +3 +2, +2 -2, +3 -3, +3 -4, +3 +4, +3 +3, +3 -1, +3 -5, -4

+5, -3 +1, +4 0, +4 +2, +3 -2, +4 -3, +4 -4, +4 +4, +4 +3, +4 -1, +4 -5, -3

+5, -2 +1, +6 0, +6 +2, +4 -2, +6 -3, +6 -4, +6 +4, +6 +3, +6 -1, +6 -5, -2

+5, -6 +1, -6 0, -6 +2, +6 -2, -6 -3, -6 -4, -6 +4, -6 +3, -6 -1, -6 -5, -6

+5, 0 +1, -4 0, -4 +2, -6 -2, -4 -3, -4 -4, -4 +4, -4 +3, -4 -1, -4 -5, 0

+5, +6 +1, -3 0, -3 +2, -4 -2, -3 -3, -3 -4, -3 +4, -3 +3, -3 -1, -3 -5, +6

+5, +2 +1, -2 0, -2 +2, -3 -2, -2 -3, -2 -4, -2 +4, -2 +3, -2 -1, -2 -5, +2

+5, +3 +1, -1 0, -1 +2, -2 -2, -1 -3, -1 -4, -1 +4, -1 +3, -1 -1, -1 -5, +3

+5, +4 +1, 0 0, 0 -2, 0 -2, 0 -3, 0 -4, 0 +4, 0 +3, 0 -1, 0 -5, +4

+5, +1 +1, +1 0, +1 -2, +1 -2, +1 -3, +1 -4, +1 +4, +1 +3, +1 -1, +1 -5, +1

+5, +5 +1, +5 0, +5 +2, +5 -2, +5 -3, +5 -4, +5 +4, +5 -3, +5 -1, +5 -5, +5

/
The resulting shape of the valid solutions would contain two red columns protruding upwards from
the opposing corners for 125 consecutive slates (all slates except the exterior shell in yellow). The bottom
slate (second from bottom) would have a horizontal and vertical row starting from the bottom-left corner
(in blue) connecting the two columns, and the top slate (second from top) would contain a mirror image of
the same ( again, hence the true geometrical meaning of symmetrical difference in set theory).

In the picture below the red and blue lines represent the viable solutions to systems of
congruences yielding a nucleus of length 10.

QED

/
Lemma 1.9.2.1; The size of M12 , S = {2, 3, 13, 17, 19} and |S| = 5 , {2,3} ∈ S
Set Theory Derivation

Let S = {2, 3,13, 17,19}

∀μ ∈ M6, μ ≡ − 1 mod 2 ≡ − 1 mod 3 ≡ a1 mod 13 ≡ a2 mod 17 ≡ a3 mod 19; a1 , a2 =/ ± 6

13, 17 and 19 must be belligerent, such that they sieve (μ − 4) and (μ − 0) and (μ + 2) , {11, 17,19} ∈ B.

μ ≡ 5 mod 6

k μ−6 μ−4 μ μ+2 μ+6

ti ui ti+1

ξ (k) ∅ 2 bx 2 3 2 by 2 bz 2 3 2 ∅

(∏ )
i=n i=n i=n i=n
ϖ (M12) = 2 (si − 2) − 3 ∏ (si − 3) + 3 ∏ (si − 4) − ∏(si − 5) ,n ≥ 5
i=3 i=3 i=3 i=3

As when we counted the size of M6, we had to double the count since there existed an additive
inverse for each system of congruences which yielded a nucleus of length 6. Likewise for each nucleus of
length 12, whose centrum is congruent to 5 mod 6, there exists an additive inverse for a centrum
congruent to 1 mod 6, which by definition must also be at the center of a nucleus of length 12 which was
not yet counted due to the uniquity demands of Chinese Remainder Theorem.

Also note that we require of minimum of three belligerent primes in S, since the primes numbers 2
and 3 leave a total of three unsieved integers within the nucleus.

This set theory derivation shall lay the groundwork for Set Theory Interpretation of counting gaps
(nucleic sets). In fact Set Theory is only basic mathematical tool that can deliver a rigid proof for the
General Formulas and the Special Formulas. Geometry fails since we cannot examine shapes in four or
more dimensions, we can only understand the geometry of higher dimensions through set theory and
linear algebra.

Our count must equal 6, since there are six solutions that generate a nucleus of length 12.
μ ≡ 5 mod 6
μ1 ≡ + 4 mod 13 ≡ 0 mod 17 ≡ − 2 mod 19
μ2 ≡ + 4 mod 13 ≡ 0 mod 19 ≡ − 2 mod 17
μ3 ≡ + 4 mod 17 ≡ 0 mod 13 ≡ − 2 mod 19
μ4 ≡ + 4 mod 17 ≡ 0 mod 19 ≡ − 2 mod 13
μ5 ≡ + 4 mod 19 ≡ 0 mod 13 ≡ − 2 mod 17
μ6 ≡ + 4 mod 19 ≡ 0 mod 17 ≡ − 2 mod 13

Each of these solutions corresponds to a cell at the end of a vector, (ie. (+4, 0, -2))

/
Proof:

S = {2,3,13,17,19}; |S| = n = 5

No prime in S may divide μ − 6 or μ + 6 . Since μ ≡ − 1 mod 2 ≡ − 1 mod 3 .

Step 0:
We the first take the initial set of all systems of congruences in respect of {13,17,19}, this would be the
following:

μ ≡ x1 mod 13 ≡ x2 mod 17 ≡ x3 mod 19; − 6 ≤ x1 ≤ 6, − 8 ≤ x2 ≤ 8 , − 9 ≤ x3 ≤ 9

We place these solutions in set Y.


i=n
|Y| = ∏ (si ) = (13)(17)(19) = 4199 . This represents a tensor of dimensions 13x17x19.
i=3

Step 1,1:
Of these solutions we eliminate all of which contain a +6 or - 6 in any of the remainders.

μ ≡ x1 mod 13 ≡ x2 mod 17 ≡ x3 mod 19; − 6 ≤ x1 ≤ 6, − 8 ≤ x2 ≤ 8 , − 9 ≤ x3 ≤ 9


x =/ ± 6, ∀x .

We place these solutions in set A.


i=n
|A| = ∏ (si − 2) = (11)(15)(17) = 2805 . This represents a tensor of dimensions 11x15x17.
i=3

Step 2, 1:
Of these solutions we eliminate all of which do not contain a +4 in any of the remainders, since at least
one prime must divide μ + 4 .

μ ≡ x1 mod 13 ≡ x2 mod 17 ≡ x3 mod 19; − 6 ≤ x1 ≤ 6, − 8 ≤ x2 ≤ 8 , − 9 ≤ x3 ≤ 9


x =/ ± 6, ∀x; x =/ + 4, ∀x

We place these solutions in set B1.


i=n
|B1| = ∏ (si − 3) = (10)(14)(16) = 2240 . This represents a tensor of dimensions 10x14x16.
i=3

/
Step 2, 2:
Of these solutions we eliminate all of which do not contain a 0 in any of the remainders, since at least one
prime must divide μ + 0 .

μ ≡ x1 mod 13 ≡ x2 mod 17 ≡ x3 mod 19; − 6 ≤ x1 ≤ 6, − 8 ≤ x2 ≤ 8 , − 9 ≤ x3 ≤ 9


x =/ ± 6, ∀x; x =/ 0, ∀x

We place these solutions in set B2.


i=n
|B2| = ∏ (si − 3) = (10)(14)(16) = 2240 . This represents a tensor of dimensions 10x14x16.
i=3

Step 2, 3:
Of these solutions we eliminate all of which do not contain a -2 in any of the remainders, since at least
one prime must divide μ + 2 .

μ ≡ x1 mod 13 ≡ x2 mod 17 ≡ x3 mod 19; − 6 ≤ x1 ≤ 6, − 8 ≤ x2 ≤ 8 , − 9 ≤ x3 ≤ 9


x =/ ± 6, ∀x; x =/ − 2, ∀x

We place these solutions in set B3.


i=n
|B3| = ∏ (si − 3) = (10)(14)(16) = 2240 . This represents a tensor of dimensions 10x14x16.
i=3

Step 3, 1:
Of these solutions we eliminate all of which do not contain both +4 and 0 in any of the remainders, since
at least two distinct prime must divide μ − 4 and μ + 0 .

μ ≡ x1 mod 13 ≡ x2 mod 17 ≡ x3 mod 19; − 6 ≤ x1 ≤ 6, − 8 ≤ x2 ≤ 8 , − 9 ≤ x3 ≤ 9


x =/ ± 6, ∀x; x =/ + 4, 0 ∀x

We place these solutions in set C1.


i=n
|C1| = ∏ (si − 4) = (9)(13)(15) = 1755 . This represents a tensor of dimensions 9x13x15.
i=3

Step 3, 2:
Of these solutions we eliminate all of which do not contain both +4 and -2 in any of the remainders, since
at least two distinct prime must divide μ − 4 and μ + 2 .

μ ≡ x1 mod 13 ≡ x2 mod 17 ≡ x3 mod 19; − 6 ≤ x1 ≤ 6, − 8 ≤ x2 ≤ 8 , − 9 ≤ x3 ≤ 9


x =/ ± 6, ∀x; x =/ + 4, − 2 ∀x

We place these solutions in set C2.


i=n
|C2| = ∏ (si − 4) = (9)(13)(15) = 1755 . This represents a tensor of dimensions 9x13x15
i=3

/
Step 3, 3:
Of these solutions we eliminate all of which do not contain both 0 and -2 in any of the remainders, since
at least two distinct prime must divide μ + 0 and μ + 2 .

μ ≡ x1 mod 13 ≡ x2 mod 17 ≡ x3 mod 19; − 6 ≤ x1 ≤ 6, − 8 ≤ x2 ≤ 8 , − 9 ≤ x3 ≤ 9


x =/ ± 6, ∀x; x =/ 0, − 2 ∀x

We place these solutions in set C3.


i=n
|C3| = ∏ (si − 4) = (9)(13)(15) = 1755 . This represents a tensor of dimensions 9x13x15.
i=3

Step 4, 1:
Of these solutions we eliminate all of which do not contain a +4, 0 and a -2 in any of the remainders,
since at least three distinct prime must divide μ − 4 , μ + 0 and μ + 2 .

μ ≡ x1 mod 13 ≡ x2 mod 17 ≡ x3 mod 19; − 6 ≤ x1 ≤ 6, − 8 ≤ x2 ≤ 8 , − 9 ≤ x3 ≤ 9


x =/ ± 6, ∀x; x =/ + 4, 0, − 2 ∀x

We place these solutions in set D.


i=n
|D| = ∏ (si − 5) = (8)(12)(14) = 1344 . This represents a tensor of dimensions 8x12x14.
i=3

/
We now recognize the following identities:

D = C1 ∩ C2 ∩ C3 .

D = C1 ∩ C2
D = C1 ∩ C3
D = C2 ∩ C3.

C1 = B1 ∩ B2 .
C2 = B1 ∩ B3 .
C3 = B2 ∩ B3 .

D = B1 ∩ B2 ∩ B3

A = B1 ⋃ B2 = B1 ⋃ C3
A = B1 ⋃ B3 = B3 ⋃ C1
A = B2 ⋃ B3 = B2 ⋃ C2

A = B1 ⋃ B2 ⋃ B3

B1 = C1 ⋃ C2
B2 = C1 ⋃ C3
B3 = C2 ⋃ C3

A = C1 ⋃ C2 ⋃ C3

A ⊂ Y.

Since D is a subset of all the other sets, placing it in union with any of them has no effect; likewise
placing any of the sets in union with A has no effect, and placing any set, including A, in union with Y has
no effect, since Y is the universal set in accordance with Chinese Remainder Theorem; meanwhile A is
the universal set of all possible solutions at the start of the algorithmic process.

/
Since some of those who reviewed this work did not find the above identities readily apparent, let
us briefly discuss some of them.

Set A contains all systems of congruences that do not have a remainder of 6 or -6.
Set B1 contains all systems of congruences that do not have a remainder of 6 or -6 or +4.
Set B2 contains all systems of congruences that do not have a remainder of 6 or -6 or 0.
Set B3 contains all systems of congruences that do not have a remainder of 6 or -6 or -2.

Therefore the intersection of B1 and B2 contains all systems of congruences that do not have a
remainder of +6, -6, +4 or 0; yet their intersection includes those systems with a remainder of -2; hence
the intersection of B1 and B2 is C1, which was already defined as containing all systems of congruences
that do not have a remainder of +6, -6, +4 or 0, while retaining those systems with a remainder of -2.

Set D contains all systems of congruences that do not have a remainder of 6 or -6 or 4 or 0 or -2.

Thus if we take the intersection of all the B sets, we result with D.

Also, if we take the intersection of any two, or all three, of the C sets, we also get D.

The union of any two B sets is A, since they complement the missing remainder from either.

The union of any two C sets is a B set, since they complement a missing remainder from either,
such that the union of all C sets is A. The union of a B set with its exclusive C set is A.

Whereas Set A contains all of the initial possible solutions, set D contains all of the absolutely
failed solutions, since not a single integer in the nucleus that wasn’t sieved by 2 or 3 is being sieved by
any of the remaining primes in S. The B and C sets are also failed solutions, since they are missing at
least one of the required remainders.

/
We now subtract three B sets from A, we then add back the C sets, since they are the
intersections of the B sets (each of the three C sets were subtracted twice, thus we must add them back
once to maintain the proper count) and then we subtract the D set, since adding back the C sets also
added back the D set (after exclusion and inclusion are accounted for).

Thus our formula appears as follows: |A| - 3|B| + 3|C| - |D|.

Since we know the size of these sets we get (remember the count is doubled for the additive inverses):

i=n i=n i=n i=n


|A| - 3|B| + 3|C| - |D| = ∏ (si − 2) − 3 ∏ (si − 3) + 3 ∏ (si − 4) − ∏(si − 5)
i=3 i=3 i=3 i=3

Let’s check this formula for {13, 17, 19} and make sure it produces 6.

(11)(15)(17) - 3(10)(14)(16) + 3(9)(13)(15) - (8)(12)(14) = 2805 - 6720 + 5265 - 1344 = 6

If we substitute the products for squares we get a variation of the standard binomial theorem:

2 2 2 2
0 = (x − 2) − 3(x − 3) + 3(x − 3) − (x − 4) , which makes sense since we need more than two belligerent
primes to complete the nucleus (need at least three!)

If we substitute the product for cubes we get an arithmetic difference of 6.

3 3 3 3
6 = (x − 2) − 3(x − 3) + 3(x − 3) − (x − 4) , which again makes sense since any set of three belligerent
primes (and only three) only has 6 unique solutions to form a nucleus of 12.

Once we go beyond cubes the answers vary depending on the value of x , which again makes sense
because having four or more belligerent primes to choose from, when only three are needed, would
generate wildly different results specific to the primes in the set.

Since the identities on page 65 would apply to any rank- (n − 2) tensor, the formula itself if verified
for any number of primes in S.

/
Lemma 1.9.2.2; The size of M14 , S = {2, 3, 17, 19, 23, 29} and |S| = 6 , {2,3} ∈ S
Set Theory Derivation

Let S = {2, 3,17, 19, 23, 29}

∀μ ∈ M6, μ ≡ − 1 mod 2 ≡ − 1 mod 3 ≡ a1 mod 17 ≡ a2 mod 19 ≡ a3 mod ...; a1 , a2 ... =/ ± 7

17, 19, 23 and 29 must be belligerent, such that they sieve (μ − 5) and (μ − 1) and (μ + 1) and (μ + 5)
,{17,19, 23, 29} ∈ B.

μ ≡ 0 mod 6

k μ−7 μ−5 μ−1 μ μ+1 μ+5 μ+7

ti ui ti+1

ξ (k) ∅ 2 bx 2 3 2 bz 2 bw 2 3 2 by 2 ∅

( )
i=n i=n i=n i=n i=n
ϖ (M14) = ∏ (si − 2) − 4 ∏ (si − 3) + 6 ∏ (si − 4) − 4 ∏(si − 5) + ∏(si − 6) , n ≥ 6
i=3 i=3 i=3 i=3 i=3

There are 4! = 24 possible solutions that allow for a length of 14, this is because each solution is
contained inside a cell at the end of a 4-D vector, that must contain a +5, +1, -1 and - 5 in each of its
coordinates, and there are 24 different orderings of +5, +1, -1 and -5.

First let’s make sure this formula has an arithmetic difference of 24 to ensure we’re not wasting
time when n = 4, and then check once more using tesseracts.

(27)(21)(17)(15) -4(26)(20)(16)(14) +6(25)(19)(15)(13) -4(24)(18)(14)(12) +(23)(17)(13)(11)

144,585 - 465,920 + 555,750 - 290,304 + 55913 = 24.

We’ll plug in tesseracts now starting with x = 20

20^4 -4(19)^4 +6(18)^4 - 4(17^4) +16^4 = 160,000 - 521,284 + 629,856 -334,084 + 65,536 = 24

j= n−2 j= n
j n j n
In general: (n − 2)! = ∑ (− 1) (n−j 2)(x − 2 − j ) , or more simply: n! = ∑ (− 1) (nj )(j) .
j=0 j=0

Since we seem to be in good standing with known mathematics, let’s now proceed to show that
counting the size of Nucleic sets of length 14 conforms to proposed formula.

/
Step 0:
We the first take the initial set of all systems of congruences in respect of {17,19,23,29}, this would be the
following:

μ ≡ x1 mod 17 ≡ x2 mod 19 ≡ x3 mod 23 ≡ x4 mod 29; − 8 ≤ x1 ≤ 8, − 9 ≤ x2 ≤ 9 , − 11 ≤ x3 ≤ 1, − 14 ≤ x4 ≤+ 14

We place these solutions in set Y.


i=n
|Y| = ∏ (si ) = (17)(19)(23)(29) = 215441 . This represents a tensor of dimensions 17x19x23x29.
i=3

Step 1,1:
Of these solutions we eliminate all of which contain a +7 or - 7 in any of the remainders.

We place these solutions in set A.


i=n
|A| = ∏ (si − 2) = (15)(17)(21)(25) = 133875 .
i=3

Step 2:
Of these solutions from A we eliminate all of which do not contain a +5 in any of the remainders, and
place them in set B1, then we eliminate all solutions (from A) without a +1 and place them in set B2, then
we eliminate all solutions (from A) without a -1 and place them in B3, and finally we eliminate all solutions
(from A) that do not contain a -5 and place them in set B4.

i=n
|B| = ∏ (si − 3) .
i=3

Step 3:
We now generate the six unique intersections of all four of the B sets (there are six ways to choose a pair
of four numbers, the C sets represent two of the necessary remainders).

C1 = B1 ∩ B2 ; C2 = B1 ∩ B3 ; C3 = B1 ∩ B4 ;
i=n
C4 = B2 ∩ B3 ; C5 = B2 ∩ B4 ; C6 = B3 ∩ B4 ; |C| = ∏ (si − 4) .
i=3

Step 4:
We now generate the four unique intersections of all six of the C sets (there are four ways to choose a
group of 3 numbers from four numbers, the D sets represent the dismissal of three necessary
remainders, and since the C sets dismiss two remainders, each C set only dismisses one unique
remainder).

D1 = C1 ∩ C2 ∩ C3 ; D2 = C1 ∩ C2 ∩ C4
i=n
D3 = C1 ∩ C3 ∩ C4 ; D4 = C2 ∩ C3 ∩ C4 |D| = ∏ (si − 5) .
i=3

/
Step 4:
We now generate the one unique intersection of the D sets, quite simply there is only one way to choose
four numbers from a set of four numbers, and the E set represents the dismissal of all the four needed
remainders, and each D set dismisses three of the four, then each D set only brings one unique
dismissal).

i=n
E = D1 ∩ D2 ∩ D3 ∩ D4 . |E| = ∏ (si − 6) .
i=3

We now subtract four B sets from A, we then add back the six C sets, since they are the
intersections of the B sets (each of the six C sets were subtracted twice, thus we must add them back
once to maintain the proper count) and then we subtract the four D sets, since adding back the C sets
also added back the four unique D sets twice, and then we add back the E set, since subtracting the D
sets subtracted their one unique intersection twice.

Thus our formula appears as follows: |A| - 4|B| + 6|C| - 4|D| +|E|

(∏ )
i=n i=n i=n i=n i=n
|A| - 4|B| + 6|C| - 4|D| +|E| = (si − 2) − 4 ∏ (si − 3) + 6 ∏ (si − 4) − 4 ∏(si − 5) + ∏(si − 6)
i=3 i=3 i=3 i=3 i=3

Again, this would apply to higher rank tensors, as the previous geometrical derivations showed
on pages 29 through 60.

Q.E.D

/
Lemma 1.9.2.3; The size of Mk , S = {2, 3, ...} and |S| = n , {2,3} ∈ S

General Formula (Belonging to the Class B2 Family)

The Class A formulas are those such that {2,3} ∈ S and all other primes in S are
greater than k , where k is length of the Nucleic Set bring counted.

In general, if si > k , ∀i ≥ 3 , and 2, 3 ∈ S, and (n − 2) ≥ β , then:

if k ≡ 0 mod 3 then

|𝚳k| = 2 ( m=β

m=0
( m
i=n
(− 1) (mβ ) ∏ (si − 2 − m)
i=3
)) , where β = ( 3k ) - 1

or, if k ≢ 0 mod 3 then

( )
m=β i=n
m
|𝚳k| = ∑ (− 1) (mβ ) ∏ (si − 2 − m) , where β = Floor[ 3k ]
m=0 i=3

For k ≡ 0 mod 3 , we must always double the count, since each centrum congruent to 1 mod 6
has an additive inverse congruent to 5 mod 6, such that there exists a bijection.

The derivation of the value of Beta is rather simple. A nucleus whose length is equal to k = 6x
always has a total of (⅓ k ) - 1 integers inside the nucleus that were not sieved by either 2 or 3, thus Beta
represents the minimal amount of primes needed to complete such a nucleus

As for those lengths not divisible by 6, we simply take the floor (round down) of ⅓ k , since that
always represents the amount of integers not sieved by either 2 or 3.

Since we know the value of Beta, which is the number of unsieved integers by either 2 or 3, then
we know the degree of the alternating binomial sum, since the process to obtain those counts will follow
the same geometric and set theory patterns as those we have already witnessed on a case by case
basis.

/
1.10 General Counting Formulas
Classes A0, A1, A2, A3, B1, B2, B3, C1, C2, D1

Theorem 1.10.0 Class A0 Formula, The Null Count.


Let Sn be a set of prime numbers, and n be the cardinality of S, such that |S| = n and
∀s ∈ S, s ≥ (n + 1) , then for all k , such that k > (n + 1) , |𝚳k| = 0.

The proof of this statement is fairly simple, let’s take an example.

Let S = {31, 59, 73, 101, 127}, n = 5 , ∀s ∈ S, s ≥ 6 . Using those five prime numbers, can we
generate gap (nucleus) whose length exceeds 6?

Let t be a totative to the product of S, then if any prime in S divides (t + 1) , then it cannot divide
(t + 6) . Call that prime sa . Now we choose another prime from the set, if that primes divides either (t + 1)
or (t + 2) , then it cannot divide (t + 6) , call that prime sb . Now choose another prime from S, if it divides
either (t + 1), (t + 2), (t + 3) , then it cannot divide (t + 6) … we continue this argument until we run out of
primes, none of which can divide will be able to divide (t + 6) . Since there are 5! , or n! , different orders in
which we could have chose these primes, and none of those orders allow us to divide t + 6 , or (t + n + 1) ,
then this set of primes can only sieve a maximum of five, or n ,consecutive integers, such that the length
of the nucleus would be 6, or n + 1 .

Let us formalize this proof:

Let |S| = n and ∀s ∈ S, s ≥ (n + 1) , then if t is a totative to the product of S, and each prime in S
divides one unique integer between t + 1 and t + n , then no prime in S can divide t + n + 1 ; furthermore,
if any prime, sw , in S, is absent (does not divide an integer) from the sequence t + 1 to t + n , then sw
divides (t + z ), n + 1 ≤ z ≤ n + sw , then it could not have divided any integer in the sequence t + 1 to t + n ,
such that there would exist at least one integer in the sequence that was not sieved (and therefore a
totative); likewise if any prime, sw , in S, is redundant (does not uniquely divide an integer) from the
sequence t + 1 to t + n , then sw divides (t + z ), 1 ≤ z ≤ n , as does at least one other prime, sy , such that
there would exist at least one integer in the sequence that was not sieved (and therefore a totative).

Thus the maximal difference between two consecutive of S, under these conditions, is precisely
equal to n + 1 ; therefore, the count of differences between consecutive totatives of S exceeding n + 1 is
zero.
QED

/
Theorem 1.10.1 Class A1 Formula, The Factorial Count.
Let Sn be a set of prime numbers, and n be the cardinality of S, such that |S| = n and
∀s ∈ S, s ≥ (n + 1) , then for all k , such that k = (n + 1) , |𝚳k| = n! .

The proof of this statement is also fairly simple, let’s take an example.

Let S = {31, 59, 73, 101, 127}, n = 5 , ∀s ∈ S, s ≥ 6 . Using those five prime numbers, how many
gaps of length 6 can we create?

We already know from the previous theorem that we need precisely n primes from S to sieve
each integer in a gap of length n + 1 , and that each prime in S sieves each integer in the gap once and
only once. Thus the amount of unique arrangements of the primes in S is also the answer to this
question.

Let us formalize this proof:

Let t1 and t2 be two consecutive totatives of S, such that t2 − t1 = n + 1 .

Then t1 ≡ − 1 mod sj 1 ≡− 2 mod sj 2 ... − n mod sj n ; sj 1 =/ sj 2 =/ ... sj n

According to Chinese Remainder Theorem, each ordering of the prime moduli generates a
unique system of congruences, and since the above form is the only form in which such a gap can occur,
it follows that the total count of gaps of length n + 1 is the total count of the ordering of S, which is n!

QED

/
Theorem 1.10.2 Class A2 Formula, The Permutative Count.
Let Sn be a set of prime numbers, and n be the cardinality of S, such that |S| = n and
∀s ∈ S, s ≥ (n + 1) , then for all k , such that k < (n + 1) , |𝚳k| = c , such that:

( )
m= (k−1) i=n
c= ∑ (− 1)m (km−1) ∏ (si − 2 − m)
m=0 i=1

Based on our previous work, it’s not difficult to see why this formula comes to be as it is.

Let t1 and t2 be two consecutive totatives to the product of S, and let k be the difference
between them such that k = t2 − t1 , then:

t1 ≡ − 1 mod sj 1 ≡− 2 mod sj 2 ... − (k − 1) mod sj n ; sj 1 =/ sj 2 =/ ... sj (k−1)


t1 ≢ 0 mod s or k mod s, ∀s

n
Then there exists (k − 1)! distinct ( k − 1 )-vectors in each of the (k 1) distinct (k − 1) -subspaces

n
of n -space, such that when each of the (k − 1)! distinct ( k − 1 )-vectors in each of the (k 1) distinct

n
(k − 1) -subspaces are orthogonally projected through each of the (n k) distinct complimentary

( n − k) sub-spaces of n -space, that after all inclusions and exclusions are accounted for in n -space
(intersections), using the same set theory approach as used earlier in this chapter (page 72), and given
condition we reject all solution vectors containing a component of either 0 or k (two less indices per axis),
then the count of these distinct solutions, c ,is precisely:

( )
m= (k−1) i=n
c= ∑ (− 1)m (km−1) ∏ (si − 2 − m) .
m=0 i=1

QED

/
Theorem 1.10.3 General Primorial to Sum Formula; Galaxy Sum.

Let Sn be a set of prime numbers, and n be the cardinality of S, such that |S| = n and
∀s ∈ S, s ≥ (n + 1)

i=n
∏ si =
i=1
k= (n+1)

k=1
([k
m=(k−1)

m=0
( m
i=n
(− 1) (km−1) ∏ (si − 2 − m)
i=1
)]) .

We know that sum of all gaps must equal the product of the primes that generates those gaps,
thus if we know the precise count of how many gaps of each specific length there are, then we can add
those counts, each multiplied by their corresponding length, and their sum will be the product of the
primes that generated those gaps.

Let’s test this formula with S = {17,19, 31, 47}, product of equals (17)(19)(31)(47) = 470,611, Euler Phi
Function returns: 397,440

( )
m=(0) i=n
m
|𝚳1| = ∑ (− 1) (km−1) ∏ (si − 2 − m) = (15)(17)(29)(45) = 332,775
m=0 i=1

( )
m=(1) i=n
m
|𝚳2| = ∑ (− 1) (km−1) ∏ (si − 2 − m) = (15)(17)(29)(45) - (14)(16)(28)(44) = 332,775 - 275,968
m=0 i=1
= 332,775 - 275,968 = 56,807

( )
m=(2) i=n
m
|𝚳3| = ∑ (− 1) (km−1) ∏ (si − 2 − m) = (15)(17)(29)(45) - 2(14)(16)(28)(44) +(13)(15)(27)(43) =
m=0 i=1
332,775 - 551,936 +226,395= 7,234

( )
m=(2) i=n
m
|𝚳4| = ∑ (− 1) (km−1) ∏ (si − 2 − m) = (15)(17)(29)(45) - 3(14)(16)(28)(44) +3(13)(15)(27)(43)
m=0 i=1
-(12)(14)(26)(42) = 332,775 - 827,904 +679,185 -183,456= 600

( )
m=(2) i=n
m
|𝚳5| = ∑ (− 1) (km−1) ∏ (si − 2 − m) = (15)(17)(29)(45) - 4(14)(16)(28)(44) +6(13)(15)(27)(43)
m=0 i=1
-4(12)(14)(26)(42)+(11)(13)(25)(41) = 332,775 -1,103,872 +1,358,370 -733,824+146,575= 24

|𝚳1| + |𝚳2| + |𝚳3| + |𝚳4| + |𝚳5| = 397,440 = Φ (470,611)

|𝚳1|+2|𝚳2|+3|𝚳3|+4|𝚳4|+5|𝚳5| = 332,775+(2)56,807+(3)7234+(4)600+(5)24= 470,611

QED

/
Below is a screenshot of the user “Tintarn” on the website, Art of Problem Solving, dissecting the
General Primorial to Sum Formula, in order to reveal its simple Polynomic Identity (RHS means RIght
Hand Side):

https://artofproblemsolving.com/community/c7h1839878_has_this_prime_number_product_to_sum_form
ula_finite_been_discovered

/
Theorem 1.10.4 Euler Phi Function Polynomic Identity; Stellar Count

Let Sn be a set of prime numbers, and n be the cardinality of S, such that |S| = n and
∀s ∈ S, s ≥ (n + 1)

i=n
∏ (si − 1) =
i=1
k= (n+1)

k=1
([ m=(k−1)

m=0
((−
m
1) (km−1)
i=n
∏ (si − 2 − m)
i=1
)]) .

By removing the factor of k from the previous formula ( k represented the length of the gaps
being counted), this sum evaluates to the total count of totatives, which is the Euler Phi Function.

We can also use Tintarn’s results to prove this polynomic identity.

Hopefully we can reveal the true identity of Tintarn so he can be credited in this paper.
QED

/
Definition 1.10.5 Immutable Preset of Length h ; [TS, h ]

Let Sn be a set of prime numbers (2 ∈ S), and n be the cardinality of S, such that |S| = n and let
T be a subset of S, such that |T| = m and m ≤ n , and let k ≡ a1 mod t1 ≡ a2 mod t2 ≡ ... am mod tm ;
finally, let h be integer and let the sequence of consecutive integers from k − h to k + h be known as an
Immutable Preset, since the remainders for each prime modulus in T are fixed and can not be changed.

Let k be named the Preset Midpoint.

A Preset is invoked by the notation [TS, h ], whose inputs tells us that some set, T, from some
superset S, has generated a system of congruences for some integer, k , and that the Preset is the range
of consecutive integers between k − h to k + h .

If 2 ∉ S, then k can be a half-integer, putting half-integer remainders in the systems of


congruences. Since the prime number 2 ultimately does not affect the pattern or count of gaps between
totatives (it only doubles each of their lengths), we often retain 2 in S in order to avoid the mess of
half-integer remainders.

Definition 1.10.6 The Unsieved Counting Function, σ

Given some Preset of length h with the midpoint k , with respect to the sieving Set T, let σ be
the function that counts the number of integers that are not sieved by any prime in T within the range of
(k − h + 1) and (k + h − 1) .

σ [TS, h ] or σ [TS, h, k ] when k must be constant (specified).

Definition 1.10.7 Absolute Belligerence, ⊕ . 2nd most important definition in Volume 1.

Let μ be a centrum of some nucleus of length h , and let ⊕ (μ) = |B|, where B is the
corresponding set of Belligerent Primes.

/
Theorem 1.10.8 Absolute Belligerence Counting Theorem

Given [TS, h ], let S - T = Y, such that |Y| = n − m = v


and ∀y ∈ Y, y > h if 2 ∉ S, OR, y > 2h if 2 ∈ S; then:

1: if v < σ [TS, h ], then there are not enough primes in Y to complete a nucleus of length 2h
whose centrum, μ , must congruent to k mod Ω (T), since ⊕ (μ) > v , and by definition no prime in Y can
sieve more than one integer in such a preset.

2: if v = σ [TS, h ], then there are exactly enough primes in Y to complete a nucleus of length 2h
whose centrum, μ , must congruent to k mod Ω (T), since ⊕ (μ) = v , and in accordance with Chinese
Remainder Theorem there exists v ! systems of congruences that can complete the nucleus in respect to
the preset.

3: if v > σ [TS, h ], then there are more than enough primes in Y to complete a nucleus of length
2h whose centrum, μ , must congruent to k mod Ω (T), since ⊕ (μ) < v , and in accordance with
Chinese Remainder Theorem there exists c systems of congruences that can complete the nucleus in
respect to the preset, such that c equals:

( )
a= (w−1) i=v
c= ∑ (− 1)a (wa−1) ∏ (y i − 2 − m) ; w = σ [TS, h ].
a=0 i=1

Notice that I had to change the letters of variables to avoid confusion.

The proof of these three statements is descended directly from the Null Count, Factorial Count
and Permutative Count theorems (respectively). Each prime in Y may only sieve one integer in the
Preset, and therefore must be belligerent, thus we need to count the number of systems of congruences
that permit the primes in Y to eliminate the unsieved integers in the Preset.

QED

Earlier in this chapter were already using Presets by setting μ ≡ 0 mod 6 or 3 mod 6 or 5 mod 6 ,
when counting gaps of 2, 4 and 6 (respectively) with {2,3} ∈ S. Likewise, the Class A0, A1 and A2
formulas were derived by using a null Preset with a non-zero length ( T = ∅).

Thus, all of these counting formulas are a result of isolating the Belligerent elements of a
particular Preset. By enforcing the lower bound on Y, y > 2h , we ensure that no prime in Y can be
Harmonic (non-belligerent) when sieving the length of the preset.

We shall now proceed to the specialized counting formulas, where the lower bound on Y is not
enforced. For instance, how do we count gaps of 10 or 12 or greater if 5 ∈ S ? What if 7∈ S? Or both 5
and 7?

/
General Formula Class B1
and
Mutable Cyclic Presets

A cyclic Preset is one whose length is the product the primes that generated the Preset.

Definition 1.10.9 Mutable Cyclic Preset; [TS, h = 21 Ω (T)]

Let Sn be a set of prime numbers (2 ∈ S), and n be the cardinality of S, such that |S| = n and let
T be a subset of S, such that |T| = m and m ≤ n , and let:

k ≡ a1 mod t1 ≡ a2 mod t2 ≡ ... am mod tm , ai =/ 0, ∀i ; finally, let:

h = 12 Ω (T) be an integer…

... and let the sequence of consecutive integers from k − h to k + h be known as a Mutable Cyclic Preset.

Let k be named the Mutable Midpoint.

Theorem 1.10.11 σ [TS, h = 21 Ω (T)]

i=m
For any Cyclic Preset, σ [TS, h = 21 Ω (T)] = -1 + Φ ( Ω (T)) = -1+ ∏ (ti − 1)
i=1

Proof:
The count of numbers unsieved by the primes in T between 0 and its product is always equal to
the Euler Phi Function. Since the midpoint of the Mutable Preset cannot be divided by any prime in T,
then neither can its endpoints (terminals), such that its terminals are themselves unsieved integers. Thus,
when we count the number of unsieved integers between the terminals (a total distance equal to the
product of T) we will always get one less than the number yielded by the Euler Phi Function. No matter
how clever we try to arrange the remainders (all non-zero) in the system of congruences for k , Chinese
Remainder Theorem demands that the total count across the Preset is the same.

Thus, unlike generalized presets, the values of the remainders for k are mutable (excluding
zero), since any arrangement of non-zero remainders will produce the same number of unsieved integers.
QED

/
Definition 1.10.12 Class B1 Counting Formula, The Cyclic Preset Count

Given [TS, h = 21 Ω (T), let S - T = Y, such that |Y| = n − m = v = -1 + Φ ( Ω (T))


and ∀y ∈ Y, y > h if 2 ∉ S, OR, y > 2h if 2 ∈ S; then:

1: if v < -1 + Φ ( Ω (T)), then there are not enough primes in Y to complete a nucleus of length
2h = Ω (T) , whose centrum, μ , must congruent to k mod Ω (T), since:

σ [TS, h = 21 Ω (T)] = -1 + Φ ( Ω (T)) > v , and by definition no prime in Y can sieve more than
one integer in such a cyclic mutable preset, and thus the count of viable systems of congruences to
complete this supposed nucleus is zero.

-1 + Φ ( Ω (T)), then there are exactly enough primes in Y to complete a nucleus of


2: if v =
length 2h = Ω (T) , whose centrum, μ , must congruent to k mod Ω (T), since:

σ [TS, h = 21 Ω (T)] = -1 + Φ ( Ω (T)) = v . The count of viable systems of congruences is


precisely equal to v ! .

-1 + Φ ( Ω (T)), then there are more than enough primes in Y to complete a nucleus of
3: if v >
length 2h = Ω (T) , whose centrum, μ , must congruent to k mod Ω (T), since:

σ [TS, h = 21 Ω (T)] = -1 + Φ ( Ω (T)) < v , and in accordance with Chinese Remainder Theorem
there exists c systems of congruences that can complete the nucleus in respect to the preset, such that
c equals:

( )
a= (w−1) i=v
c= ∑ (− 1)a (wa−1) ∏ (y i − 2 − m) ; w = σ [TS, h ] = -1 + Φ ( Ω (T)).
a=0 i=1

/
General Formula Class B2
and
Repeating Mutable Primorial Presets, Cyclic Presets

A repeating cyclic Preset is one whose length is the product the primes that generated the Preset
multiplied by some positive integer constant, g .

Definition 1.10.3 Mutable Cyclic Preset; [TS, h = (g) 21 Ω (T)]

Let Sn be a set of prime numbers (2 ∈ S), and n be the cardinality of S, such that |S| = n and let
T be a subset of S, such that |T| = m and m ≤ n , and let:

k ≡ a1 mod t1 ≡ a2 mod t2 ≡ ... am mod tm , ai =/ 0, ∀i ; finally, let:

h = (g) 12 Ω (T) be an integer…

... and let the sequence of consecutive integers from k − h to k + h be known as a Repeating Mutable
Cyclic Preset.

Let k be named the Mutable Midpoint.

Theorem 1.10.11 σ [TS, h = 21 Ω (T)]

i=m
For any Cyclic Preset, σ [TS, h = 21 Ω (T)] = -1 + ( g ) Φ ( Ω (T)) = -1+ (g) ∏ (ti − 1)
i=1

Proof:
The count of numbers unsieved by the primes in T between 0 and its product is always equal to
the Euler Phi Function. Since the midpoint of the Mutable Preset cannot be divided by any prime in T,
then neither can its endpoints (terminals), such that its terminals are themselves unsieved integers. Thus,
when we count the number of unsieved integers between the terminals (a total distance equal to g
multiplied by the product of T) we will always get one less than the number yielded by the Euler Phi
Function multiplied by g . No matter how clever we try to arrange the remainders (all non-zero) in the
system of congruences for k , Chinese Remainder Theorem demands that the total count across the
Preset is the same.

Thus, unlike generalized presets, the values of the remainders for k are mutable (excluding
zero), since any arrangement of non-zero remainders will produce the same number of unsieved integers.
QED

/
Definition 1.10.12 Class B2 Counting Formula, The Repeating Cyclic Preset Count

Given [TS, h = (g) 21 Ω (T), let S - T = Y, such that |Y| = n − m = v = -1 + (g)Φ ( Ω (T))
and ∀y ∈ Y, y > h if 2 ∉ S, OR, y > 2h if 2 ∈ S; then:

-1 + ( g ) Φ ( Ω (T)), then there are not enough primes in Y to complete a nucleus of


1: if v <
length 2h = (g)Ω (T) , whose centrum, μ , must congruent to k mod Ω (T), since:

σ [TS, h = (g) 21 Ω (T)] = -1 + (g)Φ ( Ω (T)) > v , and by definition no prime in Y can sieve more
than one integer in such a cyclic mutable preset, and thus the count of viable systems of congruences to
complete this supposed nucleus is zero.

-1 + (g)Φ ( Ω (T)), then there are exactly enough primes in Y to complete a nucleus of
2: if v =
length 2h = (g)Ω (T) , whose centrum, μ , must congruent to k mod Ω (T), since:

σ [TS, h = (g) 21 Ω (T)] = -1 + (g)Φ ( Ω (T)) = v . The count of viable systems of congruences is
precisely equal to v ! .

-1 + (g)Φ ( Ω (T)), then there are more than enough primes in Y to complete a
3: if v >
nucleus of length 2h = (g)Ω (T) , whose centrum, μ , must congruent to k mod Ω (T), since:

σ [TS, h = (g) 21 Ω (T)] = -1 + (g)Φ ( Ω (T)) < v , and in accordance with Chinese Remainder
Theorem there exists c systems of congruences that can complete the nucleus in respect to the preset,
such that c equals:

( )
a= (w−1) i=v
c= ∑ (− 1)a (wa−1) ∏ (y i − 2 − m) ; w = σ [TS, h ] = -1 + (g)Φ ( Ω (T))
a=0 i=1

QED

Although this final formula, Class B2, may seem excessive, realize that all of the formulas were
derived earlier in this chapter (on a case by case basis) were Class B2 formulas, such that {2,3} = T.
Class B2 formulas will dominate Volume 2 of this work, since Volume II concerns the Goldbach
Conjecture.

/
Lemma 1.10.13 The Senary Boolean Theorem

Let T = {2, 3} and let z be a positive integer. When T = {2, 3} is the Preset set, it it shall be
known as the Senary Preset.

Given [TS, h = z + (g) 21 Ω (T)], and its Mutable Cyclic Midpoint, k , then:

σ [TS, h ] = 2g − 1 if z ≡ 0 mod 6 ; therefore if:


g ≡ 1 mod 2 , then k ≡ (2 or 4) mod 6 , since neither 2 nor 3 may divide the terminals (endpoints) of
the Preset; however if:
g ≡ 0 mod 2 , then k ≡ (1 or 5) mod 6 , since neither 2 nor 3 may divide the terminals (endpoints) of
the Preset.

σ [TS, h ] = 2g if z ≢ 0 mod 6 ; therefore k ≡ (0 or 3) mod 6 , since


neither 2 nor 3 may divide the terminals (endpoints) of the Preset.

Proof:

If z ≡ 0 mod 6 , then σ [TS, h ] = 2g − 1 in accordance with Theorem 1.10.11.

If k ≢ 0 mod 6 , then σ [TS, h ] = 2g since σ [TS, g 12 Ω (T)] < σ [TS, h ] < σ [TS, (g + 1) 12 Ω (T)] , such
that:

2g − 1 < σ [TS, h ] < 2g + 1 , therefore 2g is the only integer that fulfills that inequality;
furthermore, since the midpoint, k , is always congruent to 0 mod 6 or 3 mod 6 (under these conditions)
then this partial Senary Preset will have exactly 2g unsieved integers.

Corollary 1.10.14 The Senary Preset Additive Inverse

If z ≡ 0 mod 6 , then the Senary Preset is a Repeating Mutable Cyclic Preset, such that there
exists a Preset whose congruences in respect to 2 and 3 are additive inverses modulo 6. Therefore the
count of all systems of congruences which complete the nucleus of this Preset must also equal the count
of those which complete the additive inverse. Thus the total count must be doubled since there exists a
bijection between them.

QED

The reason “Boolean” is in the title of definition is because when compiling these counting
formulas to computer code, a simple boolean switch is used on truth of z ≡ 0 mod 3 to determine whether
or not the preset is complete or partial. This binary nature of 3 in stellar constellations allows us to
effectively ignore it, much as we ignore the prime number 2 which only scales nucleic lengths.

/
Definition 1.10.13 Class C1 Counting Formula, Complete or Partial Senary Cyclic
Presets

Let T = {2, 3}, |T| = m = 2

Given [TS, h = z + (g) 21 Ω (T), let S - T = Y, such that:

|Y| = n − m = v ≥-1 + (g)Φ ( Ω (T)), if z ≡ 0 mod 6


|Y| = n − m = v ≥ (g)Φ ( Ω (T)), if z ≢ 0 mod 6

and ∀y ∈ Y, y > 2h if 2 ∈ S; then:

or, if h ≡ 0 mod 3 then

|𝚳h| = 2 (m=β

m=0
( m
i=v
(− 1) (mβ ) ∏ (y i − 2 − m)
i=1
)) , where β = -1 + 2 g

or, if k ≢ 0 mod 3 then

( )
m=β i=v
m
h
|𝚳 | = ∑ (− 1) (mβ ) ∏ (y i − 2 − m) , where β = 2g
m=0 i=1

QED

Well, this certainly looks familiar! As you can see we have formally derived the Formulas for
counting gaps with {2,3} in S. In theory, we could have skipped most of Chapter 1 and started with the
Class A0 formula; however, doing so would have not provided us with the notations and definitions that
are required to further our advance towards the Andrica Conjecture, nor would we have truly grasped the
meaning of the polynomic formulas (in the organic, pictorial or physical sense).

By manually deriving the counting formulas for gaps of 2, 4, 6, 10, 12 and 14 earlier in this
chapter (on a case by case basis), the reader will be far more prepared to intuitively grasp the future
concepts in this volume; furthermore, the counting formulas for the Trigesimal Presets can ONLY by
derived manually on a case by case basis!

Had the derivation of the counting formulas for the Trigesimal Presets been the reader’s first foray
into such a hands-on approach, I believe the reader would have been “shell-shocked.”

/
Radix 30, Trigesimal Presets

A trigesimal preset is one that corresponds to a cycle of 30 integers, where 30 is the product of 2,
3 and 5. Each Preset of some length, h , has three, four or six or eight corresponding presets each
belonging to some radix of 30. For instance, the centrum for gaps of 12 have six congruences modulo 30
that permit a length of 12: {5, 7, 13, 17, 13, 25}, note that each radix has a sister radix which is its
additive inverse mod 30.

Over decade ago, in 2007, when I was a young teenager, I began my journey towards solving the
Andrica Conjecture. My only formal experience in mathematics at the time was Logic, since I learned at
an early age how to program code for video games such as Warcraft III (released in 2002).

At that time, I had no education (or awareness) of modular arithmetic or Chinese Remainder
Theorem, nor any knowledge of the Andrica Conjecture itself; yet, in my programming experiments I
deduced the maximal limit between two consecutive totatives (totatives and composites were represented
as 1’s and 0’s, like Principal Dirichlet Characters) and the way in which I did so seemed so logical that it
was beyond question (the result was not even the primary object of inquiry), it did not even occur to me
that it was potentially an unsolved question in mathematics, in fact it was not until late 2011 that I was
aware of what I had done four years prior in 2007.

In my early deduction process, I had to rediscover several things on my own due to my own
ignorance (and notation systems I used were alien and as of this day discontinued):

1: Modular Arithmetic (via radix).


2: Chinese Remainder Theorem (proved via combination lock thought experiment).
3: Sieve of Eratosthenes (a stronger result was also proven, that there exists four integers such
that ξ (k) = si in any sequence of 2si 2 consecutive integers or consecutive multiples of si , this today is
known as the Quartic Bracket Theorem, and it covered in Chapter 3, with the Strong Bertrand Theorem).

To circumvent my ignorance, I used the tools I had available from programming experience:

1: Radix
2: Variable (including arrays, matrices and tensors).
3: Comparative and Boolean Logic

The radix served four main purposes:


A: It was used a substitute for modular arithmetic (unbeknownst to me at the time).
B: It greatly reduced the computational load required to verify the Strong Andrica Conjecture for sets of
eight to fourteen primes.
C: The radix system I employed could be handwritten and allowed me to quickly work with any set of two
to five primes, and also allowed me to hastily add, subtract and multiply large numbers by hand.
D: Using similar radices (two radices sharing at least one component) I was able to map injections and
surjections between gaps generated by different sets.

For highly multiplicative numbers, such as 6, 30, 210 and 2310, the radix system was the best
(and for programming purposes for things related to this Volume, the radix remains the best).

/
Here’s an example of my old handwritten radix of 2310 (don’t worry, we will not be using this notation in the volume). The
top number was mod 3, the left number was mod 5, the right number was mod 7 and the bottom number was mod 11. The center
was mod 2, expressing the binary relationship of even or odd. The radix was compiled into a 35x33 matrix on graph paper, with a
simple boolean conversion function to deal with with the number’s parity (even or odd). It was from these handwritten “radices within
radices” that I developed the Super-Radix system, which, from my experience, is the most rapid system for generating full
multiplication tables (two digits) of a large multiplicative radix through induction (reiteration) by starting with the largest prime divisor
to the least.

/
Here was a multiplication table (residues only) that accompanied it (I usually worked without 2
dividing the radix, 2 ∉ S , so this particular chart was most often used for the 1155 radix, 3*5*7*11 = 1155,
instead of 2310 radix.

/
For fast conversions (of residues) from Radix 1155 to Decimal, I used this chart: First the four components of the radix, mod 5, mod
7, mod 3 and mod 11, were matched; second, the cell was then identified as ( x1 , y 1 ) in decimal; third, we mapped the cell to (
x2 , y 2 ) = (x1 − y 1 , 0) , and then to determine the decimal value of the original cell, ( x1 , y 1 ) , the following function was employed:

1155 − (33) 12 x2 + y 1 when x2 ≡ 0 mod 2


561 − (33) 12 (x2 + 1) + y 1 when x2 ≡ 1 mod 2

Needless to say, the number of manual or machine computations are drastically reduced by these tools and methods. In
general one can apply any number of primes to any dimension of a tensor and then determine the general linear equation (after
some series of orthogonal projections) for base conversions of residues. As I said, we won’t be using these in this volume, but they
are supplementary material for anyone who wishes to write computer programs to use the ideas in this chapter or volume.

/
Definition 1.10.14a Radix Set

Let |S| = n ; |T| = m ; |Y| = n − m = v ;


and ∀y ∈ Y, y ≥ h if 2 ∉ S, OR, y > 2h if 2 ∈ S;
and ∀t ∈ T, t < h if 2 ∉ S, OR, t > 2h if 2 ∈ S; then:

Given [TS, h ] and its midpoint, k , or [TS, 2h ] , if 2 ∈ S, let X be the set of integers:

j=m
1
x ≢ ± 2
(h) mod tj , ∀j, 0 ≤ j ≤ m , such that α = |X| = ∏ (ti − 2) .
j=1

j=m
1
x ≢ ± 2
(2h) mod tj , ∀j, 0 ≤ j ≤ m , such that α = |X| = ∏ (ti − 2) , if 2 ∈ S .
j=2

X is said to be the Radix Set, and is ordered from least to greatest.


Any element of a Radix Set, x , is named a Radix Component, or simply a component.

Definition 1.10.14b Radix Isolation Array, ⊞ , counting unsieved integers in each Preset in a Radix Set.

Let be ⊞ the integer sequence: ⊞i = σ [TS, h , xi ] , ∀i, 1 ≤ i ≤ α


Let be ⊞ the integer sequence: ⊞i = σ [TS, 2h , xi ] , ∀i, 1 ≤ i ≤ α , if 2 ∈ S .

Definition 1.10.14c Frequency Function, η d (Integer Sequence)

Let be η d be the frequency function, that counts the frequency of any possible element, d , in
some integer sequence.

Definition 1.10.14c Unsieved Frequency Array, ⌘

Let be ⌘ the integer sequence: ⌘ = η d ( ⊞ ), 0 ≤ d ≤ h − 1

Let be ⌘ the integer sequence: ⌘ = η d ( ⊞ ), 0 ≤ d ≤ 12 (2)h − 1 = h − 1 , if 2 ∈ S .

Definition 1.10.14d Numbered Unsieved Frequency Matrix, ⌹

Let ⌹ be an integer matrix of dimensions h by 2, such that: ⌹( f, 1 ) = (f − 1) and ⌹( f, 2 ) = ⌘(f −1)

/
Lemma 1.10.15 Class C1 Counting Formula, Partial Trigesimal Cyclic Presets
Radix 30
Gaps of 2,4,6, 8, Senary Identity

For gaps (nucleic lengths) of 2,4,6 and 8, the count remains the same as the initial formulas we
derived earlier in this chapter. It is not until we arrive at gaps of 32,34,36 and 38, that we will be able to
examine the peculiar effects of the prime number 5 on the Radix Sets of 2,4,6 and 8.

Thus the Trigesimal Counting Formulas for gaps of 2,4,6 and 8 are identical to the Senary
Counting Formulas, since the prime number 5 > (1,2,3,4) and 2(5) = 10 > (2,4,6 and 8), which forces 5 to
be belligerent in such gaps, if and when it sieves an integer within the nucleus; however, using the prior
set of definitions, we will prove this formally.

Gaps of 2,4,6,8:

Let |S| = n ; |T| = m ; |Y| = n − m = v ;


Let T = {2,3,5}

If 2 ∉ S, then let 1 ≤ h ≤ 4, Δh = 1 , ∀y ∈ Y, y ≥ h , and ∀t ∈ T, t < h


If 2 ∈ S, then let 2 ≤ h ≤ 8, Δh = 2 , ∀y ∈ Y, y ≥ h , and ∀t ∈ T, t < h

|T| = {2,3,5} = { t1 , t2 , t3 } , then ∀h, t3 > 2h , violating the the final condition, such that 5 would have
to be removed from the Preset set, and thus T = {2,3}, which is a Senary Preset.
QED

The increment notation, Δ , used above shall now be employed for the remainder of this volume,
since it carries well into Volume 2 when the Dirichlet Filter Function is introduced for arithmetic
sequences; also, the increment notation often allows for analytical formulas for those that wish to take this
work in a different direction; finally, problems involving arithmetic sequences shall confront us in this very
volume on our quest to solve the Andrica Conjecture, long before we reach the Dirichlet Filter Function in
Volume 2, in the chapters that strengthen the Goldbach Conjecture.

Also, if we really wanted to, we could have fully applied the Radix definitions and examined all
three (or six) presets of lengths of 2,4,6 and 8, and, after simplification, ended up with the Senary
Formulas. Let us now proceed to gaps of 12, 14, 16 and 18 (gaps of 10 and 20 will be handled after).

/
Definition 1.10.16 Class C2 Counting Formula, Partial Trigesimal Cyclic Presets
Radix 30
Gaps of 12,14,16, 18, 22, 24, 26, 28

Gaps of 12 (with 5 in the sieving set) are our first foray into hybridized Polynomic Identities.

Let |S| = n ; |T| = m ; S ⊃ T; Y = S - T; |Y| = n − m = v ;


Let T = {2,3,5}, m = 3

If 2 ∈ S, then let h = 12 , ∀y ∈ Y, y ≥ 12 , and ∀t ∈ T, t < 12

Given [TS, 12 ] and its midpoint, k , let X be the set of integers:

j=m
1
x ≢ ± 2
(h) mod tj , ∀j, 0 ≤ j ≤ m , such that α = |X| = ∏ (ti − 2) , since 2 ∈ S .
j=2

Since our choices of remainders for 2 equals {-1), and 3 equals {+1,-1) and 5 equals {+2, 0, 2}, then (if
you’re programming this, you will need a separate tensor to store the viable remainders to generate X, it’s
easiest to use the full tensor of respective prime dimensions, and then exclude the terminals at ± λ )

X= {5, 7, 13, 17, 23, 25}, now let be ⊞ the integer sequence: ⊞i = σ [TS, 12, xi ] , ∀i, 1 ≤ i ≤ α

⊞ = {2, 3, 2, 2, 3, 2}, now let ⌘ the integer sequence: ⌘ = η d ( ⊞ ), 0 ≤ d ≤ 12 − 1 = 11

⌘ = {0, 0, 4, 2, 0, 0, 0, 0, 0, 0, 0, 0}

Now let ⌹ be integer matrix, of dimensions h by 2, such that: ⌹( f, 1 ) = (f − 1) and ⌹( f, 2 ) = ⌘(f −1)

⌹ ( h by 2)
0 1 2 3 4 5 6 7 8 9 10 11

0 0 4 2 0 0 0 0 0 0 0 0

Thus there are four presets with two unsieved integers, and two presets with three unsieved
integers, since we are assuming the condition that ∀y ∈ Y, y ≥ 12 , then we can apply the general class
A0, A1 and A2 formulas depending on the size of Y (we need at least two primes in Y to complete the
four presets with two unsieved integers; and at least three primes in Y to complete the two presets with
three unsieved integers).

/
Thus in order to count gaps of 12, c , we will have to take the sum of four polynomials of degree
2, and add it to the sum of two polynomials of degree 3. If there are only two primes in Y, then we take
only the sum of the four polynomials of degree 2, since the polynomials of degree 3 will return the Null
Count. If there is only one prime in Y (or no primes), we obviously have an overall null count.

Thus, for |Y| ≤ 1, c = 0 (special case)

And, for |Y| = 2, |Y| = v (special case), c = 8

( )
m= 2 i=v=2
m
c=4 ∑ (− 1) (m2 ) ∏ (y i − 2 − m)
m=0 i=1

( )
i=2 i=2 i=2
c = 4 ∏ (y i − 2) − 2 ∏ (y i − 3) + ∏ (y i − 4) = 4(2) = 8
i=1 i=1 i=1

And, for |Y| ≥ 3 , |Y| = v (general case)

( ) ( )
m= 2 i=v m= 3 i=v
m m
c=4 ∑ (− 1) (m2 ) ∏ (y i − 2 − m) + 2 ∑ (− 1) (m3 ) ∏ (y i − 2 − m)
m=0 i=1 m=0 i=1

(∏ ) (∏ )
i=v i=v i=v i=v i=v i=v i=v
c=4 (y i − 2) − 2 ∏ (y i − 3) + ∏(y i − 4) +2 (y i − 2) − 3 ∏ (y i − 3) + 3 ∏ (y i − 4) − 1 ∏ (y i − 5)
i=1 i=1 i=1 i=1 i=1 i=1 i=1

Final Simplification:

( )
i=v i=v i=v i=v
c = 2 + 1 ∏ (y i − 2) − 7 ∏ (y i − 3) + 5 ∏ (y i − 4) − 1 ∏ (y i − 5)
i=1 i=1 i=1 i=1

The general formula to obtain the specialized polynomial sum of any preset is the following:

m= ⌹(

( )
f =h f, 1 ) i=v
c = ∑ [(⌹( f , 2 )) ∑ (− 1)m (⌹(mf , 1 ) ) ∏ (y i − 2 − m) ]
f =1 m=0 i=1

The general formula isolates each frequency of each polynomial degree. Now that we’ve seen it in action,
let’s state this definition formally (next page), before proceeding to the 14,16 and 18 formulas.

/
General Class C2 Counting Formula, Partial Cyclic Presets:
Gaps of h .

m= ⌹(

( )
f =h f, 1 ) i=v
|𝚳 | = c = ∑ [(⌹(
h
f , 2 )) ∑ (− 1)m (⌹(mf , 1 ) ) ∏ (y i − 2 − m) ]
f =1 m=0 i=1

Such that,

Let |S| = n ; |T| = m ; S ⊃ T; Y = S - T; |Y| = n − m = v ;

∀y ∈ Y, y ≥ h , and ∀t ∈ T, t < h

Given [TS, h ] and its midpoint, k , let X be the set of integers:


j=m
1
x ≢ ± 2
(h) mod tj , ∀j, 0 ≤ j ≤ m , such that α = |X| = ∏ (ti − 2) , if 2 ∉ S .
j=1
j=m
1
x ≢ ± 2
(h) mod tj , ∀j, 0 ≤ j ≤ m , such that α = |X| = ∏ (ti − 2) , if 2 ∈ S .
j=2

Let be ⊞ the integer sequence: ⊞i = σ [TS, 12, xi ] , ∀i, 1 ≤ i ≤ α

Let ⌘ the integer sequence: ⌘ = η d ( ⊞ ), 0 ≤ d ≤ 12 − 1 = 11

Let ⌹ be integer matrix, of dimensions h by 2, such that: ⌹( f, 1 ) = (f − 1) and ⌹( f, 2 ) = ⌘(f −1) .

Although the majority of the frequencies are zero in our example for gaps of 12 in trigesimal
presets, for large and unwieldy presets, we must implement the entire array and corresponding matrix for
computers to precise calculate all the permutations, while also including the case of no sieved integers (a
count of h − 1 unsieved integers), for when the Preset set is empty (this happens often for massive
calculations that take the sum of all gaps for each instance of v > h − 1 ).

The C2 formula will most often be executed by programmers to test my claims on numerical data
for Fractured Prime Sets later in this volume (definition of fractured prime sets repeated below), since the
limit of the maximal gap (for each count of absolute belligerence) is greater than many of the primes in
the P set. Also, we could do this for gaps of 10 and 20 in the trigesimal sets without any alteration, but
you will soon see the reason (involving them having four presets) why I’d rather perform the gaps of 10
and 20 by hand, before we permit them into our Class C2 formula.

Fractured Set of Primes:


Given the set Pn-k , then:
Let si ∈ Sc, such that ∀si ∈ Sc, si > pn
denote the union of the sets Pn-k ⋃ Sc.
c
Let W n−k

By definition Pn-k ⋂ Sc = ∅ , therefore |W


c
n−k | = (n − k + c)

/
Radix 30
Gaps of (12,14,16, 18, 22, 24, 26, 28) +60b, b ≥ 0

For gaps of 14, 16 and 18, I’m simply going to provide the Radix Sets and the frequencies of their
unsieved integers, and then the specialized formula that comes from those. Bear in mind that if there’s not
enough primes to complete their nuclei, that the formula is reduced from that which I shall present.

Let |S| = n ; |T| = m ; S ⊃ T; Y = S - T; |Y| = n − m = v ;


Let T = {2,3,5}, m = 3

If 2 ∈ S, then let 14 ≤ h = 28 , Δh = 2, h =/ 10 , and ∀y ∈ Y, y ≥ h , and ∀t ∈ T, t < h

Given [TS, h ] and its midpoint, k , let X be the set of integers:

j=m
1
x ≢ ± 2
(h) mod tj , ∀j, 0 ≤ j ≤ m , such that α = |X| = ∏ (ti − 2) , since 2 ∈ S .
j=2

For h = 14
X = {24, 0=30, 6} (depending on your radix you may use 0 or 30 at your discretion, I prefer 0 since it’s a single digit).
⊞ = {3, 2, 3}

⌹ ( h by 2)
0 ... 2 3 4 5 6 ...

0 ... 1 2 0 0 0 ...

( )
i=v i=v i=v i=v
|𝚳14| = c = + 3 ∏ (y i − 2) − 8 ∏ (y i − 3) + 7 ∏ (y i − 4) − 2 ∏ (y i − 5)
i=1 i=1 i=1 i=1

For h = 16
X = {9, 15, 21}
⊞ = {3, 4, 3}

⌹ ( h by 2)
0 ... 2 3 4 5 6 ...

0 ... 0 2 1 0 0 ...

( )
i=v i=v i=v i=v i=v
|𝚳16| = c = + 3 ∏ (y i − 2) − 10 ∏ (y i − 3) + 12 ∏ (y i − 4) − 6 ∏ (y i − 5) + ∏ (y i − 6)
i=1 i=1 i=1 i=1 i=1

/
For h = 18
X = {7, 5, 13, 17,25, 23}
⊞ = {4, 3, 4, 4, 3, 4}

⌹ ( h by 2)
0 ... 2 3 4 5 6 ...

0 ... 0 2 4 0 0 ...

( )
i=v i=v i=v i=v i=v
|𝚳18| = c = 2 + 3 ∏ (y i − 2) − 11 ∏ (y i − 3) + 15 ∏ (y i − 4) − 9 ∏ (y i − 5) + 2 ∏ (y i − 6)
i=1 i=1 i=1 i=1 i=1

For h = 22
X = {18, 0, 12}
⊞ = {5, 4, 5}

⌹ ( h by 2)
0 ... 2 3 4 5 6 ...

0 ... 0 0 1 2 0 ...

( )
i=v i=v i=v i=v i=v i=v
|𝚳22| = c = + 3 ∏ (yi − 2) − 14 ∏ (yi − 3) + 26 ∏ (yi − 4) − 24 ∏ (yi − 5) + 11 ∏ (yi − 6) − 2 ∏ (yi − 7)
i=1 i=1 i=1 i=1 i=1 i=1

For h = 24
X = {1, 19, 25, 5, 11, 29}
⊞ = {5, 6, 5, 5, 6, 5 }

⌹ ( h by 2)
0 ... 2 3 4 5 6 ...

0 ... 0 0 0 4 2 ...

|𝚳24| = c =

( )
i=v i=v i=v i=v i=v i=v i=v
2 + 3 ∏ (y i − 2) − 16 ∏ (y i − 3) + 35 ∏ (y i − 4) − 40 ∏ (y i − 5) + 25 ∏ (y i − 6) − 8 ∏ (y i − 7) + 1 ∏ (y i − 8)
i=1 i=1 i=1 i=1 i=1 i=1 i=1

/
For h = 26
X = {24, 0, 6}
⊞ = {6, 6, 6 }

⌹ ( h by 2)
0 ... 2 3 4 5 6 ...

0 ... 0 0 0 0 3 ...

|𝚳26| = c =

( )
i=v i=v i=v i=v i=v i=v i=v
3 + 3 ∏ (y i − 2) − 6 ∏ (y i − 3) + 15 ∏ (y i − 4) − 20 ∏ (y i − 5) + 15 ∏ (y i − 6) − 6 ∏ (y i − 7) + 1 ∏ (y i − 8)
i=1 i=1 i=1 i=1 i=1 i=1 i=1

For h = 28
X = {3, 15, 27}
⊞ = {6, 6, 6 }

⌹ ( h by 2)
0 ... 2 3 4 5 6 ...

0 ... 0 0 0 0 3 ...

|𝚳28| = c =

( )
i=v i=v i=v i=v i=v i=v i=v
3 + 3 ∏ (y i − 2) − 6 ∏ (y i − 3) + 15 ∏ (y i − 4) − 20 ∏ (y i − 5) + 15 ∏ (y i − 6) − 6 ∏ (y i − 7) + 1 ∏ (y i − 8)
i=1 i=1 i=1 i=1 i=1 i=1 i=1

/
Now that we have our data for the gaps of 12, 14, 16, 18, 22, 24, 26 and 28, we can add multiples of 60
to them and produce an even more generalized formula for Partial Trigesimal Sets. Before I insert the
formula, let’s take a look at this picture for a gap of 88 (88 = 28 + 60):

Because any cycle of 30 integers, modulo 30, will contain 7 unsieved integers, then if we add blocks of 30
(taking into account that the terminals of adjacent blocks will merge into a single integer) then we also
add 7 unsieved integers per block. Since we must retain the position of the midpoint, we must add a block
to either side (a total of 60 integers) and thus a pattern emerges modulo 60, by adding 14 to each degree.

For r = 18
For h = r + 60b
X = {7, 5, 13, 17,25, 23}

Foundation( r, 30) ⊞ = {4, 3, 4, 4, 3, 4}; Extrapolation = 14


Extrapolation( r + 60b, 30) ⊞ = {4+14b, 3+14b, 4+14b, 4+14b, 3+14b, 4+14b}

⌹ ( h by 2)
0 ... 2 3+14b 4+14b 5 6 ...

0 ... 0 2 4 0 0 ...

/
We can generalize this result for any preset (the initial case is called the foundation, and the
reiterations of the cyclic sequence are the extrapolations, the frequencies remain the same, even though
the degrees are increased by an equal increment).

i=m
For some Preset Foundation, r , such that r < 2 ∏ ti = Ω (T)
i=1
The for some h = r + 2Ω(T )b

Foundation( r, Ω(T )) ⊞i ] = g i

[Foundation( r, Ω(T )) ⊞i ] + (Φ[Ω(T )] − 1)b = [Extrapolation( r, 30) ⊞i ] = g i

⌹ ( h by 2)
0+ ... g i−1 gi g i+1 ... ... g r−1
(Φ[Ω(T )] − 1)b

same ... same same same same same ...

This generalization would consider every foundation from 2 to 58, including 10, 20, 30 40, 50 and
60. The reason one must execute the foundation to a number that is double the radix is because there is
no predictable pattern (generally) in which the positive and negative halves of the partial will combine
together to produce their respective frequencies, thus, until each half has a length of 30 (or more), there
will be no determinable repetition (although we shall use backwards induction later in this volume to
show that the isolations and frequencies of every possible Preset for a particular maximal length, in
respect to its internal belligerence, can be determined for Fractured Prime Sets under controlled
conditions, which will be necessary in the final steps of proving the Andrica Conjecture).

What makes the overall proof of the Andrica Conjecture succeed in this Volume is that we are
only working with those things which are determinable, finite and countable, and as far as I’m
concerned there is no other way to prove it, which is why there has been a century long stagnation in the
field of elementary number theory in respect to prime numbers. If we ever hope to see a proof of the
Riemann Hypothesis in our lifetimes...then this is how we must approach it, by counting!

Since we other things to discuss in this chapter, I’m only going to show the specialized results for
gaps of 10 and 20, before we onto Rank Euler Polynomial Identities.

Anyone who has continued and invested interest in the subject of counting gaps has already
been armed with sufficient information to write a computer program that counts all the permutations of any
Preset of any length they desire, thus if the reader desires the formulas for gaps of 32 to 58, they can do
it themselves. The Trigesimals don’t even require a computer or any hard calculation for a human being,
just a pencil and some graphing paper and the willpower to sieve and then count unsieved integers on a
giant number line.

/
Gaps of 10 and other Augmented Senary Presets

Gaps of 10 have a unique form, and in general, for any prime, s3 , gaps of 2(s3 ) also take on the
same unique formulation. These are known as Augmented Senary Presets.

( )
i=v i=v i=v
4
|𝚳10| = c = 3 ∏ (y i − 2) − ( 32 )2 ∏ (y i − 3) + 2
1 ∏ (y i − 4)
i=1 i=1 i=1

The general formula for some Augmented Senary Preset, {2,3, s3 }, of length 2(s3 ) is:

Definition 1.10.17 Class B3 Counting Formula, Augmented Senary Preset:

In general, if si > k , ∀i ≥ 3 , and 2, 3 ∈ S, and (n − 2) ≥ β , then:

Since 2(s3 ) ≢ 0 mod 3 , then

( )
m=β i=n
(s3 −1−m) m 2(s3 )
|𝚳 2(s3) | = ∑ (s3 −2−m) (− 1) (mβ ) ∏ (si − 2 − m) , where β = Floor[ 3
]
m=0 i=3

How this formula comes to be is rather simple. Since s3 divides the length, only one choice of
possible remainders is eliminated: 0 mod s3 . Thus our initial set of possible solutions now includes one
additional remainder for the modulus s3 , as a consequence, each diminishing set of solutions also
contains that additional remainder for the modulus s3 .

Geometrically, each distinct solution vector (the solution vector containing β components for the
β number of distinct unsieved integers) not containing a modulus of s3 , will be orthogonally projected
through the complimentary (n − β ) -space, and those complimentary spaces will contain the dimension of
s3 (subtracted by the proper diminishing number for each Dismissal Set, revisit page 72 for any
clarification you may require); likewise, s3 , due to its own size, is permitted to divide any of the unsieved
integers in the Senary Preset and will be belligerent, and therefore any of the β unsieved integers will
correspond to a unique remainder for the modulus s3 ; thus, the solution vectors containing a modulus of
s3 will project normally unto the other complementary subspaces, and thus the formula requires no further
adjustment.

Also, we could have arrived at this answer with the C2 formula, but the above dissertation is the
organic and natural way in which I had derived it years ago, and is much cleaner in appearance than a C2
formula.

/
Gaps of 20 and 30

A gap of 20 has four elements in its radix set.

For h = 20
X = {3, 9, 21, 27}
⊞ = {4, 5, 5, 4}

⌹ ( h by 2)
0 ... 2 3 4 5 6 ...

0 ... 0 0 2 2 0 ...

( )
i=v i=v i=v i=v i=v i=v
|𝚳22| = c = + 4 ∏ (yi − 2) − 18 ∏ (yi − 3) + 32 ∏ (yi − 4) − 28 ∏ (yi − 5) + 12 ∏ (yi − 6) − 2 ∏ (yi − 7)
i=1 i=1 i=1 i=1 i=1 i=1

A gap of 30 has eight elements in its radix set, which are all the totatives of the Cyclic 30 set.

For h = 30
X = {1, 7, 11, 13, 17, 19, 23, 29}
⊞ = {7, 7, 7, 7, 7, 7, 7, 7} Quite simply, all the presets have Φ(30) − 1 unsieved integers, since
each preset conforms to a Class B1 Formula.

⌹ ( h by 2)
0 ... 2 3 7 8 9 ...

0 ... 0 0 8 0 0 ...

(∏ )
i=v i=v i=v i=v i=v i=v i=v i=v
|𝚳30| = 8 (y i − 2) − 7 ∏ (y i − 3) + 21 ∏ (y i − 4) − 35 ∏ (y i − 5) + 35 ∏ (y i − 6) − 21 ∏ (y i − 7) + 7 ∏ (y i − 8) − ∏ (y i − 9) .
i=1 i=1 i=1 i=1 i=1 i=1 i=1 i=1

From this gap of 30 we get our final generalized formula:

Class D1 Formula
Gaps of Ω (T)

|𝚳 Ω(T ) | = Φ ( Ω (T)) ( a= (w−1)



a=0
( a
i=v
(− 1) (wa−1) ∏ (y i − 2 − m)
i=1
)) ; w = σ [TS, h ] = -1 + Φ ( Ω (T)).

See Appendix B for a collection of all the Class Formulas and the Senary and Trigesimals.
.

/
Section II
Maps

Section II Abstract

In Section 2 we will develop a system for mapping Nucleic Sets of the same length that are
generated by a set of distinct primes (Primordial Set) unto Nucleic Sets of same the length, or greater,
that are generated by a different set of distinct primes.

There are five distinct exclusive relations that exist between two distinct sets of primes, from
which follow several simple logical statements.

It is not important that we discover how to count the exact number of surjections (maps) between
Nucleic Sets in order to solve the Andrica Conjecture. The only thing we must recognize is whether or not
there exists a surjection, bijection or injection between Nucleic Sets, and how to discern which of them is
true.
2.1 Preliminary Definitions and Lemmas

Definition 2.1.1, Maximal Gap, Maximum Nucleic Length, Maximum Span

Let δ [S] equal the maximum difference between two consecutive totatives to Ω [S].
This will most often be referred to as the Maximum Span of S.

Lemma 2.1.2, The Five Possible Exclusive Relations

Let δ [S] equal the maximum difference between two consecutive totatives to Ω [S]

Let S and T be two distinct sets of primes such that S =/ T, then:

1: |S| = |T| and δ [S] = δ [T], or,


2: |S| = |T| and δ [S] > δ [T], or,
3: |S| > |T| and δ [S] = δ [T], or,
4: |S| > |T| and δ [S] > δ [T], or,
5: |S| > |T| and δ [S] < δ [T].

In this volume the logical equivalents are more often invoked,most notably:

5: |T| < |S| and δ [T] > δ [S]; for example:

T = {2,3,5,7,11}, |T| = 5, δ [T] = 14; S = {31,37,43,47,53,59,101,113,127}, |S| = 9, δ [S] = 10.

Theorem 2.1.3 The (Proper) Superset Span Theorem

Let S be a superset of T, then δ [S] > δ [T].

Although this theorem appears to be rather trivial, there are many problems that will confront us
on our journey to solve the Andrica Conjecture whose only solution is to invoke this theorem. When all
hope seems lost, this simple theorem always manages to come and save the day.

Proof:

Let S ⊃ T, and let |S| = n and let |T| = m , and let V = S - T, and finally let |V| = c , then:

n>m ∴ n−m≥1 ∴ c≥1

Let μα ≡ a1 mod t1 ≡ a2 mod t2 ≡ ...am mod tm , such that Ψ(μα ) = δ [T].

1
Let μβ ≡ a1 mod t1 ≡ a2 mod t2 ≡ ...am mod tm ≡ 2
( δ [T]) mod v 1 , then Ψ(μβ ) > δ [T].

QED
We could increase the rigor of this proof via induction from v 1 to v c , but I believe this shall suffice
for this volume (for induction show that each increment of v j is a superset of the previous set and
therefore the maximal gap must increase with each succession).

For the remainder of this Volume, “Span” means that a set of primes can complete the length of
nucleus of the specified length.
Lemma 2.1.4, Logical Equivalents of The Proper Superset Span Theorem

The Superset Span Theorem rarely appears in its raw form, more often than not one of its logical
equivalents appears in disguise and must be revealed.

Let S be a proper superset of T, then δ [S] > δ [T].

From this it follows that:

1: Let T be a proper subset of S, then δ [T] < δ [S].


2: If S cannot span some particular distance, then neither can any of its proper subsets.
3: If T can span some particular distance, then so can any finite proper superset.
4a: If neither A nor B can span some particular distance, then neither can any proper subset of A
nor B.
4b: If neither A nor B can span some particular distance, then neither can (A ∩ B).
4c: If either A or B can span some particular distance, then so can (A ⋃ B).
4d: If (A Δ B) can span some particular distance, then so can (A ⋃ B).
4e: If (A ∩ B) can span some particular distance, then so can A and B.
4f: If (A ⋃ B) cannot span some particular distance, then neither can A nor B.
4g: If (A ∩ B) can span some particular distance, then so can (A ⋃ B).
4h: If (A ⋃ B) cannot span some particular distance, then neither can (A ∩ B) .
4k: If (A Δ B) can span some particular distance, then so can some subset of (A Δ B).
4m: If (A Δ B) cannot span some particular distance, then neither can any proper subset of
(A Δ B).

Definition 2.1.6, Owner of a Nucleic Set , Owner.

Let [S]𝚳k denote the Nucleic Set of length k in respect to the primes in S.

Definition 2.1.5, Primordial Set, the Union of all Nucleic Sets.

i = δ(S) i = δ(S)
Let [S]𝚳all = ∪ i=1
[S]𝚳i if 2 ∈/ S; or, let [S]𝚳all = ∪i=2
[S]𝚳i, Δi = 2, if 2 ∈ S.

i=n
|[S]𝚳all| = ∏(si − 1) , since it must contain each centrum between all consecutive totatives.
i=1
Definition 2.1.6 Principal Dirichlet Filter (Array Function), λ[a, b, c...n] , Kernel

Let Sn be ordered from least to greatest then:

Let C be an array of integers: [ c1 , c2 ...cn ], 0 ≤ ci < si , and let d ≡ c1 mod s1 ≡ c2 mod s2 ≡ ...cn mod sn .

Let λ(k) = 0 if (k − d) divides Ω[Sn], and let λ(k) = 1 if (k − d) does not divide Ω[Sn].

Let d be known as the Dirichlet Kernel, as there exists a vector in n-space whose respective components
assume the values of C, which terminates inside the corresponding cell of a rank-n tensor which contains
each and every possible system of congruences in respect to Ω[Sn].

Other than for the sake of counting surjections, we will not see the Filter function used again until
Volume 2 when approaching the Goldbach Conjecture.
2.2
Maps Between |S| = |T| and δ [S] = δ [T],

Lemma 2.2.1, Comparison of the Sizes of Primordial Sets

Let S and T be two distinct sets of primes such that S =/ T, and |S| = |T| and δ [S] = δ [T], then:

Either Ω (S) > Ω (T), or, Ω (S) < Ω (T) because of the Fundamental Theorem of Arithmetic. We
shall assume Ω (S) > Ω (T) for simplicity; therefore it follows that: |[S]𝚳all| > |[T]𝚳all|.

Definition 2.2.2, Large Prime set

Let S be a set of primes such that ∀s ∈ S, s > n , where n = |S|. Such a set is named a Large
Prime Set.

This can also be called a Belligerent Set, since every prime in the set is forced to be Belligerent is
it becomes active in the sieving of a nucleus.

Lemma 2.2.3, Surjection Lemma, 2 ∈ S

Let S and T be two distinct Large Prime Sets, such that S =/ T, and |S| = |T|, then there exists a
surjection from each set [S]𝚳k unto each set [T]𝚳k for all k , 2 ≤ k ≤ ( δ [S] - 2) and there exists a
bijection between [S]𝚳k and [T]𝚳k when k = δ [S].

Proof:

We know from the Class A1 Formula that δ [S] = δ [T] = n + 1 , where |S| = n , and that
| [S]𝚳 | = | [T]𝚳k | = n! when k = δ [S]. This proves the bijection statement.
k

We also know from the Class A2 formula that | [S]𝚳k | > | [T]𝚳k |. This is because of their innate
and simple polynomic identities:

Theorem 1.10.2 Class A2 Formula, The Permutative Count.


Let Sn be a set of prime numbers, and n be the cardinality of S, such that |S| = n and
∀s ∈ S, s ≥ (n + 1) , then for all k , such that k < (n + 1) , |𝚳k| = c , such that:

( )
m= (k−1) i=n
k
| [S]𝚳 | = ∑ (− 1)m (km−1) ∏ (si − 2 − m) .
m=0 i=1
QED
As we had started Section I with crude means of determining the counts, we shall do the same thing here. It does the
reader a great disservice to not initially engage and grapple these problems from a standpoint of total ignorance, since doing so
reveals the true nature of the problem at hand. As I said in the previous chapter, it is my opinion that the field of elementary number
theory has experienced stagnation for the better part of a century because no one wants to count anymore.

Let’s first examine a small numerical exercise (there’s no need to examine the bijective case of
the maximal gaps, you can mix and match them however you choose; also, the Class A1 and A2
formulas apply whether or not the prime number 2 is in S (other than doubling the size of the gap), thus,
to keep the modular arithmetic confined to integers (rather than half-integers), we shall retain 2 in S and
setup a map between the Nucleic sets of equal length of two different owners).

Let S = {2,7,67,71} and let T = {2,11,13, 19}, maximal gap equals 8. F is Nugatory Set.

S, Gap of 6, Principal Dirichlet Character on top, zeta function bottom, F = {71}


1 0 0 μ1 0 0 1

ξ (k) =∅ 2 7 2 67 2 ∅

T, Gap of 6, Principal Dirichlet Character on top, zeta function bottom, F = {19}


1 0 0 μ2 0 0 1

ξ (k) =∅ 2 11 2 13 2 ∅

Since 71 and 19 are the largest primes in their respective sets, they are allowed to be redundant
and included in the final count. This is how we prepare for exclusion and inclusion.

For these presets, there are 69 possible variations for S and 17 possible variations for T, thus
there a total of 52 more variations for S. We shall first identify the obvious bijection between the first 17
valid systems:

u1 ≡ 0 mod 2 ≡ 1 mod 7 ≡ − 1 mod 17 ≡ a mod 71; a =/ ± 3; + 9 ≥ a ≥− 9


u2 ≡ 0 mod 2 ≡ 1 mod 11 ≡ − 1 mod 13 ≡ a mod 19; a =/ ± 3; + 9 ≥ a ≥− 9 .
Recall that I reverse positive and negatives for remainders, since it corresponds to the direction they travel in a system.

In order to map greater remainders of 71 unto 19, we will need two independent Dirichlet Filters:

For each number between -35 and +35, eliminate those that are 3 mod 19 or − 3 mod 19 :
These numbers are: {+35, +22, +16, +3, -3, -16, -22, -35}. This is the “Surjection Denial Set.”

s s
The quantity of numbers eliminated is bounded between (f )(F loor[ t j ] and (f )(Ceiling[ t j ] , where
j j

f is the number of independent filters, but there is no explicit formula to obtain the precise count between
the bounds.

We now remove {+3 and -3} entirely from the Denial Set, since the remainders of neither 71 nor
19 may assume those values, and we are left with {+35,+22,+16, -16, -22, -35}. This is a total of six
numbers, which means we have a total of (71 − 2 − 6) = 63 systems of congruences which have a direct
surjection unto the cyclic remaiders of 19, and 6 non-trivial denials.

To be as formal as possible, we will divide all remainders of 71 into their four divisions modulo 19:

38 to 19, 18 to 0, 0 to -18 and -19 to -38.

First we realize that 0 is repeated twice, and that the bounds are +35 to -35. For the remaining
numbers in each of the four divisions, there exists a bijection between them and their respective cyclic
remainders of 19, giving us a total of 63 valid systems for the modulus 71 that can be mapped unto any of
the 17 valid systems modulo 19.

As for the remaining 6 remainders in the Denial Set, they tell us that they map unto a nucleus
whose length is longer than the original nucleus. Herein lies the crowning achievement of the
Surjection Theorem (the focus of Section 4 of this chapter), where we show that a nucleus generated by
any set of primes, Sn, maps unto a nucleus of the same length or longer for the set of consecutive
primes, Pn. However, we still have a ways to go before we can prove that statement, but it’s the first big
step towards solving the Andrica Conjecture and therefore well worth the effort.

As for now, we have identified the 63 systems which preserve the integrity of the nucleus and the
6 systems which do not. If we take the additive inverses (reversing the positions of all the primes), we
have thus accounted for 126 surjections and 12 non-trivial denials.

Let us continue with this numerical exercise to identify the remaining surjections and denials, so
we can see how the inclusion exclusion of repeated solutions is handled.

S = {2,7,67,71}; T = {2,11,13, 19}


S, Gap of 6, Principal Dirichlet Character on top, zeta function bottom, F = {67}

1 0 0 μ1 0 0 1

ξ (k) =∅ 2 7 2 71 2 ∅

T, Gap of 6, Principal Dirichlet Character on top, zeta function bottom, F = {13}


1 0 0 μ2 0 0 1

ξ (k) =∅ 2 11 2 19 2 ∅

Our first system has 64 possible variations for the modulus 67, and our second system has 10
possible variations for the modulus 13. Why 64 and 10 instead of 65 and 11? Because 67 cannot divide
the integer which is being divided by 71, nor can 13 divide the integer which is being divided by 19. Those
redundant systems were already counted in the previous iteration. Thus in order to exclude them, we
must add a third Dirichlet Filter for systems − 1 mod 13 .

In this particular case, since the prime 13 is relatively small, we can count all the of the
surjections and denials along a number line divided into six partitions (13 was chosen on purpose for this
reason).
Partition A, Bijective Partition, Range: +6 to -6; 10 surjections, 1 denial (2 trivial denials, not counted).
13 +6 +5 +4 +3 +2 +1 0 -1 -2 -3 -4 -5 -6

67 +6 +5 +4 +3 +2 +1 0 -1 -2 -3 -4 -5 -6

The systems in yellow at +3/-3 are invalid for both moduli, they cannot be allowed to divide the
terminals of the nucleus; the systems in blue are the third Dirichlet Filter acting on excluded entities. This
partition provides us with a total of 10 surjections (this particular partition is the named the Bijective
Partition). Remember that the third filter acts on all integers -1 mod 13.

Partition B1, Surjective Partition. Range: +19 to +7; 10 surjections, 3 denials.


13 +6 +5 +4 +3 +2 +1 0 -1 -2 -3 -4 -5 -6

67 +19 +18 +17 +16 +15 +14 +13 +12 +11 +10 +9 +8 +7
Partition B2, Surjective Partition. Range: -7 to -19; 10 surjections, 3 denials.
13 +6 +5 +4 +3 +2 +1 0 -1 -2 -3 -4 -5 -6

67 -7 -8 -9 -10 -11 -12 -13 -14 -15 -16 -17 -18 -19

Partition C1, Surjective Partition. Range: +32 to +20; 10 surjections, 3 denials.


13 +6 +5 +4 +3 +2 +1 0 -1 -2 -3 -4 -5 -6

67 +32 +31 +30 +29 +28 +27 +26 +25 +24 +23 +22 +21 +20
Partition C1, Surjective Partition. Range: -20 to -32; 10 surjections, 3 denials.
13 +6 +5 +4 +3 +2 +1 0 -1 -2 -3 -4 -5 -6

67 -20 -21 -22 -23 -24 -25 -26 -27 -28 -29 -30 -31 -32

Partition D1, Surjective Partition. Range: +45 to +33; 1 surjections, 0 denials.


13 +6 +5 +4 +3 +2 +1 0 -1 -2 -3 -4 -5 -6

67 +45 +44 +43 +42 +41 +40 +39 +38 +37 +36 +35 +34 +33
Partition D2, Surjective Partition. Range: -33 to -45; 11 surjections, 0 denials.
13 +6 +5 +4 +3 +2 +1 0 -1 -2 -3 -4 -5 -6

67 -33 -34 -35 -36 -37 -38 -39 -40 -41 -42 -43 -44 -45

Since there are no more remainders for 67 beyond +/- 33, the overwhelming mass of the sister
D1 and D2 partitions are excluded (in purple), and we accept only the terminals (in green). We now sum
together all of the surjections, we will execute a telescopic sum for algorithmic purposes:

A + (B1+B2) + (C1+C2) +(D1 + D2)


10 + (10+10) + (10+10) + (1+1) = 52
Thus there are 52 surjections among the 67 remainders of the modulus 67, because there was a
total of 15 denials, of which 2 are trivial denials. If we are cautious to remember that two of those denials
are automatic (trivial) in the Bijective Partition, then we have a total of 13 non-trivial denials.

This makes counting surjections a relatively simple process by subtracting the denials, although
we need to address the tedious process of counting the denials in the final sister partitions, ε . I choose ε
to represent this number since it falls off for analytical applications and estimations (as does the floor
function itself).

s
Number of Denials = (f )(F loor[ t i ]) + 2ε , where f is the number of filters.
i

In order to determine ε we have to apply a direct sieve over a range of c consecutive integers,
where the sieve utilizes the filters. There is no other way to determine ε . We can determine c however:

1 1
a= (s
2 i − 1) = 33 = 2
(67 − 1) .

b = a − 12 (ti − 1) = 27 = 33 − 12 (13 − 1) .

c = b mod ti ; 0 ≤ c ≤ ti = 27 mod 13 = 1

z = F loor( at )
i

Apply sieve over the range: z (ti ) + c ; Number of Denials in the final partition is the number of
integers sieved in that range by the filters.

Returning to the numerical example we have 52 surjections and 12 non-trivial denials between
these two presets, we now double these numbers for the additive inverses of the presets: 104 and 24.
Let’s finish the first part of the problem with the final rank of presets (you will soon see why I choose the
word rank, and why this problem is not yet nearly over):

S = {2,7,67,71}; T = {2,11,13, 19}


S, Gap of 6, Principal Dirichlet Character on top, zeta function bottom, F = {7}

1 0 0 μ1 0 0 1

ξ (k) =∅ 2 67 2 71 2 ∅

T, Gap of 6, Principal Dirichlet Character on top, zeta function bottom, F = {11}


1 0 0 μ2 0 0 1

ξ (k) =∅ 2 13 2 19 2 ∅

Here we are looking only for the instances where 7 and 11 are absent. They are not allowed to be
redundant (by dividing μ ± 1 ), since those solutions have already been counted by the two previous
iterations. Thus we must now apply four filters: 3 mod 7; 1 mod 7, − 1 mod 7; − 3 mod 7 .

Partition A, Bijective Partition, Range: +3 to -3; 3 surjections, 0 denials, 4 trivial denials.


7 -1 -2 -3 +3 +2 +1 0 -1 -2 -3 +3 +2 +1

11 -5 +5 +4 +3 +2 +1 0 -1 -2 -3 -4 -5 +5

In this partition, everything beyond +/- 3 is excluded. The prime number 7 is smaller than 11, and
thus our surjection instead transforms to an injection from the remainders of 7 into the remainders of 11.
Thus we result with 3 surjections and 0 denials and 4 trivial denials. The denials in red were already
counted in the previous iteration, those denials are: μ ≡ 1 mod 7 ≡ 1 mod 67 ; μ ≡− 1 mod 7 ≡− 1 mod 71 .

That’s it! We just have to double the count for the additive inverses of the presets: 6 and 0.

Let’s now sum up the surjections and denials from all the iterations:

We have 126+104+6= 236 surjections, and, 12+24+0 = 36 non trivial denials. If we add together
236 and 36, we get 272, which is the size of the Nucleic Set of length 6 for three primes (length 3 when 2
is not in the set).

S = {2,7,67,71}; T = {2,11,13, 19}

( )
m= (k−1) i=n
m
c= ∑ (− 1) (km−1) ∏ (si − 2 − m) = 22,425 -34816+12663 = 272.
m=0 i=1

In the above formula k = 3 , since we are counting gaps of 6 (gaps of 3 with 2 in S).
Let’s count the size of the nucleic set of length 6 for T:

S = {2,7,67,71}; T = {2,11,13, 19}

( )
m= (k−1) i=n
m
c= ∑ (− 1) (km−1) ∏ (ti − 2 − m) = 1683 -2560+945 = 68.
m=0 i=1

Thus of the 272 systems for S, 236 of them map onto the 68 systems for T, with each system in T
being projected onto an average of 3.47 times (236/68).

For the remaining 36 Denials from S, they each map onto some other nucleus of T whose length
is greater than 6. In this case we know that all of those are nuclei of length 8, since 8 is the maximal
length of any Nucleic Set for either S or T; a set of three large primes can’t sieve more than three integers
(or three odd integers with 2 in the set).
Let us now examine the geometry behind what actually occurred here. The geometry is what
allows us discern the true mechanism by which these counts occur. We will see that they too are
polynomic in nature, but due to the indivisibility of prime numbers by other prime numbers, no direct
solutions exists. The value of ε in each iteration simply has no formulic value, it can only be obtained
through an algorithmic process.

Here was the first iteration of our counting process:

Let S = {2,7,67,71} and let T = {2,11,13, 19}, maximal gap equals 8. F is Nugatory Set.

S, Gap of 6, Principal Dirichlet Character on top, zeta function bottom, F = {71}


1 0 0 μ1 0 0 1

ξ (k) =∅ 2 7 2 67 2 ∅

T, Gap of 6, Principal Dirichlet Character on top, zeta function bottom, F = {19}


1 0 0 μ2 0 0 1

ξ (k) =∅ 2 11 2 13 2 ∅

Let’s remove the belligerent primes to reveal the unsieved integers at μ ± 1 (two unsieved). The
Principal Dirichlet Characters will be replaced with integers.
μ−3 μ−2 μ−1 μ μ+1 μ+2 μ+3

ξ (k) =∅ 2 ∅ 2 ∅ 2 ∅

Since we have three belligerent primes and two unsieved integers, we have six vectors in
2-space that correspond the necessary remainders for each modulus that are required to complete the
length of this nucleus. Let W be generalization of both S and T: W = {2, w1 , w2 , w3 }. We need at least two
primes in W to sieve the integers at μ − 1 and μ + 1 : μ ≡ + 1 mod wi ≡ − 1 mod wj . Thus our six vectors
occur in this form:

1: μ ≡ + 1 mod w1 ≡ − 1 mod w2
2: μ ≡ + 1 mod w1 ≡ − 1 mod w3
3: μ ≡ + 1 mod w2 ≡ − 1 mod w3
4: μ ≡ + 1 mod w2 ≡ − 1 mod w1 (additive inverse of 1)
5: μ ≡ + 1 mod w3 ≡ − 1 mod w1 (additive inverse of 2)
6: μ ≡ + 1 mod w3 ≡ − 1 mod w2 (additive inverse of 3).

We can now replace each w with their respective s and t moduli. Let’s proceed to the next page
go get a thorough look of these 2-vectors occurring on plane.
S = {2,7,67,71}; T = {2,11,13, 19}

For S, x -axis will belong the modulus 7, and for T it will belong to the modulus 11.
For S, y -axis will belong the modulus 67, and for T it will belong to the modulus 13.
For S, z -axis will belong the modulus 71, and for T it will belong to the modulus 19.

Let’s first look at the x - y plane for S and T side by side, bounded by +3 to -3 and +33 and -33 for S,
and +5 to -5 and +6 to -6 for T.

The locations of (1,-1) and (-1, 1) are marked by the letter G. These are our solution vectors. The blue
lines show the bounds of the plane. The first plane is thinner, but taller, likewise the second plane is wider,
but shorter. We’ll also place yellow marks with the letter H to show the automatic rejection that occurs at
+/-3 for the terminals of the nucleus.

+ 3 ≥ x ≥− 3; + 33 ≥ y ≥− 33 + 5 ≥ x ≥− 5; + 6 ≥ y ≥− 6
x x x x x -6 x x x x x

-33 x -5 x
x x x x x x
x -4 x
x ... x
x -3 x
x -3 x
x -2 x
x -2 x
x G -1 x
x G -1 x
5 4 3 2 1 0 -1 -2 -3 -4 -5
3 2 1 0 -1 -2 -3
x 1 G x
x 1 G x
x 2 x
x 2 x
x 3 x
x 3 x
x 4 x
x ... x
33
x 5 x
x x x x x x
x x x x x 6 x x x x x
Similar to Section 1 of this Chapter, we’ll now place yellow marks to exclude the trivial rejections
at +/-3. Neither S nor T contain the yellow solutions, and thus they are trivial denials.

+ 3 ≥ x ≥− 3; + 33 ≥ y ≥− 33 ; x, y =/ ± 3 + 5 ≥ x ≥− 5; + 6 ≥ y ≥−; , x, y =/ ± 3
x x x x x -6 x x x x x
-33
x x x x x x x x -5 x x

x ... x x x -4 x x

x x x x x x x x x x x x x x x x x x

x -2 x x x -2 x x

x G -1 x x x G -1 x x

3 2 1 0 -1 -2 -3 5 4 3 2 1 0 -1 -2 -3 -4 -5

x 1 G x x x 1 G x x

x 2 x x x 2 x x

x x x x x x x x x x x x x x x x x x

x ... x x x 4 x x
33
x x x x x x x x 5 x x

x x x x x 6 x x x x x
Now let’s impose the matrix S over the matrix T. The x-axis will be bounded at +/- 3, since that
represents the injection from modulus 7 unto modulus 11, but the y-axis will be repeated modulo 13, until
it “crashes’ into the boundary at +/- 33, since it represents the surjection of numbers modulo 67 unto
modulus 13. The blue denials are − 3 mod 13 and the purple denials are + 3 mod 13 .
With the above in mind, let’s now take a look at the x, y ; x, z and y , z planes super-imposed upon
each other (there will be some artistic license here for the y , z plane, since I’m trying to convey the
general idea, and thus the y , z plane will not be drawn to scale nor to exact numerical perfection).

In each plane, the red squares at +/- 1, show us the solution vectors which are then orthogonally projected through each
cell of the complimentary missing dimension, albeit respecting the cyclic bounds of that missing dimensions as well. Similar to our
Section I drawing, from each plane would extend two columns into the third orthogonal direction (slates), although some of slates
would be rejected (blue or purple, instead of only the slates in yellow). If we were in four dimensions (four big primes in the set),
we’d be projecting the the 2D solution vectors through every white cell of the complementary 2D-space.
Optional Reading,
Skip to page 161, Appendix C, if you wish to know how to obtain the precise count of
of maps between two sets of big primes of the same cardinality.
2.3
Types of Subsets and Intersections, Exchange Set

Definition 2.3.1a, Divorced Set

Let S and T be sets of primes such that S ⋂ T is empty, then these sets are said to be divorced.

Definition 2.3.1b, Familial Set

Let S and T be sets of primes such that S ⋂ T = T , then T is a subset of S, these sets are said
to be familial.

Definition 2.3.1c, Estranged

Let S and T be sets of primes such that S ⋂ T is not empty, but is not equal to S or T , then
these sets are said to be estranged.

Definition 2.3.1d, Exchanged Set

Let |S| = |T| = n , and let S and T be estranged, such that S ⋂ T = D


then let E be the Exchange Set, which will be equal to the symmetric difference of X and Y,
E = (S U T) - (D), or more formally E = S Δ T.

Now let |D| = m , and let ℓ = n − m


and let E′ ⊆ E, such that |E′| = ℓ.

Finally, let X = D U E′, and therefore |X| = m + ℓ = n

X is said to be an exchanged set, as it a proper subset of the union S and T, while having the
same cardinality as S and T, and containing all elements of their intersection, with the remaining elements
being derived from their symmetric difference.
Maps Between |S| > |T| and T ⊂ S.

Lemma 2.3.1, Comparison of the Sizes of Familial Primordial Sets

Let S and T be sets of primes such that T ⊂ S, and |S| = n ; |T| = m ; then,

Let V = S - T, |V| = n − m = c

Ω (S) > Ω (T); ( Ω (S)) / ( Ω (T)) = ( Ω (V)).

Lemma 2.3.2, Familial Surjection Lemma, 2 ∈ S

Let S and T be sets of primes such that T ⊂ S, and |S| = n ; |T| = m ; then,

Let V = S - T, |V| = n − m = c .

Then δ [S] > δ [T] = d .

( )
i=c
∀k, 2 ≤ k ≤ d, Δd = 2 , | [S]𝚳k | ≥ k
∏(v i − 2) | [T]𝚳 |
i=1

Proof:

Let μ ≡ a1 mod t1 ≡ a2 mod t2 ...am mod tm , a =/ ± 12 k .

Each solution that permits of length of k for the primes in T can also be written as:

μ ≡ a1 mod t1 ≡ a2 mod t2 ...am mod tm ≡ am+1 mod v 1 ≡ am+2 mod v 2 ...am+c mod v c ; a =/ ± 12 k .

Since each of the solutions for the Primes in T already completed the nucleus, the primes in V
are free to sieve any integers within (redundant) or outside (absent) the nucleus, except the terminals of

(∏ )
i=c
the nucleus itself. As such, there exists a minimum of (v i − 2) | [T]𝚳k | solutions in respect to the
i=1

primes in V that can complete a nucleus of length k due to Chinese Remainder Theorem.

Therefore there exists a surjection from [S]𝚳k unto [T]𝚳k.

QED
Lemma 2.3.3, Familial Bijection Lemma, 2 ∈ S

Let S and T be sets of primes such that T = S, and |S| = n ; |T| = m ; then,

Let V = S - T, |V| = n − m = c = 0 .

Then δ [S] = δ [T] = d .

∀k, 2 ≤ k ≤ d, Δd = 2 , | [S]𝚳k | = |[T]𝚳k |.

Therefore there exists a bijection between [S]𝚳k and [T]𝚳k.

Lemma 2.3.4, Familial Injection Lemma, 2 ∈ S

Let S and T be sets of primes such that T ⊃ S, and |S| = n ; |T| = m ; then,

Then δ [S] < δ [T] = d .

∀k, 2 ≤ k ≤ d, Δd = 2 , | [S]𝚳k | < | [T]𝚳k |

Proof:

Corollary of Lemma 2.2.2.

Therefore there exists an injection from [S]𝚳k into [T]𝚳k.


QED
2.4
Orthogonal Surjections, Bijections and Injections.

Proposition 2.4.1, Absolute Orthogonal Surjection

Let S and T be sets of primes and let D = S ⋂ T , |S| = |T| = n ; |D| = m ; δ [S] ≤ δ [T] ; δ [S] ≤ c .

If si ≥ ti , ∀i, i ≤ n , and ∃ j such that sj > tj , then:

Let V = S - D and let W = T - D; |V|, |W| = n − m = f

And let D, V and W be ordered from least to greatest; finally, let Ψ(μ) = k , therefore γ = 12 k , then:

There exists an absolute orthogonal surjection from all nuclei of length k , k ≤ c , generated by S
unto those generated by T, since each and every dimension of the Rank- n Tensor corresponding to the
primes in V is greater than each and every dimension of the Rank- n Tensor corresponding to the primes
in W.

Therefore every belligerent solution vector in (k − x) -space, that is in a subspace of D-space, is


orthogonally projected through each cell (n − k + x) space, which is a superspace of V-space, within the
bounds − 12 (v j − 1) ≤ bj ≤ + 12 (v j − 1), bj =/ ± γ , ∀j ≤ f ...

...such that if every belligerent solution vector in (k − x) -space, that is in a subspace of D-space,
is orthogonally projected through each cell (n − k + x) space, which is a superspace of W-space, within the
bounds − 12 (wj − 1) ≤ bj ≤ + 12 (wj − 1), bj =/ ± γ , ∀j ≤ f , then the volume of the bounded W-space would
be smaller than the volume of the bounded V-space, with both W and V being in f − space.

Proof:
This Proposition cannot be proven until:

1: The Surjection Theorem is proven (Chapter 1, Section IV).


2: The Belligerence Mandate is proven (Chapter 2).

We shall continue this proof towards the end of Chapter 2.

Proof on Hold.

The reason we cannot proceed at this time to prove this proposition is because we don’t yet know
that there always exists two or more belligerent primes inside of any nucleus, which is the Belligerence
Mandate. Furthermore, the Belligerent Mandate cannot be proven until the Surjection Theorem is proven.

The Belligerence Mandate assumes that {2,3} are in both S and T, such that all gaps of length 6
or greater (length 3 or greater without the prime 2 in S or T) have a minimum of two belligerent primes.

Gaps of 2 and 4 (or 1 and 2 without the prime 2 in S) have only one belligerent prime. If neither 2
nor 3 is in S or T, then the Belligerence Mandate can be easily invoked via the Surjection Theorem.
Lemma 2.4.2, Absolute Orthogonal Bijection

Let S and T be sets of primes and let D = S ⋂ T = S ⋃ T ;.

Since D = S ⋂ T = S ⋃ T; D = S = T, therefore |D| = |S| = |T| = n ; |D| = m ; δ [D] = δ [S] = δ [T] ; δ [S] = c

Let V = S - D and let W = T - D; |V|, |W| = n − m = 0

And let D be ordered from least to greatest; finally, let Ψ(μ) = k , therefore γ = 12 k , then:

There exists an absolute orthogonal bijection from all nuclei of length k , k ≤ c , generated by S
unto those generated by T, since the bounds on the dimensions of the Rank- n tensor in D-space are
equal for both S and T.

Proof:
This equivalent to Lemma 2.3.3, the Familial Bijection Lemma.

QED
Proposition 2.4.1, Absolute Orthogonal Injection

Let S and T be sets of primes and let D = S ⋂ T , |S| = |T| = n ; |D| = m ; δ [S] ≤ δ [T] ; δ [S] ≤ c .

If sI ≥ ti , ∀i, i ≤ n , and ∃ j such that sj > tj , then:

Let V = S - D and let W = T - D; |V|, |W| = n − m = f

And let D, V and W be ordered from least to greatest; finally, let Ψ(μ) = k , therefore γ = 12 k , then:

There exists an absolute orthogonal injection from all nuclei of length k , k ≤ c , generated by T
into those generated by S, since each and every dimension of the Rank- n Tensor corresponding to the
primes in W is less than each and every dimension of the Rank- n Tensor corresponding to the primes in
V.

Therefore every belligerent solution vector in (k − x) -space, that is in a subspace of D-space, is


orthogonally projected through each cell (n − k + x) space, which is a superspace of W-space, within the
bounds − 12 (wj − 1) ≤ bj ≤ + 12 (wj − 1), bj =/ ± γ , ∀j ≤ f ...

...such that if every belligerent solution vector in (k − x) -space, that is in a subspace of D-space,
is orthogonally projected through each cell (n − k + x) space, which is a superspace of V-space, within the
bounds − 12 (v j − 1) ≤ bj ≤ + 12 (v j − 1), bj =/ ± γ , ∀j ≤ f , then the volume of the bounded V-space would be
larger than the volume of the bounded W-space, with both W and V being in f − space.

Proof:
This Proposition cannot be proven until:

1: The Surjection Theorem is proven (Chapter 1, Section IV).


2: The Belligerence Mandate is proven (Chapter 2).

We shall continue this proof towards the end of Chapter 2.

This is also a corollary of 2.4.1, and vice versa, once either is proven, so is the other.

Proof on Hold.
Section III
The Strong Andrica Conjecture

The Andrica Conjecture concerns itself only with the limit of the difference between a pair of
consecutive prime numbers, yet the work that we have been doing concerns the limit of the difference
between a pair of consecutive totatives.

If we can show that the limit of the difference between a pair consecutive totatives is less than
twice greatest prime dividing the Ω-modulus of those totatives, and, by definition, since a pair of
consecutive prime numbers are totatives to the product of all the primes less than the square root of the
lesser consecutive prime number (Sieve of Eratosthenes), then we shall have proved the Andrica
Conjecture.

Cramér's conjecture
It should also be noted that if we can show that the maximal gap cannot occur between the
squares of the two greatest primes in the sieving set (within the quadratic interval), and furthermore, that
gaps of similar size (but lesser size) also cannot occur within the quadratic interval, then we can solve the
Cramer conjecture as well.

Disclaimer
There are many statements in Section III that operate under the assumption the Strong
Andrica, Cramer and Firoozbakht Conjectures are true. The reader MUST remember that no proof
of these conjectures is actually claimed or asserted to have been completed or inferred in Section
III.

This Section serves only to state the strengthened forms of the conjectures with some
accompanying illustrations and figures to demonstrate how I arrived at the original hypotheses
that gave me the guidance and direction to eventually prove them. This Section also mentions the
Goldbach Conjecture (the focus of Volume II) and the K-Tuple Conjecture (also the focus of
Volume II), and will also make a brief mention of the Riemann Hypothesis (Volume III). The reader
also cannot skip this section since it contains a great multitude of definitions.

Also, neither Volume II nor Volume III contains a proof of the K-Tuple Conjecture or the
Riemann Hypothesis. I do hope however that work I have done today shall benefit the
mathematicians of the future on those endeavours, because they certainly won’t be solved in our
lifetimes, but I know one day they will be, and the work done today is the foundation upon which
those proofs shall be built.

This section is also written in a somewhat informal manner.

The reader can skip to Section IV (The Surjection Theorem) if they wish, but they must
return to Section III before continuing on to Chapter 2.

/
3.1 Strengthening the Andrica,
Cramér, and Firoozbakht Conjectures

We are fortunate that the limit of the difference between consecutive totatives is already known in
respect to sets of primes whose cardinality is less than or equal to 13. This limit was obtained by brute
force. The brute force was performed manually for all sets of consecutive primes starting from either 2 or
3, whose cardinality is less than or equal to 7, and the brute was performed by computer for all sets of
consecutive primes (starting from 2 or 3) whose cardinality was less than or equal to 13 (and the manual
brute force experiments were re-checked by computer for sets of primes whose cardinality was less than
or equal to 7). See Appendix A for a sample of the computer script.

Let P be a set of consecutive primes starting from 2 and let |P| = n . The maximum span of P is
less than or equal to 2pn−1 , and the system of congruences for each centrum of the nuclei of maximal
length 2pn−1 has the following form:

μ1 ≡ 0 mod 2 ≡ 0 mod 3 ≡ 0 mod 5... ≡ 0 mod pn−2 ≡ − 1 mod pn−1 ≡ + 1 mod pn


μ2 ≡ 0 mod 2 ≡ 0 mod 3 ≡ 0 mod 5... ≡ 0 mod pn−2 ≡ + 1 mod pn−1 ≡ − 1 mod pn

Notice that both of these centrum are just additive inverses modulo Ω[P].

Thus, there is only one unique system of congruences that can span 2pn−1 if n ≤ 13 , if we
discount the additive inverse.

We will use this observation as our base case for proof by induction of the Andrica Conjecture.
The case n = 7 will be used since it can be confirmed very quickly with a computer that the maximum
span of {2, 3, 5, 7, 11, 13, 17} is indeed 26 and occurs in the stated form, and only occurs once (not
counting its additive inverse).

The next step towards solving the Andrica Conjecture will be to show this applies to any set of
primes with a cardinality less than or equal to 7.

/
Definition 3.1.1 The Strong Andrica Conjecture

Let P be a set of consecutive primes starting from 2 and let |P| = n . The maximum span of P is
equal to 2pn−1 , and the system of congruences for each midpoint of the nucleus generating a length
2pn−1 have the following form:

μ1 ≡ 0 mod 2 ≡ 0 mod 3 ≡ 0 mod 5... ≡ 0 mod pn−2 ≡ − 1 mod pn−1 ≡ + 1 mod pn


Ω[P] - μ1 = μ2 ≡ 0 mod 2 ≡ 0 mod 3 ≡ 0 mod 5... ≡ 0 mod pn−2 ≡ + 1 mod pn−1 ≡ − 1 mod pn

Definition 3.1.2 The Andrica Conjecture Base Case,

Let P be a set of consecutive primes starting from 2 and let |P| = n . The maximum span of P is less than
or equal to 2pn−1 if n ≤ 7 and it only occurs once (ignoring the additive inverse). The system of
congruences for the midpoint of this maximum span is:

μi ≡ 0 mod 2 ≡ 0 mod 3 ≡ 0 mod 5... ≡ 0 mod pn−2 ≡ − 1 mod pn−1 ≡ + 1 mod pn

The Base Case uses n = 7 , which generates the set {2, 3, 5, 7, 11, 13, 17}.
End of Definition 3.2.1

Notice the centrum is divisible the product Ω[Pn-2], thus, if the Strong Andrica Conjecture is true,
then when n ≥ 7, pn = 17 , that Ω[Pn-2] ≥ 2310 , such that maximal gap can no longer occur between the
squares of 13 and 17 (169 and 289), or more generally, can no longer occur between the squares of pn−1
and pn . Thus the true limit (limit superior) of the difference between two consecutive prime numbers is
actually less than 2pn−1 .

The last pair of consecutive prime numbers that obey the limit of 2pn−1 are 113 and 127, which is
a gap of 14, where 14 = 2(7) = 2pn−1 , since the square root of 127 is slightly more than 11, 11 = pn , which
just permits the prime number 11 to enter the sieving set.

Later in this volume, we will see a similar result for the Strong Cramer Conjecture, where similarly
large gaps cannot occur between the squares of pn−1 and pn since such gaps contain integers that are
divisible by the product of too many primes in Pn. Our quest to solve the Andrica Conjecture is also
simultaneously a quest to solve the Cramer Conjecture as well!

Let’s manually check the set {2,3,5,7,11,13,17}. See next page.

/
P = {2,3,5,7,11,13,17}; n = 7 , Ω(P) = 510,510; 𝟇Ω(P) = 92,160

i=n i=n
|M2| = ∏ (pi − 2) = 22,275 |M4| = ∏ (si − 2) = 22,275
i=3 i=3

(∏ )
i=n i=n
|M6| = 2 (si − 2) − ∏ (si − 3) , n ≥ 3 = 26,630
i=3 i=3

i=n i=n i=n


|M8| = ∏ (si − 2) − 2 ∏ (si − 3) + ∏ (si − 4) , n ≥ 4 = = 6812
i=3 i=3 i=3

i=n i=n i=n


|M10| = 43 ∏ (si − 2) − 2 3
2 ∏ (si − 3) + 21 ∏ (si − 4) , n ≥ 4 = 7,734
i=3 i=3 i=3

See Appendix G for the derivations of remaining formulas counting the gaps of 12 through 26:

|M12| = 4096 |M14| = 1406 |M16| = 432 |M18| = 376 |M20| = 24 |M22| = 78 |M24| = 20 |M26| = 2

2(22275)+4(22275)+6(26630)+8(6812)+10(7734)+12(4096)+14(1406)+16(432)+18(376)+20(24)+22(78)+
24(20)+26(2) = 510510, check.

(22275)+(22275)+(26630)+(6812)+(7734)+(4096)+(1406)+(432)+(376)+(24)+(78)+(20)+(2) = 92160,
check.

Food for thought!


There is an incredible multitude of positive integer solutions to the system of linear equations:
92, 160 = x1 + x2 + ...x13 and 5101510 = 2x1 + 4x2 + ...26x13 ; or more generally,

𝟇Ω(P) = x1 + x2 + ...x(pn −1) and Ω(P) = 2x1 + 4x2 + ...2(pn−1 ).x(pn −1)

What are such alternate positive integer solutions counting in respect to the permutations of the
primes in P?

Hint: Imagine a sphere with two perpendicular longitudinal axes intersecting at the south and north poles, the first axis are
the reals and the second axis are the imaginary numbers. The coordinate at the south pole is (0, 0 i ) and the coordinate at the north
pole is ( Ω(P), Ω(P) i ). All vectors from the south pole are longitudinal lines that meet at the North pole, and along these vectors
exist Gaussian primes.

Such an entity is known as a Riemann Sphere, and when integer coordinates along its longitudinal lines are converted to
either 1’s or 0’s ( λ function), it becomes a Dirichlet Sphere, and when projected onto a flat plane, it becomes known as a Dirichlet
Disk.
http://mathworld.wolfram.com/RiemannSphere.html

This entity serves as the bridge from the elementary proceedings of Volumes I and II into the Riemann Hypothesis in
Volume III. Other positive integer solutions to the above two systems of linear equations count different phases of the Galaxy along
non-real longitudes. Also remember that Ω(P) is congruent to 0 in modulo Ω(P). The equatorial line at 12 Ω(P) on the sphere
1
represents a great circle of symmetry, just as the positions of totatives can be inverted (reflected) over 2
Ω(P).

However, unlike the traditional Riemann Sphere, this Sphere represents a finite ring of Gaussian primes and composites.

/
Before I begin to strengthen the Cramer and Firoozbakht Conjectures, a large new set of
definitions will be needed. I shall borrow words from the lexicon of physics since they best describe the
visual structures of the sieving primes. When I was teenager, I was fascinated with the study of physics, it
was from my passion on the subject that I used these to best describe the patterns and figures that I
observed.

Definition 3.1.3 Star, radius.


A star is a set of consecutive inclusive nuclei, starting with some totative of Ω[S] and likewise
ending on some totative of Ω[S]. The half-length of this star is the radius. A star with no interior totatives is
a lone nucleus.

Definition 3.1.4 Constellation

Starting with some totative of Ω[S], tx and likewise ending on some totative of Ω[S], ty , such that
the difference (ty − tx ) divides Ω[S], implying that (ty − tx ) = Ω[S′], where S′ is some subset of S.

Definition 3.1.5 Nuclear Fusion

A change in the system of congruences that combines two or more successive nuclei within the
original star by altering the remainders. This process must preserve the radius of the original star.

Definition 3.1.6 Nuclear Fission

A change in the system of congruences that separates any nuclei within the original star by
altering the remainders. This process must preserve the radius of the original star.

Definition 3.1.7 Baryogenesis

A change in the system of congruences that alters the moduli, but preserves the radius of the
original star from the parent galaxy. Such a change may include adding or subtracting primes to (or from)
S (increasing or decreasing the number of moduli), or relegating/removing primes to/from nugatory status.

Definition 3.1.8 Nova, Nebulae; Collapse, Collapsar

A nova is change in the system of congruences that does not preserve the radius, causing the
radius to increase, the resulting figure is called a nebulae; likewise a change that causes the radius to
shrink is a collapse, and the resulting figure is called a collapsar.

Definition 3.1.9 Singularity, White Hole

A mathematical contradiction due to a change in a system of congruences that causes the zeta
function to have multiple evaluations for any integer within a star. A white hole is a singularity which
contains a null value among its evaluations.

The singularity is one of our strongest tools, it pervades all three volumes!

/
Definition 3.1.10 Beta Decay

An incident of nuclear fission by which a belligerent (or bracketed) prime is relocated (or
destroyed), creating a totative(s) at the location(s) where that belligerent (or bracketed) prime had
originally sieved an integer within the original star. See Definition 3.1.15, Bracketed Prime.

Definition 3.1.11 Solar System

A star whose length is equal to 2p for any p ∈ Pn, p ≥ 5 . By definition both 2 and 3 must divide
the center of this star to avoid sieving the terminals.

Definition 3.1.12 Planet

Any totative within a solar system is a planet. This includes the totatives that bound the star, thus
all solar systems have a minimum of two planets.

Definition 3.1.13 Goldbach Solar System

A star whose length is equal to 2px−1 for any p ∈ Pn, p ≥ 5 and contains only one nucleus (itself),
such that:
μ ≡ 0 mod 2 ≡ 0 mod 3 ≡ ... 0 mod px−2 ≡ − 1 mod q ≡ + 1 mod t . where (q, t) = ξ (μ ± 1) , (q, t) ∈ Pn

Definition 3.1.14 The Quartic Set, ⍴

∃py ∈ Px such that py < √ px−1 and py+1 > √ px−1 .


Then let Q ⊂ Px and ∀q ∈ Q, q < √ px−1 , such that Q = { 2, 3, 5...py }
Q is said to be the Quartic Set in respect to px .
Let |Q| = ⍴

/
Definition 3.1.15 Bracketed Prime (meson), Anchor Δ .

If there exists a prime in S that is active and not belligerent (harmonious), but only sieves two
integers inside of a nucleus, then this prime is said to be bracketed, and the midpoint of the bracket is
called an anchor, which is denoted by Δ instead of μ (centrum). A bracketed prime may also be referred
to as a meson, based on the immediate context.

A system of congruences where the prime number 5 is bracketed (gap of 18).


λ 1 0 0 0 0 0 0 0 0 μ 0 0 0 0 0 0 0 0 1

ξ ∅ 2 5 2 3 2 37 2 79 2 3 2 5 2 29 2 41 2 ∅

[5 Δ 5]

Δ ≡ 0 mod 2 ≡ 0 mod 3 ≡ 0 mod 5.

The strength of brackets is that they form a “sub-gap,” from which the bracketed prime itself is
absent (and therefore nugatory), allowing us to reduce the size of sieving set within the bracket. This
nugatory feature of brackets is what allows us to reduce a set of Sn+1 primes to Sn in the final stages of
completing the proof of the Andrica Conjecture via induction on the set Pn+1 .

/
Definition 3.1.16 Perturbed Goldbach Solar System

A Goldbach solar system that underwent nuclear fission to produce one or more planets, such
that the original centrum was subjected to this particular transformation:

Given px−k > √ px−1 ; px−k ≤ px−2 and for z =/ 0 :


μ ≡ z mod px−k ≡ 0 mod 2 ≡ 0 mod 3 ≡ ... 0 mod px−k−1 ≡ 0 mod px−k+1 ≡ ... 0 mod px−2 ≡ − 1 mod q ≡ + 1 mod t.

This causes (μ ± px−k ) to become totatives (beta decay).

Each instance of beta decay at μ ± (px−k ) produces a smaller Goldbach Solar System, since its
new diameter is the sum of two equal prime numbers, 2px−k . This new Solar System is nested within the
original larger Solar System of diameter 2 px−1 and contains a total of three nuclei. Two of these nuclei
have a length ( px − px−k ) and are adjoined to either terminal of the interior nucleus of length 2px−k .

If a fusion event occurs at either μ + (px−k ) or μ − (px−k ) , then one the exterior nuclei fuses with
the interior nucleus to create a new nucleus of length px−1 + px−k , which is a proper Goldbach pairing
which sums to an even integer. However we will not address the Goldbach Conjecture in this volume (The
Goldbach Conjecture is the sole focus of Volume Two), since we do not need to prove it in order to solve
the Andrica Conjecture.

At the heart of the Goldbach Conjecture lays the assumption (by contradiction) that if the Goldbach Conjecture were false
for a particular even number, then there would exist a singularity for every integer contained inside of gap whose length was also
that even number; since the zeta function would return both “null,” and “some prime in P,” stating that no prime number in P may
divide any integer inside such a gap, and yet at least one prime number in P must divide it in order to complete the gap, but any
prime you choose will be spit out! This null singularity creates the “white hole paradox,” which falsfies the statement: “there exists an
even number which is not the sum of two primes,” and thus proving the Goldbach Conjecture. Sadly this must wait for Volume 2
since the proof is very intense and requires that the Strong Andrica, Cramer and Firoozbakht’s Theorems are proven in advance.

/
Definition 3.1.17 Tidal Fusion

When a fusion event occurs at either μ + (px−k ) or μ − (px−k ) inside a Perturbed Goldbach Solar
System of radius px−1 . One of the exterior nuclei fuses with the interior nucleus to create a new nucleus
of length px−1 + px−k .

Definition 3.1.18 Barycenter (type of anchor), Δ

Given a Perturbed Goldbach Solar System of radius px−1 , the Barycenter is located at the center
of the star (the centrum of the original unperturbed Goldbach Star). The Barycenter, Δ , is the integer for
which we ascribe our system of congruences, as we do for centrum ( μ ) for general nuclei.

Definition 3.1.19 Tidal Core

The range of integers Δ − px−k to Δ + px−k .

Definition 3.1.20 Tidal System, Left and Right Terminals.

The range of integers Δ − px−k to Δ + px−1 ; or, Δ − px−1 to Δ + px−k . These bounds are the left and
right terminals of the Tidal System. A Tidal System is also a complete nucleus.

Definition 3.1.21 Positron, Electron, Proton, Antiproton, Neutron, Quark, Meson

For any p > √ px−k , p ≤ px−1 .


Positron: Any belligerent prime that divides an integer left of the left terminal.
Electron: Any belligerent prime that divides an integer right of the right terminal.
Proton: The first instance from the left of Δ that ξ (z) = p within the Tidal Core.
Antiproton: The first instance from the right of Δ that ξ (z) = p within the Tidal Core.
Meson: Any harmonious prime that divides Δ , dividing two integers equidistant from the
barycenter within the tidal core forming a bracketed prime.
Neutron: Future instances that ξ (z) = p within the Tidal Core, beyond the proton and antiproton,
and also such future instances contained within the remaining portion of the Tidal System.

For any q < √ px−k , where Q is set all of such primes, and L = Px - Q
Quark: All instances where ξ (z) = q within the Tidal System, except those instances where z can
be divided by some prime in L, which are neutrons.

Definition 3.1.22 Alpha Decay

Any process that converts a proton or antiproton or meson into an electron or positron or a
totative(s).

/
Let’s now take a look at Goldbach Solar System with a radius of 19.

The Quarks are colored in blue, since they can be divided by either 2 or 3, but cannot be divided
by any prime greater than 3. The protons are colored orange and the neutrons and colored purple. The
two planets are colored green at the terminals of the star.

This particular Solar System is a single nucleus of length 38 since it hasn’t been perturbed.
Δ ≡ 0 mod (2, 3, 5, 7, 11, 13, 17) ≡ + 1 mod 19 ≡ − 1 mod 23

Now let’s perturb this system. We’re going to perform a Baryogenesis by removing the prime
number 11 from the sieving set and replacing it 53, and then set the remainder of this modulus to -11.

The meson of 11 undergoes alpha decay since the prime number 11 was destroyed, creating two
totatives at μ ± 11 . Then a fusion occurs at μ + 11 since we position 53 to divide it. The mesons of 13 and
17 also experience alpha decay from this perturbation and become electrons (colored black).

Perturbed Goldbach Solar System with a Tidal length of (11+19) = 30, with an interior planet
located at (μ − 11) for the Perforated Set: {2,3,5,7,59,13,17,19,23} against P9.

Δ ≡ 0 mod (2, 3, 5, 7, 13, 17) ≡ + 1 mod 19 ≡ − 1 mod 23 ≡ − 11 mod 59

/
3.2 The Strong Cramér Conjecture

Definition 3.2.1 The Strong Cramer Conjecture, Pressure Limit

Let Pn be a set of consecutive primes starting from 2 and let |P| = n . Let |Q| be the quartic set in
respect to Pn, and let |Q| = 𝝆. Let V = Pn - Q, |V| = n − �� =. The
c variable c is known as the pressure
limit.

Let r = ⊕ (μ) , such that r is the count of belligerent primes for any nucleus in the Galaxy of Pn.

The maximum nucleic length of μ with belligerence r is equal to:

r ≤ c; Ψmax (μ) = pn−a + pn−b ; a + b = r; a ≥ b; a ≤ r − 1


whose tidal system contains a tidal core [ Δ − px−r to Δ + px−r ], such that:

Δ ≡ 0 mod pi , ∀i ≤ n − r , with two of the belligerent primes dividing Δ ± 1 , and with any remaining
belligerent primes dividing a set of consecutive planets (totatives) on either side of the tidal core, or on
both sides of the tidal core.

The explicit evaluations of a, b vary for any particular n , except for the case of r = 2 and r = 3 ,
whose (a, b) values are always a = 1, b = 1 and a = 2, b = 1 respectively. However, for belligerences of
four of greater (r ≥ 4) . there a multiple positive integer pairs which can sum to r .

The variable c represents the concept of “pressure limit.” Any Perturbed Goldbach System that
has neither redundant nor absent primes has a tidal system length varying between:

r = 2 ; ψ = 2pn−1 (exact, since a = 1, b = 1 )


and

r = c ; ψ ≈ 2pn 1 , if r is even, or, ψ ≈ (pn r+1 + pn r−1 ) , if r is odd.


−2r − 2 − 2
(approximate, since there are multiple positive integer pairs that add to c ).
End of Definition 3.1.1

In these systems all primes in Pn are active. Once the maximum pressure is reached, c , the star
must either go into nova and expand from 2px−1 to 2px+k (if we force the pressure to increase), or begin to
disintegrate its quarks (perturb the Quartic Primes) and collapse from 2px−1 to 2px−k . In either scenario the
original star is destroyed and is reformed through baryogenesis in a new galaxy, since a nova requires an
addition to the set of generating primes, and a collapse requires a subtraction, absence or redundancy
from the set of generating primes.

/
Also, since the Barycenter of such a system is a highly divisible integer, such systems cannot
i=k
occur for a barycenter Δ ≤ pn 2 , since ∏ pi ; k ≥ ρ grows exponentially in respect to pn 2 , and therefore
i =1
such a Barycenter cannot occur in the local quadratic interval from pn−1 2 to pn 2 when applying the Sieve
of Eratosthenes over that range by sieving with the primes in Pn. Therefore the true limit between
consecutive primes in the quadratic interval must collapse rapidly for increasing n .

/
However, the moment we attempt to perturb the locations of Quartic Primes, we begin a process
that no longer disintegrates mesons (beta decay), but rather one that begins to disintegrate the quarks!
For instance, if we perturb the locations of 5, we create totatives at Δ ± 5 and Δ ± 25 . This leaves a total
of four totatives that must now be divided by four belligerent primes. Although the Quartic Bracket
Theorem (Chapter 3) will eventually remedy the seeming chaos of these perturbations, the sieving action
of the prime number 5 (and/or other quartic primes) will no longer occur in a symmetric manner over the
centrum or barycenter(s).

If we continue to perturb a succession of quartic primes then our tidal system rapidly collapses,
because we don’t have enough primes in Pn that can be both active and belligerent. If we begin to
collapse our tidal system beyond ψ ≈ 2pn− 1 r then certain primes will be forced into the nugatory set (which
2

is passive form of baryogenesis), and the value of c will decrease linearly with the value of |F|, which is
the size of the Nugatory Set.

We can see this with a star (nucleus) of length 8. The maximum and only value of c is 2. Any
other prime in the sieving set must be redundant or absent. If we force c to increase, then this star goes
nova and nucleus is forced to expand. The length that it assumes is based on both the pressure and
count of active primes. In the simplest case this nucleus would expand to a length of 12, which is the first
gap that has three belligerent primes.

λ 1 0 0 0 μ 0 0 0 1

ξ ∅ 2 px 2 3 2 py 2 ∅

Soon I shall define a gluon, which is a subset of the Quartic Primes. The product of the Gluons
must be less than the square of px−1 . The Gluons form a cyclic constellation inside of gaps, which allow
us to employ the Class C2 and D1 Formulas (partial and complete mutable cyclic presets). In the above
nucleus of length 8, both 2 and 3 are gluons.

The gluons cannot be significantly perturbed by a mere repositioning (due to their rapid cyclic
nature), but removing them entirely through baryogenesis is nothing short of catastrophic.

The gluons capture the essence of “sub-prime” numbers in a dynamic and well defined manner;
gluons would perhaps satisfy questions and disagreements (such as those in the url link below), which
occur all the time in textbooks, classrooms, lecture halls, conferences, conservations and even within our
own heads:

https://www.quora.com/Should-2-and-3-be-considered-sub-primes-because-they-are-only-prime-
numbers-because-they-are-too-small-to-have-factors-other-than-1-and-themselves

/
Definition 3.1.2 The Gluon Set, σ

i=y
2
∃py ∈ Px such that ∏ pi < (px−1 ) .
i=1
Then let G ⊂ Px such that G = { 2, 3, 5...py }
G is said to be the Gluon Set in respect to px .
Let |G| = σ
Let’s look at a length of 14 (remember that Goldbach Solar Systems must have a radius that is a
prime). We will look at the quarks only, Q = {2,3}

λ 1 0 1 0 0 0 1 Δ 1 0 0 0 1 0 1

ξ ∅ 2 ∅ 2 3 2 ∅ 2 ∅ 2 3 2 ∅ 2 ∅

The maximum number of belligerent primes is equal to 4.

x y z
Now let’s try a length of 58 = (2)(29), Q = {2,3,5}. Quarks must in the form: k = 2 3 5 ; k < 29.
x y z
There are 17 integers in the form 2 3 5 between 2 and 28. Since an unperturbed Goldbach Solar
System is symmetric, we need only look at the right side from the barycenter and then double the count of
unsieved integers to account for the left side.

0 1 2 3 4 5 6 7 8 9 0 1 2 3 4 5 6 7 8 9 0 1 2 3 4 5 6 7 8 9

2 2 3 2 5 2 2 3 2 2 3 2 2 2 2 5 3 2 ∅

The integers that remain are {+1, +7, +11, +13, +14, +17, +19, +21, +22, +23, +26}, and thus
c = 2(11) = 22 , which is the worst case scenario for a gap of 58 using a Perforated Set that expelled all its
Quartic Primes, but retained it’s non-quartic elements. The pressure limit c must be increased drastically
in order to complete this nucleus with such a Perforated Set. Let’s see what this set would be (definition of
a Perforated Set is repeated below):

Perforated Set:
Given the sets Pk and Sk, then:
Pk ⋂ Sk = Tm; m ≥ 1

Then let:
Sk - Tm = Vr ; r ≥ (k − m)

= Tm ⋃ Vr is said to be a Perforated Set against Pk.


r
Then the set T m

3 11 34 34
P 1 -P 4 = T8 = {7, 11, 13, 17, 19, 23, 29, 31}, r = 2(17) = 34 , T 8 ; |T 8 | = 42.

Increasing the amount of primes in the generating set from 11 to 42 is massive baryogenesis. As
you can see, expelling the quartic primes (and therefore disintegrating the quarks) is a catastrophic event.

/
If we include the neutrons then we get {+1, +7, +11, +13, +17,+19, +23}, thus c = (2)7 = 14 , which
is the worst case scenario (maximum pressure) for a gap of 58 using a Fractured Set that had retained all
its Quartic Primes, which also happens to be a stable Perturbed Goldbach Tidal System.

0 1 2 3 4 5 6 7 8 9 0 1 2 3 4 5 6 7 8 9 0 1 2 3 4 5 6 7 8 9

2 2 3 2 5 2 2 3 2 2 2 3 2 2 2 3 2 2 5 2 3 2 ∅

The above Tidal System requires 14 belligerent primes to complete a gap of 58 (nuclear length
of 58) with Q = {2,3,5}. Since |Q| = ⍴ = 3, and c = 14 , then we need the Fractured Set:

14
W 3 = {2,3,5, s1 , s2 ...s14 } to complete this gap, which is a total of 17 primes.

μ ≡ 0 mod 2, 3, 5 ≡ ± [1, 7, 11, 13, 17, 19, 23] mod (s1 , s2 ...s14 }

We can of course reduce this to 2 belligerent primes by taking the Fractured Set:

2
W 9 = {2,3,5,7,11,13,17,19,23, w10 , w11 } , which contains 11 primes.

μ ≡ 0 mod 2, 3, 5, 7, 11, 13, 17, 19, 23 ≡ ± [1] mod (w10 , w11 } .

2 1
Thus the sets W 9 ,W 10 and P 11 are sturdy enough to build gaps of 58 by activating all of their
z
11 primes, but the set W3 cannot for z ≤ 13 , since a Goldbach Solar System of 29 leaves 14 totatives
when it’s core consists only of the quartic primes {2,3,5}.

/
Later in this volume we will use a particular lower bound to count quarks, let’s take the quartic set
Q = {2,3,5,7}, |Q| = 4 = ρ

Given k = 2x 3y 5z 7t ; k < 350, what is the lower bound on the number of integers that satisfy k ?

z = F loor[log 7 (350)] = 3 , then we may choose any three primes in {2,3,5,7}, including repetitions,
in which the order does not matter, and multiply them together, such as {2,2,3} or {5,5,7} or {7,3,5}.

Thankfully the general solution to resolving this multiset count is already well known. The number
of multisets of size a from a set of b elements is given by the formula:

(a+b−1)! (z + ρ −1)! (3+4−1)! 6! 720


(a!)(b−1)!
= (z)!(ρ−1)!
= (3)!(4−1)!
= 2 = 36
= 20
(3!)

Since our quartic set will always terminate with a prime that is only slightly less than the square
root of a particular number, then we set z = 2 , and our lower bound on quarks within a Goldbach Solar
System becomes (on either side of the barycenter):

(z + ρ −1)! (2 + ρ −1)! 1 (ρ +1)!


(z)!(ρ−1)!
= (2)!(ρ−1)!
= 2 (ρ−1)!
= 12 (ρ)(ρ + 1) = 12 (ρ2 + ρ) .

Since we must double this entire count to include all quarks on both sides of the barycenter, we
get (2)( 12 )(ρ2
+ ρ) = (ρ2 + ρ) , as our lower bound. This bound is strong enough for Volume I.

It is possible to improve this lower bound by isolating the values of log q (pk−1 ) for each prime in
the Quartic Set (where pk−1 was the prime used to generate the Quartic Set) and then using successively
higher values for z and subtracting away the overlap from results obtained from the lower values of z , but
such a strong lower bound is not needed in this volume. However I will give a brief demonstration:

If ∃py ∈ Px such that py < (px−1 ) / and py+1 > (px−1 ) / ,


(1 k) (1 k)

Then let Qk ⊂ Px and ∀q ∈ Qk, q < √ px−1 , such that Qk = { 2, 3, 5...py }


Let | Qk| = ⍴, then F loor[log qρ (px−1 )] = k , now set z = k , then:

(2 + ρ −1)! 1 (ρ +1)!
z = 2; (2)!(ρ−1)!
= 2 (ρ−1)!
= 21 (ρ)(ρ + 1) = 21 (ρ2 + ρ) .

(3 + ρ −1)! 1 (ρ +2)!
z = 3; (3)!(ρ−1)!
= 6 (ρ−1)!
= 61 (ρ)(ρ + 1)(ρ + 2) = 61 (ρ3 + 3ρ2 + 2ρ) .

(k + ρ −1)! 1 (ρ +2)! 1 1
z = k; (k)!(ρ−1)!
= k! (ρ−1)!
= k!
(ρ)(ρ + 1)...(ρ + k − 1) = k!
([Stirling Expansion on ρ) .
Sum of Stirling Coefficients = k!
http://mathworld.wolfram.com/StirlingNumberoftheFirstKind.html

/
Let’s see what the Strong Cramer Conjecture would state about the maximal gaps of the set
P12 = {2,3,5,7,11,13,17,19,23,29,31,37}, Q = {2,3,5}. G = {2,3,5}, where G is the Gluon set.

|Q| = 3. Let V = Pn - Q, |V| = 12 − �� =, V


9 = {7,11...37}

r ≤ c; Ψmax (μ) = pn−a + pn−b ; a + b = r; a ≥ b; a ≤ r − 1


r = 2; Ψmax (μ) = pn−1 + pn−1 = 62
r = 3; Ψmax (μ) = pn−1 + pn−2 = 60
r = 4; Ψmax (μ) = pn−2 + pn−2 or pn−1 + pn−3 = 58 or 54; therefore 58.
r = 5; Ψmax (μ) = pn−2 + pn−3 or pn−1 + pn−4 = 52 or 50; therefore 52.
r = 6; Ψmax (μ) = pn−3 + pn−3 or * pn−2 + pn−4 * or pn−1 + pn−5 = 46, 48 or 48; ∴ 48.
r = 7; Ψmax (μ) = pn−3 + pn−4 or * pn−2 + pn−5 * or pn−1 + pn−6 = 42, 46 or 44; ∴ 46.
r = 8; Ψmax (μ) = pn−4 + pn−4 or pn−3 + pn−5 or * pn−2 + pn−6 * or pn−1 + pn−7 = 38, 40, 42 or 42; ∴ 42.
r = 9; Ψmax (μ) = pn−4 + pn−5 or pn−3 + pn−6 or * pn−2 + pn−7 * or pn−1 + pn−8 = 36, 36, 40 or 38; ∴ 40.

I placed asterisks around the sums that were maximal in each iteration after r = 5 . Notice that
each these start with the summand (pn−2 ) . This initial occurrence has a rebound effect in the iterations
that follow, which prevents a future iteration from having a larger maximum than any iteration that
precedes it. Thus it is impossible for the maximum of r + 1 to exceed the maximum of r, since the
maximum of r + 1 is always nested within the maximum of r.

This rebound can be broken for sufficiently large n , but only in a manner that further decreases
the maximum, and then proceeds to initiate a new rebound. Such an event is called a rupture.

There is another important observation: The nucleus of length 36 generated by r = 9 is the


smallest nucleus that can be created by P9 when all of its primes are active without perturbing the quartic
primes.

On a final note of observation, none of the gaps from r = 1 to r = 7 can actually occur between
the squares of 31 and 37, which are 961 and 1369. This is because their barycenters are divisible by the
product of the first five primes {2,3,5,7,11}, which is equal to 2310.

However, the barycenter of r = 8 is divisible by 210 and the final barycenter of r = 9 is divisible
by 30, which means that it is possible for the systems of congruences for these barycenters to be
resolved within the quadratic interval of the Sieve of Eratosthenes, and thus gaps of length 36 to 42 are
possible.

Let’s take a look at the primes between the squares of 31 and 37, which starts with the 163rd
prime and ends with the 219th prime. The largest gap that occurs is 34, between the 217th and 218th
primes, which are 1327 and 1361. Since this gap is just shy of 36 and the Quartic Primes are not
perturbed, then there will be at least one nugatory prime. A gap of 34 cannot contain the pressure of a
fully active P12 set with all of its Quartic Primes dividing the barycenter!

/
Let’s see if this prediction holds. The barycenter occurs at 1350, since it can be divided by 30.

Δ ≡ 0 mod 2, 3, 5 ≡− 1 mod 7 ≡− 3 mod 11 ≡ (− 2; + 11) mod 13 ≡ 7mod 17 ≡


≡ 1 mod 19 ≡− 7 mod 23 ≡− 13 mod 29 ≡ (− 14; + 17) mod 31 ≡ 18 mod 37

Left Side (1327 to 1349):


27 28 29 30 31 32 33 34 35 36 37 38 39 40 41 42 43 44 45 46 47 48 49

∅ 2 3 2 11 2 31 2 3 2 7 2 13 2 3 2 17 2 5 2 3 2 19

Right Side(1350 to 1361): The primes 29 and 37 are absent and therefore Nugatory!
50 51 52 53 54 55 56 57 58 59 60 61 63 65 67 69 71 73

2 7 2 3 2 5 2 23 2 3 2 ∅ 29 37 ... ∅

F = {29,37}; B = {11,13,17,19, 23, 31}; H = {2,3,5,7}; D = H U B = {2,3,5,7,11,13,17,19,23,31}

1
The Active Set D is a Diminished Fractured Set of P12. D = W 9 . This set can form a gap of
(2)(23). Therefore a Goldbach Solar System of radius 23 would be better to represent this, which would
range from the integers 1327 to 1373.

It is also no coincidence that 1373 turned out to be a prime number. Many of the larger primes in
P12 must be nugatory for sufficient duration following 1361, and demonstrates why unusually large nuclei
relative to Pn are always followed by a rapid burst of smaller nuclei; this is nature’s way of ensuring the
integrity of the Prime Number Theorem! It’s also nature’s way of enforcing the Goldbach Conjecture as
well you will see in Volume II.

/
Let’s look at the consecutive primes 113 and 127, and then the primes that are immediately
adjacent to them from 101 to 139. √139 ⇒ Pn = {2,3,5,7,11}

Primes: 101, 103, 107, 109, 113


101 02 03 04 05 06 07 08 09 10 11 12 13 14 15 16 17 18 19 120

∅ ∅ ∅ ∅ ∅ 5 3 7 2

The integers in the next table are reversed from greatest to least to compare the prime positions.

Primes: 139, 137, [133 composite], 131, 127


139 38 37 36 35 34 33 32 31 30 29 28 27 26 25 24 23 22 21 120

∅ ∅ 7 ∅ ∅ 5 3 11 2

Since the nucleus of length 14 bounded by the primes 113 and 127 contains two consecutive
odd instances (odd multiples of 5) of ξ (k) = 5, then we know the next three consecutive odd instances in
either direction will be redundant, such that either 2 or 3 will divide them. Because of this 5 must be
nugatory for the next 19 integers in the respective directions from the last active instance. The prime
number 5 cannot act between the integers 96 and 114, nor can it act between the integers 126 and 144.

Thus the set {2,3,5,7,11} has been reduced to a fractured set, {2,3,7,11} within the range of 96 to
114 and 126 to 144.

However, the prime number 11 also cannot act for at least one adjacent odd instance from either
side of it’s belligerent position, and thus the next 21 integers from either side of its single act are also
devoid of its influence. These would be the integers 100 to 120 and 122 to 142. Thus we have further
reduced the sieving set from {2,3,7,11} to {2,3,7} between the integers 100 and 142.

Finally the prime number 7 can only activate once in the range of 99 to 188, or, 120 to 140;
however, it cannot activate in both ranges simultaneously, since 3 must divide one of these odd multiples
of 7. The Dirichlet String [1010001010001...] for the primes {2,3} applies not only to consecutive integers,
but also to consecutives integers in an arithmetic sequence (whose interverals are not divisible by 2 or 3).

Thus 7 will activate one side...it must activate (in this case it activates on the right side at 133),
but it cannot activate on the other side. Thenceforth the sieving set shall be reduced to {2,3} on one side
and {2,3,7} on the other. We check the integer (119-14) = 105 and determine that it is divisible by 3,
immediately implying that (119 +14) = 133 is not, and therefore 133 is prime, and the rest of primes
between 100 and 140 are forced to appear due to the Gluonic Constellation of {2,3}, whose only gaps
are alterations of 2 and 4.

Lo! The integers {113,109,107,103,101} must be prime since we start the reversed Gluon
constellation at 113 (with the factors of the barycenter 120 having already provided sufficient information
to determine that 109 is the preceding prime instead of 111). By reflecting these values over the
barycenter of 120, we know that {139, 137, [133 composite], 131, 127} are also forced to be prime (we
already had sufficient information to know that 133 was the composite number due to our divisibility test
on 105).

/
In Volume 2, when a sieving set of primes is reduced to any subset of the Gluons over any
interval, it induces Gluonic Genesis, which means a forced sequence of very small consecutive gaps.
Once the Goldbach Conjecture is completed in Volume 2 we move onto the K-tuple Conjecture. The
K-Tuple Conjecture is redefined and strengthened as:

Gluonic Genesis must occur within the Sieve of Eratosthenes (the full quadratic interval from 1 to pn 2 ) for
every possible subset of Gluons.

Once the above is proven, we can then invoke the Goldbach Conjecture which forces every
possible gap to exist between 2 and 2pσ−k , since the perturbations, novas and collapses of Gluonic
Goldbach-Stars produce tidal lengths whose measures include every even integer up to 2pσ−k .

The variable k will not be defined in Volume 1, since explaining the formula (which converges on lower bound) would take
a great deal of time. You’ll just have to wait for Volume 2!

If every possible subset of G has an instance of Gluonic Genesis between 1 and pn 2 , and every
even nucleic length from 2 to 2pσ−k must be produced by the constellations of G, then the K-tuple
conjecture must be true, so long as we can demonstrate the final step: That each Gluonic Constellation
occurs at least once between 1 and 12 pn 2 ; and at least once between 12 pn 2 and pn 2 .

The solution to this final step eludes me to this day, and thus no complete proof of the K-tuple
Conjecture has been discovered. However, at least I managed to prove that there exists at least one gap
of any particular size (even size). I do sincerely hope that someone will complete the proof in Volume 2,
I have spent too many years failing to achieve it.

/
It is now time to directly address the Cramer and Firoozbakht conjectures, and show the stronger
elementary versions. Whereas it only took two pages to strengthen the Andrica Conjecture and restate it
in elementary language, the same cannot be said for the other two conjectures.

First, let us compare the growth rates of Ω (Pn-2) to pn 2 to Q

Sloan A002110 - OEIS; https://oeis.org/A002110 vs. pn 2 vs. Q


n: Ω (Pn) n: pn 2 Q
0: 2, 0: 4 {}
1: 6, 1: 9 {}
2: 30, 2: 25 {2}
3: 210, 3: 49 {2}
4: 2310, 4: 121 {2,3}
5: 30030, 5: 169 {2,3}
6: 510510, 6: 189 {2,3}
7: 9699690, 7: 361 {2,3}
8: 223092870, 8: 529 {2,3}
9: 6469693230, 9: 841 {2,3,5}
10: 200560490130, 10: 961 {2,3,5}
11: 7420738134810, 11: 1369 {2,3,5}
12: 304250263527210, 12: 1681 {2,3,5}

At n = 4 , Ω (Pn-2) < pn 2 ; however for n ≥ 5 , Ω (Pn-2) > pn 2 . Thus the maximal gap of 2pn−1
(Andrica) can no longer occur inside the Sieve of Eratosthenes, since the centrum of a such a gap
corresponds to a non-zero multiple of Ω (Pn-2) .

We then move onto the Strong Cramer Conjecture, which halts at the Quartic Primes. At what
value of n does the growth of Ω (Q) overtake pn 2 ?

If Q = {2,3,5,7}, then pn > 49 ∴ pn ≥ 53 ; Ω (Q) = 210; 532 = 2809


If Q = {2,3,5,7, 11}, then pn > 121 ∴ pn ≥ 127 ; Ω (Q) = 2310; 1272 = 16129
If Q = {2,3,5,7, 11,13}, then pn > 169 ∴ p31 ≥ 173 ; Ω (Q) = 30030; 1732 = 29, 929 . Overtakes.

If we look at the primes between the squares of 173 and 179, we find an unusually large gap
between 31397 and 31469, a total difference of 72. Let’s take a look!

/
P = {2,3,5...179}, n = 41
1
2
(31397 + 31469) = 31433 = μ = (17)(432 ); 31440 = Δ ≡ 0 mod (2, 3, 5)

31469 − 31440 = 29; 31440 − 31397 = 43 , therefore we have a Perturbed Goldbach Star whose inner
radius is 29, and has its:

Positrons located at Δ − (29, 31, 37, 41)


Protons located at Δ − (1, 7, 11, 13, 17, 19, 23)
Antiprotons located at Δ + (1, 7, 11, 13, 17, 19, 23)
a a a
Gluons located at Δ ± k , ∀k = (2 1 ) (3 2 ) (5 3 ); ∣k∣ < 43, {2, 3, 5} = G since 𝚷(G) = 30, 30 ≤ Ψ(μ)
Quarks MAY be located at Δ ± z k , z =/ (2 1 ) 3
b
( ) (5
b2 b3
) , ∣zk∣ < 43 , k = 1 or k = (2a1 ) (3a2 ) (5a3 )

ξ (Δ − 1) = 143; ξ (Δ + 1) = 23 ξ (Δ − 17) = 7; ξ (Δ + 17) = 83


ξ (Δ − 7) = 17; ξ (Δ + 7) = 13 ξ (Δ − 19) = 13; ξ (Δ + 19) = 163
ξ (Δ − 11) = 53; ξ (Δ + 11) = 7 ξ (Δ − 23) = 89; ξ (Δ + 23) = 73
ξ (Δ − 13) = 11; ξ (Δ + 13) = 71

In accordance with the above, there exists no mesons in respect to any prime greater than 5,
since none divide the barycenter, thus our mesons are located at Δ ± (2, 3, 5) . Bear in mind that both 7
and 13 are bracketed within the tidal core as the above data shows.

Positron zeta yield:


ξ (Δ − 29) = 101; ξ (Δ − 31) = 7 ; ξ (Δ − 37) = 31 ; ξ (Δ − 41) = 17 .

The prime number 17 is therefore bracketed within the overall nucleus, since it also divides (Δ − 7) .

Let’s take a look at our Active Set, D:


D = {2,3,5,7,11,13,17,23,31,53,71,73,83,89,101,143,163}, |D| = 17 = n2

p17 = 59 , 2p16 = 2(53) = 106 . Since 106 > 72, let's now examine the Belligerent Set.

B = D - H, H = {2,3,5,7,13,17}; B = {11,23,31,53,71,73,83,89,101,143,163}, |B| = 11 = r .

Using this value for r , let’s see what the Strong Cramer Conjecture predicts (using the odd formula).

ψ ≈ (pn r+1 + pn r+1 ) , if r is odd, we will use n 2 in this formula, the size of the active set D.
− 2 − 2

ψ = 72 ≈ (p17− 11+1 + p17− 11−1 ) = (p11 + p12 ) = (31 + 37) = 68 .


2 2

It turns out that a gap of size 72 is not that large at all in respect to where it first occurred, it’s
actually much much smaller than the maximal span of P = {2,3,5...179}, n = 41 , which is 346 (remember
that F = P - D, and therefore |F| = 24, that’s a lot of unused nugatory primes, and only three primes
symmetrical divide the barycenter, {2,3,5} !!!).

/
Next we ask ourselves, assume that every prime in P41 was active, what is the smallest gap we
could create? They can’t all be belligerent, the gluons {2,3,5} will sieve absolutely multiple times, the
Quartic Set {2,3,5,7,11,13} is also an issue here.

In the following chapters we will propose and prove Quartic Sovereignty Theorem, which forces
ξ (k) to evaluate to a particular quartic prime a minimum of four times within any nucleus whose length is
greater than twice the square of that particular quartic prime.

ρ = 6 = |Q| = {2,3,5,7,11,13}, n = 41, c = n − ρ = 35 , c is odd, therefore, r = c :

ψ ≈ (pn r+1 + pn r−1 ) = p41−18 + p41−17 = p23 + p24 = 83 + 89 = 172


− 2 − 2

This would correspond to the system of congruences:


μ ≡ 0 mod (2, 3, 5, 7, 11, 13) , with all of the remaining primes in P41 acting in a belligerent manner.

The prime number 7 must actively sieve a minimum of four integers in such a gap, since
2
2(7 ) < 172 , in accordance with Quartic Sovereignty. However, the primes {11,13} are free of this
mandate, so it is possible for us to make an even smaller gap by relegating {11,13} to the belligerent set,
which induces a collapsar, since the pressure limit, c , would be exceeded. The approximate radius of this
collapsar would be:

p23 + p24 = 83 + 89 = 172 → p21 + p22 = 73 + 79 = 152 .


This transformation reduces the subscripts by two, since two quartic primes were relegated to
belligerence, and these quartic primes, {11,13} had sieved the original nucleus a minimum of two times
since they were mesons (or at the very least bracketed).

Let’s now examine p21 + p22 = 73 + 79 = 152 . We shall write it as pa + pb , a + b = 43 . Which


combination of a + b gives us the least value?

a = 14, 21; b = 29, 22 , pa + pb = 152 = (43+109) = (73+79). Since 152 > 2(72 ) , we cannot relegate the
prime number 7 to belligerent status without causing a massive collapse, since such an act would leave
four unsieved integers. However, let us indeed do such a thing!

p21 + p22 = 83 + 89 = 172 → p19 + p20 = 67 + 71 = 138 .


Except, there’s a problem... 138 > 2(72 ) , which means 7 is still not belligerent! Since nothing in
this section (Section 3) actually contains any proofs, we’re just going to leave it here. 152 is the smallest
gap that can be generated by P41 when it’s fully active, any attempt to relegate 2, 3, 5 or 7 to belligerence
would create white hole singularities (since ξ (k) = 7 and ∅ simultaneously) and therefore cannot be done.

This sure makes the gap of 72 between 31397 and 31469 look quite small given the above! Bear
in mind that the prime number 7 sieves that gap three times, and had the gap been of length 98, it would
have been forced to sieve it four times by the virtue of Quartic Sovereignty. Also any gap of length 42
must also contain a bracket of 7, since it is not possible to sieve six consecutive multiples of 7 using the
primes {2,3,5}, and any prime greater than 7 cannot sieve a multiple of 7, since it would be redundant.

/
Now we turn our attention to the actual Cramer Conjecture:

Now we look at a weaker form:

The results of Westzynthus are not of any interest here, that is the domain of analytical insanity,
and will never bear any useful fruit in the foreseeable future; we are concerned with the preceding
passage:

pk+1 − pk = O(ln(pk )√
2
pk ) , or more generally
pk+1 − pk < O√
R
pk , where R = 2 is the Andrica Conjecture.

However, given the nature of Section III, let’s restate this as:

Let pn be the greatest prime less than √pk+1 , and let Q be the quartic set of Pn, then:

pk+1 − pk ≤ 2(pn 1 ) , where c = n − ρ , ρ = |Q|, pn ≥ 173


−2c
pk+1 − pk ≤ (pn c+1 + pn c−1 ) if c is odd.
− 2 − 2

/
Let’s take a look at this graph of formulas that predict maximal gaps:

Although the Wolf formula is the most attuned of the above predictions, it is clearly in severe error
since it often dives under the actual maximal gaps. The Cramer and Firoozbakht models however seem
to diverge at a constant rate (logarithmically), and also succeed at remaining above the actual record.
Let’s take a look at Firoozbakt’s model:

/
Let’s replace n with k in the Firoozbakht model as to not confuse ourselves:

√ pk+1 <
√pk .
k+1 k

Let’s convert the above radical horror into something meaningful that we can actually count in
respect to the Sieve of Eratosthenes.

Let pn be the greatest prime less than √pk+1 , and let G be the Gluonic set of Pn. Now create a barycenter
n−σ
that can be divided by every prime in G, Δ ≡ 0 mod g, ∀g . We now convert Pn into the fractured set P σ
where σ = |G|, G = P𝞂 and G′ = P𝞂+1

n σ
We now dictate that all primes in V, V = P σ− - P𝞂, are greater than Ω(G′), forcing all of them to
be belligerent, since we are also forcing them all to be active. Can the primes in V span a gap longer than
Ω(G) itself under these circumstances?

i=σ
The amount of unsieved integers in Ω(G) is precisely ɸ Ω(G). Can (n − σ ) > ∏(g i − 1) for
i=1
sufficiently large n ?

pn
n≈ ln(pn ) ,
Ω(G) < pn < Ω(G ⋃ {pσ+1 })
Let G′ = { G ⋃ {pσ+1 }} , then:

𝝈, pσ ɸ Ω(G) Ω(G) Ω(G′)


a= n≈ ln(Ω(G))
b=n≈ ln(Ω(G′))

1; 2 1 2.88 3.34

2; 3 2 3.34 8.82

3; 5 8 8.82 39.27

4; 7 48 39 298

5; 11 480 298 2912

6; 13 5760 2912 38482

7; 17 92,160 38482 602,929

8; 19 1,658,880 602,929 11,605,458

9; 23 36,495,360 11,605,458 286,391,334

10: 29 1,021,870,080 286,391,334 7,706,638,035

/
𝝈, pσ ɸ Ω(G) Ω(G) Ω(G′)
a=n≈ ln(Ω(G))
b=n≈ ln(Ω(G′))

Skip to 20
25 24 26
20; 71 7.13 x 10 9.05 x 10 6.18x 10
27 26 28
21; 73 5.13 x 10 6.18x 10 4.58x 10

Skip to 30

30; 113 3.62x 1045 2.95x 1044 3.58x 1046

Skip to 50

50; 229 1.94x 1090 9.07x 1088 2.06x 1091

We can continue this table ad infinitum and yield the same result. The most important thing to
observe is that the first column is always greater than the second column (for σ > 4 ).

https://www.wolframalpha.com/input/?i=50%23 , where σ # is the primorial function.


https://www.wolframalpha.com/widgets/gallery/view.jsp?id=c98e7eb508ad306c40cc4d7773a3a69a ,
where the above link calculates the Euler Phi Function on σ # .

As to how we prove that the first column is greater than the second column, is not that
Ω(G)
complicated. The function ɸ Ω(G) counts all of the totatives to Ω(G), whereas n ≈ ln(Ω(G))
is approximating
the number of which those totatives are also prime numbers...since all prime numbers in Ω(G) are also
totatives to Ω(G), then the number of primes cannot exceed the number of totatives!

Thus π (Ω(G)) < ɸ Ω(G) by definition, where π is the prime counting function.

Now returning to the matter of whether or not the set V can span Ω(G), with its gluons dividing a
common barycenter...yes, if |V| ≥ ɸ Ω(G), which the above table demonstrates can happen. However,
the set V can never span Ω(G′), since |V| is bounded between a and b , where b ≈ π (Ω(G′)) < ɸ Ω(G′).

n−σ
pk+1 − pk < ɸ Ω(G)
Ω(G) < 2(pn− 1 c ) < 2 pn−1 , since ɸ nΩ(G)
−σ is the ratio of V to ɸ Ω(G)
2

gi
pk+1 − pk < n ∏( g −1 ) < 2(pn− 1 c ) < 2 pn−1 , since σ is negligible, ɸ Ω(G) = ∏ (gi − 1)
i 2

√pk+1 ∏( gi ) < 2(p 1 ) < 2 p


pk+1 − pk < ln(√pk+1 )g i −1 n− c n−1 , since n = π (pn ) ≈ π √pk+1
2

i=x
pk+1 − pk < √pk+1 (2ln(p )) < 2(p p 1
) < 2 pn−1 , since ∏( ip− ) ≈ 21 ( ln(p
1
)
ln(√pk+1 ) σ n− 12 c i x)
i=1

/
Thus we have (for some sufficiently large n ):
Actual Gap < Strong Firoozbakht < Strong Cramer < Strong Andrica.

pk+1 − pk < n (2ln(pσ )) < 2(pn 1 ) < 2 pn−1 .


−2c

So, using pk+1 ≈ 9 trillion , how would we use the above inequality?
3,000,000
n≈ ln(3,000,000) ≈ 200, 000 , Pn = {2,3,5,7,11... pn };
G = {2,3,5,7,11,13,17}, σ = 7 , since Ω(G) < pn and since Ω(G′) > pn
Q = {2,3,5,7,11,13,17,19,23... pρ }, ρ ≈ 232 ≈ √
3000000 1732
≈ ln(1732) , pρ ≈ 1459
ln(√3000000
2ln(pσ ) = 2ln(17) ≈ 5.66
c = n − ρ ≈ 199, 800

pk+1 − pk < n (2ln(pσ )) < 2(pn 1 ) < 2 pn−1


−2c
pk+1 − pk < (200, 000)(5.66) < 6, 000, 000
pk+1 − pk < 1, 133, 000 < 6, 000, 000 < 2 √9 trillion + ξ )

However, the largest prime gaps in the range of 9 trillion are around 500, which is far smaller than
1.13 million. Thus, something even more sinister must be at play. Although the set P200,000 is fully capable
of creating a nucleus of length 6 million, it fails to create any gap longer than 500 in the local quadratic
interval between the squares of p199,999 and p200,000 .

The malign actor at play is Quartic Sovereignty. If we were to truly create a gap of size 1.13
million the majority of the Quartic Primes in Q could not be belligerent, they would in fact coalesce and
center themselves around multiple barycenters within the same nucleus, with each of these barycenters
being divisible by too many primes in Q, forming giant overlapping webs of nested mesons. Thus the
numeric value of each of these barycenters would be astronomically greater than 9 trillion, and therefore
whatever nucleus is formed by such an act, would be far beyond the local quadratic range of the Sieve of
Eratosthenes.

Let’s take a look at how the sovereignty of the prime number 7 affects a large gap. We will
consider a sequence of 15 consecutive multiples of 7, a range of 98 = 2(72 ) , and attempt to sieve as
many multiples of 7 as possible using {2,3,5}. We know that {2,3,5} cannot create a gap larger than 6, so:

ξ (k) = 7 when k = x ± (21, 25)


k 49 42 35 28 -21 − 14 x−7 x x+7 14 21 28 35 42 49

ξ 3 2 7 2 7 2 3 2 5 2 7 2 7 2 3

λ 0 0 1 0 1 0 0 0 0 0 1 0 1 0 0

[ • • • • ]

/
The four dots enclosed above create a structure known as a Quartic Bracket. Is it possible for a
range of 15 consecutive multiples of 7 to contain less than four integers such that ξ (k) < 7 ?
k 49 42 35 28 -21 − 14 x−7 x x+7 14 21 28 35 42 49

ξ 7 2 5 2 3 2 7 2 7 2 3 2 5 2 7

λ 1 0 0 0 0 0 1 0 1 0 0 0 1 0 1

[ • • • • ]

As of now we don’t have the tools to answer this question, the Theory of Tension is required
before we can address and prove the concept of quartic sovereignty. For now though, you can prove the
individual case of 7 with the primes {2,3,5} by examining the Galaxy of 30 and realizing there is no
sequence of four consecutive totatives to 30 whose bounds differ by more than 14.

If we were to remove 7 from the sieving set, we would now need four more primes to cover the
dots of 7’s quartic bracket. The basic rules of modular arithmetic make it impossible for any prime greater
than 7 to sieve more than one odd multiple of 7 in a range of 98 integers.

Quite simply, using the prime number 11, if 11 sieves ( x + 7) , then the next odd multiple of 7 that
11 could possibly sieve is x + 7 + 2(7)(11) , which is far beyond the range of 98, that’s a span of 154
integers. Thus, to fill (sieve) each of the dots (terminals) of the quartic bracket of 7, we require a minimum
of four distinct prime numbers being forced coalesce around some common barycenter.

Then we have yet another problem, let us take an arithmetic sequence in the form of 7x + y ,
where y ≤ 6 , then once again we can only use the primes {2,3,5} to only sieve a portion of fifteen
consecutives numbers in such a sequence...there will always be a minimum of four ghosts of 7. Between
actual multiples of 7 and their ghosts, the additional primes are coerced into stellar dance. There is more
order among the chaos of primes than you know, in fact...the chaos nearly choreographed.

Thus, whether or not we remove 7 (or any other quartic prime) from the sieving set, the
supremacy of 7 still dominates the structure of any gap we try to create of length 98 or larger. With this in
mind, we will proceed to the final part of this section before proceeding to Section IV (which proposes and
proves the Surjection Theorem). The concluding part of this section will immediately show why the
Surjection Theorem is needed, and thankfully, we already have all the tools and definitions to prove it.

/
3.3 The Strong Andrica, Cramér, Firoozbakht’s Conjectures
And the Proper Andrica, Cramér, Firoozbakht’s Conjectures

A couple of pages earlier the following statement was made (quoted below):

“Thus we have (for some sufficiently large n ):


Actual Gap < Strong Firoozbakht < Strong Cramer < Strong Andrica.

pk+1 − pk < n (2ln(pσ )) < 2(pn− 1 c ) < 2 pn−1 .”


2

The problem with the above conjectures is that they are actually very weak compared to their
traditional counterparts, the actual Cramer and Firoozbakht Conjectures. Both Cramer’s and
Firoozbakht’s models predict a maximal prime gap between 500 to 600 in the range of 9 trillion, yet the
models I presented above put the range between 1.13 and 6 million.

The issue at hand (due to Quartic Sovereignty) is that there is a strict limit on the number of
active primes in the quadratic interval of the Sieve of Eraothenses. Although there’s no limit of the number
of active primes within an entire galaxy, due to Chinese Remainder Theorem (any system of congruences
that we write must exist within the galaxy), the quadratic interval from 1 to pn 2 is too small for any (or
most) of the sieving primes to become active in a single nucleus.

While the Strong Conjectures concern the entire Galaxy, the Proper Forms (next page) concern
the quadratic interval.

/
Definition 3.3.1 The Activity Limit, Eratosthenes Strip (Quadratic Strip).

Let the range of 1 to pn 2 be known as the Eratosthenes Strip (or the quadratic strip).

Let ϓ be the limit of the number of primes in Pn that may be active for any nucleus that resides within the
quadratic strip.

Let D be the active set of primes for any nucleus that resides within the quadratic strip.

Then |D| ≤ ϓ .

We will determine ϓ later on this volume after the Strong Andrica Conjecture is proven.

Definition 3.2.2 The Strong Conjectures; Andrica, Cramer, Firoozbakht.

Strong Andrica Conjecture: tk+1 − tk < 2 pn−1 .


Strong Cramer Conjecture: tk+1 − tk < 2(pn− 1 c ) .
2

Strong Firoozbakht Conjecture: tk+1 − tk < n (2ln(pσ )) .

Where tk+1 − tk is the maximal distance between consecutive totatives in any part of the Galaxy
when all primes in Sn are active in accordance to: The belligerence of the nucleus and the amount of
Quartic Primes in Sn relative to its corresponding set of consecutive primes Pn .

Definition 3.2.3 The Proper Conjectures; Andrica, Cramer, Firoozbakht.

Let D ⊂ Pn, |D| ≤ ϓ , and let G be the gluonic set of P𝚼, σ = |G|
Proper Andrica Conjecture: pk+1 − pk < 2 pϓ−1 .
Proper Cramer Conjecture: pk+1 − pk < 2(pϓ 1 c ) .
−2
Proper Firoozbakht Conjecture: pk+1 − pk < ϓ (2ln(pσ )) .

Where pk+1 − pk is the maximal distance between consecutive prime numbers (totatives) in the quadratic
strip when all primes in D𝚼 are active in accordance to: The belligerence of the nucleus and the amount of
Quartic Primes in D𝚼 relative to its corresponding set of consecutive primes P𝚼 .

Unfortunately, we cannot proceed to prove the Proper Forms until we have first proven the Strong
Forms concerning totatives in general. Also, before we can prove the Strong Forms, we first have to
prove the Surjection Theorem, so let us now proceed to Section IV, the Surjection Theorem!

/
Section IV
The Surjection Theorem

In Section IV we will prove that if the Strong Andrica Conjecture is true (see the beginning of
Section III if you chose to skip all of Section III) for a set a consecutive primes starting from 2 (set P), of
size n, then for any other set of primes, S, whose cardinality is less than or equal to n, that the primes in S
also cannot sieve more than 2pn−1 consecutive integers, and furthermore that there exists a surjection of
every gap generated by S unto the gaps generated by P, but there does not always exist an injection from
the gaps generated by P into the gaps generated by S.

/
Theorem 4.1.1 Stretch Failure for the Andrica Base Case

Assume that the Andrica Base Case is proven up to some n , and let P′n be a proper subset of Pn,
then if we encounter a scenario where P′n must span 2pn−1 , then there is a contradiction, and therefore
the scenario is false.

The phrase “span k” means to sieve k consecutive integers.

Proof:
This is implied by the Proper Superset Span Theorem, Theorem 2.1.3, Page 103.

Let S be a proper subset of Pn.

If δ [Pn] = 2pn−1 , δ [Sw] < 2pn−1 , w < n ∴ δ [P′n] < 2pn−1 , since P′n ⊂ Pn
QED

/
Theorem 4.1.2 The Surjection Theorem

Assuming that if it is proven by brute force that the limit between two consecutive totatives to the
integer Ω[Pn] is precisely 2pn−1 , and that there is only one system of congruences (ignoring the additive
inverse) for the midpoint between two consecutive totatives that yields a difference of 2pn−1 between
them, then the distance between two consecutive totatives of any integer Ω[S] is less than or equal to
2pn−1 , if |S| ≤ n .

Therefore, there exists a surjection from all of the gaps generated by Sn unto all of the gaps
generated by Pn, but there does not always exist an injection from all of the gaps generated Pn by unto all
or some of the gaps generated by Sn.

Proof (contradiction):
Let Pn = {2 , 3 , 5 … pn−1 , pn }, ordered from least to greatest.
Let Sn = { s1 , s2 , s3 … sn−1 , sn }, ordered from least to greatest.

Lemma 4.1.2.1 The Null Intersection


Case 1: Pn and Sn are divorced (empty intersection).

If Pn ∩ Sn = ∅, then si > n, ∀s ∈ Sn, such that the maximal gap that can generated by Sn is precisely
n + 1 and we know that n + 1 ≪ 2pn−1 . We know the maximal gap is (n + 1) due to the Class A1 Formula
on page 73.

Therefore it follows that if it is possible for Sn to span a distance longer than 2pn−1 , then the intersection
betwixt them, Pn ∩ Sn , cannot be empty.

Lemma 4.1.2.2: The Full Intersection


Case 2: Pn and Sn are familial (the intersection is an improper subset of both).

If |Pn ∩ Sn | = n , then Pn = Sn, therefore Sn has the same span because they are the same set.

Therefore it follows that if it is possible for Sn to span a distance longer than 2pn−1 , then Pn and Sn must
be estranged.

Lemma 4.1.2.3: The Partial Intersection


Case 3: Pn and Sn are estranged (the intersection is a proper subset of both).

Let Pn ∩ Sn = T, and suppose that Sn spans (k), k > 2pn−1 .

Let L = Pn - T, |L| = α
Let M = Sn - T, |M| = α
|T| = β
β+α= n

It follows that ∀m, mi > pn

/
In order to proceed, we must divide Case 3 into three subcases
.

Case 3a: T = Pn ∩ Sn = {2, … pn−2 }


Case 3b: T = Pn ∩ Sn = {2, 3, 5 … pn−j } j > 2
Case 3c: T = Pn ∩ Sn = { d1 , d2 , d3 ....dβ }

Case 3a is a special case of Case 3b, and Case 3b is a special case of the General Case, Case 3c.
Proving Case 3c in fact proves Case 3b and 3a, but the separate proofs of Case 3a and Case 3b have
their own beauty, a beauty which later transfers into the Inductive Case of the Andrica Conjecture.

Lemma 4.1.2.4: Maximal Gap Attained


Case 3a:
T = Pn ∩ Sn = {2, … pn−2 }

Since the sets Pn and Sn share all of the primes from 2 to pn−2 , then the only difference between the sets
is the existence of pn−1 and pn against sn−1 and sn .

Since the set Pn cannot sieve more than 2 pn−1 consecutive integers, then whenever pn−1 or pn become
active in generating a gap, they must be belligerent, since it is not possible to have two consecutive odd
multiples of pn−1 or pn contained within the same gap, since that gap would have to be larger than 2pn−1 .

Thus, if the set Sn could sieve more than 2pn−1 consecutive integers, and the locations (remainders) of all
the active primes in T were held stationary (unchanged) and the locations of where the primes sn−1 and sn
sieved an integer were exchanged with pn−1 and pn , then the gap generated by Sn sieving more than
2pn−1 consecutive integers could also be mapped onto a gap generated by the set Pn that also sieved
more than consecutive integers! This is an absurdity, since it has already been shown by brute force that
the set Pn cannot sieve more than consecutive integers!

However, since T = {2, … pn−2 }, it is possible to map each gap generated by Pn unto a gap generated by
Sn, since each system of congruences containing a belligerent occurrence of either pn−1 or pn can be
replaced by a system of congruences using the same remainders for all of the primes in Pn in respect to
each prime in Sn. Thus the following system of congruences for Sn would also generate a gap of length
2pn−1 .

μ1 ≡ 0 mod 2 ≡ 0 mod 3 ≡ 0 mod 5... ≡ 0 mod pn−2 ≡ − 1 mod sn−1 ≡ + 1 mod sn


μ2 ≡ 0 mod 2 ≡ 0 mod 3 ≡ 0 mod 5... ≡ 0 mod pn−2 ≡ + 1 mod sn−1 ≡ − 1 mod sn

Thus, if the set Sn can sieve more than 2pn−1 consecutive integers, then T ≠ {2, … pn−2 }.
QED

/
Let’s take a look at Case 3a and what it means visually:

Suppose we had the following nucleus:

μ ≡ 0 mod 2 ≡ 1 mod 3 ≡ 2 mod 5 ≡ 0 mod 7 ≡− 4 mod 11 , Pn = {2,3,5,7,11}


k μ

ξ ∅ 2 3 2 5 2 7 2 3 2 11 2 ∅

λ 1 0 0 0 0 0 0 0 0 0 0 0 1

We want to know if a gap of the same size exists for the fractured sets:

S1 = {2,3,5,7, s1 } or S2 = {2,3,5, s1 , s2 } or for the perforated set S3 = {2,3,5, s1 ,11}.

Quite obviously we can write the following system of congruences to yield the above gap for each set via
an exchange operation:

μ ≡ 0 mod 2 ≡ 1 mod 3 ≡ 2 mod 5 ≡ 0 mod 7 ≡− 4 mod s1 , S1 = {2,3,5,7, s1 }


μ ≡ 0 mod 2 ≡ 1 mod 3 ≡ 2 mod 5 ≡ 0 mod s1 ≡− 4 mod s2 , S2 = {2,3,5, s1 , s2 }
μ ≡ 0 mod 2 ≡ 1 mod 3 ≡ 2 mod 5 ≡ 0 mod s1 ≡− 4 mod 11 , S3 = {2,3,5, s1 , 11}

However, that observation alone doesn’t prove Case 3a, let’s look a little closer:

If δ [Pn] = 2pn−1 , then the primes pn−1 and pn will always be redundant, absent or belligerent, therefore
the prime modulus of pn−1 and/or pn can always be exchanged out for any prime greater than
themselves.

Assume pn−1 and pn are active (not nugatory), then:


μ ≡ a1 mod 2 ≡ a2 mod 3 ≡ a3 mod 5 ≡ ...an−1 mod pn−1 ≡ an mod pn , Pn = {2,3,5... pn−1 , pn }

k μ

ξ ∅ pn−1 pn ∅

λ 1 0 0 ... 0 0 0 0 0 ... 0 0 1

μ ≡ a1 mod 2 ≡ a2 mod 3 ≡ a3 mod 5 ≡ ...an−1 mod s1 ≡ an mod s2 , S2 = {2,3,5... pn−2 , s1 , s2 }

k μ

ξ ∅ s1 s2 ∅

λ 1 0 0 ... 0 0 0 0 0 ... 0 0 1

Quite simply, there exists an injection from the gaps of Pn into the gaps of S1 , S2 and S3 .
Likewise, there exists a surjection from the gaps of S1 , S2 and S3 onto the gaps of Pn.
See Appendix E for an extremely rigorous treatment of Case 3a.

/
Lemma 4.1.2.5: Maximal Gap Not Retained

Case 3b, The Fractured Set


T = Pn ∩ Sn = {2, 3, 5 … pn−j } j > 2 . Note that Sn qualifies as a “Fractured Set of Primes,” under these conditions.
Let L = Pn - T, |L| = α
Let M = Sn - T, |M| = α
|T| = β
β+α= n

Proposition 2.2.3.6: The Belligerence Decree


Since each prime in M is greater than pn , then each prime in M must be belligerent if it actively sieves an
integer within a gap generated by the set Sn, such that there if Sn could exceed the maximal span of Pn
then there would exist an illegal map of the maximal gap generated by Sn unto a gap generated by Pn of
whose span would also be greater than 2pn−1 . This is because each remainder for every prime modulus
in M that generated the gap, can be mapped onto a prime modulus in L, such that the gap would be of at
least the same size or greater.

In order to prove this Proposition (that each of the primes in M are belligerent) we must perform
both a proof by induction and contradiction (this is very similar to the actual proof of the Inductive Case for
the Andrica Conjecture and is the reason Case 3b is not simply declared a corollary of Case 3c even
though it is within our power to do so) and show that this will lead to infinite descent.

Let us first examine the case for P7 = {2,3,5,7,11,13,17} and some corresponding S7 and then generalize
the proof for Pn.

P7 = {2,3,5,7,11,13,17},
S7 = {2,3,5,7,61,73,83}, a fractured set.

Step 1, 1:
If S7 could generate a gap greater than 26 and the prime number 83 was not belligerent, then the set, S6,
which equals {2,3,5,7,61,73} must sieve all of the integers between two consecutive odd multiples of 83,
which is a span of 166 consecutive integers. However, the set S6 cannot sieve more than 22 consecutive
integers, because it has already been proven by brute force that P6 cannot sieve more than 22
consecutive integers and the intersection of P6 and S6 contains all but the two greatest primes in P6, and
Case 3a already showed that sets of size w sharing all but the two greatest primes with Pw cannot sieve
more than 2pw−1 consecutive integers.

We know from the above (Case 3a) that δ {2,3,5,7,11,61,73} = δ [P6] = 2(13) = 26, since we write
{2,3,5,7,11, s1 , s2 } instead of {2,3,5,7,11,61,73}, and we already know that there exists a surjection from
{2,3,5,7,11, s1 , s2 } onto [P6], therefore δ {2,3,5,7,11, s1 , s2 } = δ [P6] = 26. Thus, we cannot span a bracket
of 83, which is a length of 2(83) = 166, with the set {2,3,5,7,11,61,73}.

/
Step 1, 2:
If S7 could generate a gap greater than 26 and the prime number 73 was not belligerent, then the set, S6,
which equals {2,3,5,7,61,89} must sieve all of the integers between two consecutive odd multiples of 73,
which is a span of 146 consecutive integers. However, the set S6 cannot sieve more than 22 consecutive
integers, because it has already been proven by brute force that P6 cannot sieve more than 22
consecutive integers and the intersection of P6 and S6 contains all but the two greatest primes in P6, and
Case 3a already showed that sets of size w sharing all but the two greatest primes with Pw cannot sieve
more than 2pw−1 consecutive integers.

We know from the above (Case 3a) that δ {2,3,5,7,11,61,89} = δ [P6] = 2(13) = 26, since we write
{2,3,5,7,11, s1 , s2 } instead of {2,3,5,7,11,61,89}, and we already know that there exists a surjection from
{2,3,5,7,11, s1 , s2 } onto [P6], therefore δ {2,3,5,7,11, s1 , s2 } = δ [P6] = 26. Thus, we cannot span a bracket
of 73, which is a length of 2(73) = 146, with the set {2,3,5,7,11,61,89}.

Step 1, 3:
If S7 could generate a gap greater than 26 and the prime number 61 was not belligerent, then the set, S6,
which equals {2,3,5,7,73,89} must sieve all of the integers between two consecutive odd multiples of 61,
which is a span of 122 consecutive integers. However, the set S6 cannot sieve more than 22 consecutive
integers, because it has already been proven by brute force that P6 cannot sieve more than 22
consecutive integers and the intersection of P6 and S6 contains all but the two greatest primes in P6, and
Case 3a already showed that sets of size w sharing all but the two greatest primes with Pw cannot sieve
more than 2pw−1 consecutive integers.

We know from the above (Case 3a) that δ {2,3,5,7,11,73,89} = δ [P6] = 2(13) = 26, since we write
{2,3,5,7,11, s1 , s2 } instead of {2,3,5,7,11,73,89}, and we already know that there exists a surjection from
{2,3,5,7,11, s1 , s2 } onto [P6], therefore δ {2,3,5,7,11, s1 , s2 } = δ [P6] = 26. Thus, we cannot span a bracket
of 61, which is a length of 2(51) = 122, with the set {2,3,5,7,11,73,89}.

Thus it has been shown, in general, that if the intersection of Sw and Pw contains all but the three greatest
primes in Pw, then the maximal gap cannot exceed 2pw−1 .

End of Step 1.

We then proceed to show these steps over and over for increasing sizes of the fracture (n − w) .

/
Lemma 4.1.2.6: Image of the Maximal Gap Not Retained by Sn

Case 3b, The Fractured Case


T = Pn ∩ Sn = {2, 3, 5 … pn−j } j > 2
Let L = Pn - T, |L| = α
Let M = Sn - T, |M| = α
|T| = β
β+α= n

Step A: The Case of Three Belligerent Primes, j = 3


T = Pn ∩ Sn = {2, 3, 5 … pn−3 }

If Sn could generate a gap greater than 2pn−1 and the prime number sn was not belligerent, then the set
Sn-1 must sieve all of the integers between two consecutive odd multiples of sn , which is a span of 2sn
consecutive integers. However, the set Sn-1 cannot sieve more than 2pn−2 consecutive integers, because
it has already been proven by brute force that Pn-1 cannot sieve more than 2pn−2 consecutive integers and
the intersection of Pn-1 and Sn-1 contains all but the two greatest primes in Pn-1, and Case 3a already
showed that sets of size w sharing all but the two greatest primes with Pw cannot sieve more than 2pw−1
consecutive integers.

Therefore if T = Pn ∩ Sn = {2, 3, 5 … pn−3 } and it is claimed that the primes Sn can convene to sieve
more than 2pn−1 consecutives integers, then a stretch failure will occur for the subset Sn-1 if any of the
primes in M are said to not be belligerent.

Therefore if the set Sn could span a gap of 2pn−1 then each of primes in M must be belligerent, however,
once the system of congruences for that gap were made available, we could simply exchange (map) each
prime modulus in M onto each prime modulus in L, while retaining the same remainders, while all of the
prime modulii in T would map onto each other, also retaining the same remainders.

Such a map would imply that the set Pn was also capable of sieving more than 2pn−1 consecutive
integers, however it has already been proven by brute that the set Pn cannot do so, therefore it is not
possible for the set Sn to sieve more than 2pn−1 consecutive integers either if T is the intersection betwixt
them.

Thus it follows that if Sn can sieve more than 2pn−1 consecutive integers and the intersection of Sn and Pn
is a consecutive list of primes starting from 2, then Pn ⋂ Sn = {2, 3, 5 … pn−j } j > 3 , therefore, we shall
state the following lemma:

Lemma 4.1.2.7; “The Step A Lemma.”


Furthermore, it has been shown that no set of Sw integers can sieve more than 2pw−1 consecutive
integers if Pw ∩ Sw = {2, 3, 5 … pw−3 }, assuming that the Andrica Base Case has been proven for Pw .

/
Step B: “Pandora’s Box.”
T = Pn ∩ Sn = {2, 3, 5 … pn−j } j = 4
Let L = Pn - T, |L| = α = j
Let M = Sn - T, |M| = α = j
|T| = β
β+α= n

If Sn could generate a gap greater than 2pn−1 and one (and only one) of the prime numbers, mk , in M,
was not belligerent, then the set Sn-1 must sieve all of the integers between two consecutive odd multiples
of mk , which is a span of 2mk consecutive integers. However, the set Sn-1 cannot sieve more than 2pn−2
consecutive integers, because it has already been proven in the prior step that Sn-1 cannot sieve more
than 2pn−2 consecutive integers if the intersection of Pn-1 and Sn-1 contains all but the three greatest
primes in Pn-1.

Therefore, any claim the demands that one, and only one, of the primes in M are not belligerent
when the primes in Sn convene to span a length greater than 2pn−1 , would then cause a stretch failure for
some subset of Sn-1 since it has already been shown in Step A (Step A Lemma) that Sn-1 cannot sieve
more than 2pn−2 consecutive integers, which is a span that is far less than the ( 2mk − 1 ) consecutive
integers between the two odd consecutive multiples of 2mk .

Thus, if T = Pn ∩ Sn = {2, 3, 5 … pn−j } j = 4 , and it is claimed that Sn, can generate of gap
whose length is greater than 2pn−2 , then either none of the primes in M are belligerent; or, two, three, or all
four of the primes in M are not belligerent.

In order to proceed, let us define a temporary function that will represent the count the amount of
primes in M that are belligerent (this function will be expanded upon later in the paper in a more robust
fashion) and then restate the above paragraph in respect to this new function.

Lemma 4.1.2.8: The Estranged Belligerence Function


Let the function ⊖ (M) be equal to the amount of primes in M are belligerent in respect to a gap
generated by the primes in Sn. The value of ⊖ (M) is bounded, such that 0 ≤ ⊖ (M) ≤ α .

Any contradiction that would force ⊖ (M) to assume a value outside those bounds shall
henceforth be known as an Estrangement Failure. Now let us resume the proof of Case 3b and restate
the italicized paragraph.

Thus, if T = Pn ∩ Sn = {2, 3, 5 … pn−j } j = 4 , and it is claimed that Sn can generate of gap


whose length is greater than 2pn−1 , then ⊖ (M) =/ 1; therefore, ⊖ (M) = 0 or 2, or 3 or 4.

Then let us examine the case of ⊖ (M) = 0.

/
Step C: The Case of j = 4 and ⊖ (M) = 0.

If Sn could generate a gap greater than 2pn−1 and none of the prime numbers in M were
belligerent (but active), then there would be two odd multiples of each prime in M within this gap that was
supposedly generated by the all the primes in Sn (remember that is has already been shown that the
nugatory set is non-empty when the maximal gap occurs, therefore all of the primes in M are active, but
the claim says that none of them are belligerent, therefore each prime in M must have sieved two or
more odd multiples of themselves within the boundaries of μ − γ and μ + λ ).

Then let us choose any prime in M, mi , and identify the two consecutive odd multiples of mi that
occur within the boundaries of (μ − γ ) and (μ + λ) that the prime mi has sieved as (k)mi and (k + 2)mi .

Now let S’ = S - {mi } , then the set S’ must sieve every integer between (k)mi and (k + 2)mi , such
that they sieved a total of 2mi consecutive integers; however, the intersection of S’ and Pn-1 contains all
but the three greatest primes in Pn-1 and Lemma 2.3.3.7 (The Step A Lemma) already informs that when
such an intersection occurs that the set Sw-3 cannot span more than 2pw−1 consecutive integers. In this
case, w = (n − 1) , therefore S’ cannot span more than 2pn−2 consecutive integers, and thus cannot span
the distance between the two odd consecutive multiples of mi .

Therefore, if the set S’ cannot span this distance, then no other subset of S’ could span this
distance either, since the length of the maximal gap generated by a set of primes can only increase as the
size of the set increases, and therefore, can only decrease if the size of the set also decreases.

Thus, if mi is active and not belligerent, there exists no subset of Sn that can sieve the integers
between the two consecutive odd multiples of mi , thus a stretch failure has occurred, therefore:
⊖ (M) =/ 0.

Thankfully, the manner in which we disproved the the above case of ⊖ (M) = 0 provided us with a
generalized method to disprove the cases of ⊖ (M) = 2 and ⊖ (M) = 3. In fact, the the above method
resembles the general disproof so closely that I only need to copy and paste the above method and make
a few tweaks (the disproof for the case ⊖ (M) = 4 is the stepping stone between the other cases and the
generalized fallacy of infinite descent, which is why it shall be excluded from the general case of j = 4).

Before we proceed to the general case of j = 4, a simple theorem from the prior disproof will be
formally stated:

Theorem 4.1.2.10: The Superset and Subset Span Theorem (a restatement of Theorem 2.3.1)

Let S’ ⊇ S, then since S’ contains every element of S, then maximal span of S’ is greater than
the maximal span of S, because the system of congruences for the maximal span of S can be mapped
into a corresponding system of congruences for S’ where the extra elements in S’ are either all nugatory,
such that the gap remains the same length, or one or more of the primes in S’ become active, sieving
μ − γ and/or μ + γ , making the gap larger, such that the maximal span of S’ is greater than the maximal
span of S.

/
Thus, as a corollary, if S’ ⊆ S, the maximal span of S’ must decrease in respect to the maximal
span of S, such that if S can be shown to be incapable of spanning of certain distance, than neither can
S’ or any other subset of S span that distance.

This is also the same as the Proper Superset Span Theorem, just worded differently for the
sake of the proof of the Surjection Theorem.
QED

Step D: The General Case of j = 4 and ⊖ (M) ≤ (j − 1)

If Sn could generate a gap greater than 2pn−1 and at least one prime, mi , in M was not belligerent,
such that ⊖ (M) ≤ 3 , then there would exist two odd multiples of mi within this gap that was supposedly
generated by all the primes in Sn .

Now let S’ = S - {mi } , then the set S’ must sieve every integer between (k)mi and (k + 2)mi , such
that they sieved a total of 2mi consecutive integers; however, the intersection of S’ and Pn-1 contains all
but the three greatest primes in Pn-1 and Lemma 2.3.3.7 (The Step A Lemma) already informs that when
such an intersection occurs that the set Sw-3 cannot span more than 2pw−1 consecutive integers. In this
case, w = (n − 1) , therefore S’ cannot span more than 2pn−2 consecutive integers, and thus cannot span
the distance between the two odd consecutive multiples of mi .

Therefore, if the set S’ cannot span this distance, then no other subset of S’ could span this
distance either (Theorem 4.1.2.10).

Thus, if mi is active and not belligerent, there exists no subset of Sn that can sieve the integers
between the two consecutive odd multiples of mi , thus a stretch failure has occurred, therefore:
⊖ (M) =/ 0, 1, 2 or 3.

Thus, the only way in which Sn can sieve more than 2pn−1 consecutive integers, while j = 4, if is
⊖ (M) = 4. If we can disprove this case, then it will be shown that this gap cannot occur when j = 4.

Step E: The General Case of j = 4 and ⊖ (M) = 4

If Sn could generate a gap greater than 2pn−1 when all of the primes in M were belligerent, such
that ⊖ (M) = 4 , then the system of congruences for Sn generating this gap could be directly mapped
unto a system of congruences for Pn, replacing each prime modulus in M with a prime in L, that would
also have generated a gap greater than 2pn−1 . However, this is absurd since it has already been proven
by brute force that the span of Pn cannot exceed 2pn−1 . Therefore, j =/ 4 .

Lemma 4.1.2.11; “The Dynamic Lemma.”


Furthermore, it has been shown that no set of Sw integers can sieve more than 2pw−1 consecutive
integers if Pw ∩ Sw = {2, 3, 5 … pw−4 }, assuming that the Andrica Base Case has been proven for Pw .

For the sake of the remainder of this proof, we restate the above Lemma in it’s dynamic form.

/
Furthermore, it has been shown that no set of Sw integers can sieve more than 2pw−1 consecutive
integers if Pw ∩ Sw = {2, 3, 5 … pw−j }, assuming that the Andrica Base Case has been proven for Pw .

We have now managed to close the lid on the proverbial “Pandora’s Box,” which was the title that
I gave to Step B. Let us now proceed to the final steps showing a disproof by infinite descent.

Step F: The General Case of j = x and ⊖ (M) = x


T = Pn ∩ Sn = {2, 3, 5 … pn−j } j = x
Let L = Pn - T, |L| = α = j
Let M = Sn - T, |M| = α = j
|T| = β
β+α= n

If Sn could generate a gap greater than 2pn−1 when all of the primes in M were belligerent, such
that ⊖ (M) = x , then the system of congruences for Sn generating this gap could be directly mapped
unto a system of congruences for Pn, replacing each prime modulus in M with a prime in L, that would
also have generated a gap greater than 2pn−1 . However, this is absurd since it has already been proven
by brute force that the span of Pn cannot exceed 2pn−1 .

Therefore the case for j = x fails when ⊖ (M) = x , therefore ⊖ (M) =/ x

Step G: The General Case of j = x and ⊖ (M) < x


T = Pn ∩ Sn = {2, 3, 5 … pn−j } j = x
Let L = Pn - T, |L| = α = j = x
Let M = Sn - T, |M| = α = j = x
|T| = β
β+α= n
5 ≤x≤n
Assume that the General Case has been proven for j < x such that the Dynamic Lemma has been proven
up to (x-1).

If Sn could generate a gap greater than 2pn−1 then at least one prime, mi , in M was not
belligerent, such that ⊖ (M) < x , then there would exist two odd multiples of mi within this gap that was
supposedly generated by all the primes in Sn .

Now let S’ = S - {mi } , then the set S’ must sieve every integer between (k)mi and (k + 2)mi , such
that they sieved a total of 2mi consecutive integers; however, the intersection of S’ and Pn-1 contains all
but the (x − 1) greatest primes in Pn-1 and Lemma 4.1.2.11 (The Dynamic Lemma) already informs that
when such an intersection occurs that the set Sw-j+1 cannot span more than 2pw−1 consecutive integers. In
this case, w = (n − 1) , therefore S’ cannot span more than 2pn−2 consecutive integers, and thus cannot
span the distance between the two odd consecutive multiples of mi , since 2pn−2 < 2mi .

Therefore, if the set S’ cannot span this distance, then no other subset of S’ could span this
distance either (Theorem 2.3.1).

/
Thus it matters not if any, all, some or none of the remaining primes in M do or do not actively
sieve an integer between (k)mi and (k + 2)mi , because any active subset of primes in Sn that do sieve
integers within the range of (k)mi and (k + 2)mi would be some subset of S’, which, as stated above,
cannot span the distance between (k)mi and (k + 2)mi .

Thus, if mi is active and not belligerent, there exists no subset of Sn that can sieve the integers
between the two consecutive odd multiples of mi , thus a stretch failure has occurred, therefore:
⊖ (M) ≥ x.

Step H: Infinite Descent


T = Pn ∩ Sn = {2, 3, 5 … pn−j } j = x
Let L = Pn - T, |L| = α = j = x
Let M = Sn - T, |M| = α = j = x
|T| = β
β+α= n
5 ≤x≤n

Since Step G demonstrates that ⊖ (M) ≥ x and Step F demonstrates that ⊖ (M) =/ x , then in order for
Sn to span a gap greater than 2pn−1 when j = x, then ⊖ (M) > x. However, in accordance with Definition
4.1.2.8:
0 ≤ ⊖ (M) ≤ α , and α = j = x, thus an Estrangement Failure has occurred.

Therefore it is not possible for Sn to span a gap greater than 2pn−1 when j = x, and thus the limit
of the Dynamic Lemma has been increased from (x-1) to (x).

Thenceforth, we can repeat all of the same arguments in Step F and Step G for j = (x+1) and
further increase the limit of the Dynamic Theorem to (x+1), whence one could perpetually repeat Steps F
and G until j = x = n, which would exhaust all of the possible intersections between Pn ∩ Sn , therefore
finally proving that the conditions of Case 3b cannot violate the Strong Andrica Conjecture.
QED Case 3b

/
Case 3c: T = Pn ∩ Sn = { t1 , t2 , t3 ....tβ , }; The Perforated Case (and the General Case)

Let L = Pn - T, |L| = α
Let M = Sn - T, |M| = α
|T| = β
Case 3c is the most generalized case of all the preceding cases. In fact, all of the previous cases
are special cases of the Case 3c. However, the tools and methods that we have developed to solve the
previous cases will also apply to this case (and, in my opinion, those methods could not have been
discovered by mere intuition for the generalized case of 3c).

n
We begin by realizing that Case 3c concerns the power set of Pn, such that there are 2 distinct
subsets of Pn. Thus, the intersectory set, T, which is Pn ∩ Sn, must be some element of power set of Pn.

Our goal in Case 3c is the same as our goal was in Case 3b: To prove that each prime in M must
be belligerent. Once this goal has been achieved, we can show that there exists an illegal surjection.

However, in Case 3c, certain counterclaims and countersuppositions arise that are not as easily
dealt with as they are in Case 3b. Sheer intuition allowed us to dismiss these possibilities in the previous
cases, such that they did not even need to be addressed in the proofs of the prior cases; however, we
cannot avoid them in Case 3c, they must be dealt with.

Since all of the prior cases were specialized cases of Case 3c, the manner in which we deal with
these countersuppositions in Case 3c also also completes the proofs of the previous cases (even though
it’s rather trivial to see why those countersuppositions are easily dismissed in the prior cases). Thus, the
completed proof of Case 3c is in fact a complete proof of all of the prior cases as well.

Before we begin the proof of Case 3c, we will need a few more definitions and theorems:

Definition 4.1.2.11; An Absolutely Belligerent Prime:


If si ∈ D, and si only actively sieves one integer between (μ − γ + 1) and (μ + γ − 1) then, by definition, si
is the only prime that divides that integer and therefore cannot be removed from S without forfeiting the
integrity of the gap, this prime is said to be absolutely belligerent to μ .

Definition 4.1.2.12; A Partially Belligerent Prime:


If si ∈ D, and si only actively sieves one integer between (μ − γ + k1 ) and (μ + γ − k2 ) then, by definition, si
is the only prime that divides that integer and therefore cannot be removed from S without forfeiting the
integrity of the gap within the range of (μ − γ + k1 ) and (μ + γ − k2 ) , this prime is said to be partially
belligerent within the range of (μ − γ + k1 ) and (μ + γ − k2 ) .

Definition 4.1.2.13; A Bracketed Prime (restated for the Surjection Theorem):


If si ∈ D, and si only actively sieves two integers between (μ − γ + 1) and (μ + γ − 1) then, by
definition, si is the only prime that divides those integers and therefore cannot be removed from S without
forfeiting the integrity of the gap, this prime is said to be bracketed within μ .

/
The word “bracket,” comes from the physical interpretation of attaching the ends of a bracket to
either of the integers being sieved by that prime (multiples of that prime). A nest of bracketed primes
becomes an immovable object, a concept that will apply to inductive case of the Andrica Conjecture.

Since the term “bracket” is a physical interpretation, the phrase “within the bracket of s,” refers to
the range of consecutive integers between the two integers being actively sieved by s.

The ultimate strength of a bracketed prime is that they represent “subgaps” within a gap
generated by Sn. Since the prime si bounds either end of this “subgap” then we know that si itself is not
active within that range (it is absent), thus some smaller subset of Sn (of size n-1 or smaller) is
responsible for sieving those integers.

It is this special ability of bracketed primes (the ability that reduces the size of the active set of
primes) which ultimately enables us to solve the inductive case of the Andrica Conjecture by reducing the
active set of Pn+1 to some smaller subset of size n (or less). Once we are dealing with a set of primes of
size n (or less), then we already know, by the virtue of the Surjection Theorem, when a stretch failure will
occur since the Base Case has already been proven up to n by brute force. However, Case 3c still
remains to be proved before we have a complete proof of the Surjection Theorem!

Definition 4.1.2.14; A Partially Bracketed Prime:

If si ∈ D, and si only actively sieves two integers between (μ − γ + k1 ) and (μ + γ − k2 ) then, by


definition, si is the only prime that divides those integers and therefore cannot be removed from S without
forfeiting the integrity of the gap within the range of (μ − γ + k1 ) and (μ + γ − k2 ) , this prime is said to be
partially bracketed within the range of (μ − γ + k1 ) and (μ + γ − k2 ) .

Definition 4.1.2.15; Anchor, The Delta and Gamma Functions, The 1-0-1 Anchor:

Let s be a prime, when two consecutive odd multiples of that prime, k(s) and (k + 2)(s) , are
actively sieved by s, they form a partially bracketed prime within the range k(s) and (k + 2)(s) . The
midpoint of that bracket is an even multiple of that prime, (k + 1)(s) , and since 3 divided neither k(s) nor
(k + 2)(s) , then 3 must have divided (k + 1)(s) .

The function that returns the value of an anchor for the bracket of s is the Delta Function, Δ(s) ,
and if a gap contains several distinct anchors,

The Gamma Function counts the number of distinct anchors within a gap: Γ(μ) , this is known as
the Anchorage of μ .

The Delta-Array is an ordered list (from least to greatest) of the integers located at each of the
[
distinct anchors within a gap, appearing as: Δ1 , Δ2 ...ΔΓ(μ) ]
The strength of anchors is that they represent the midpoint of bracket, and, as previously stated,
bracketed primes represent “subgaps” within gaps. Thus, just as we represent a gap by a system of

/
congruences in respect to its midpoint, μ , we can also represent a “subgap” or bracket by a system of
congruences in respect to its midpoint Δ(s) .

Thus anchors provide us with a powerful method by which we can parse and dissect partitions of
gaps (brackets/subgaps) within a controlled environment, and ultimately provide us with just enough
control to tame the inductive case of the Andrica Conjecture and force it into compliance with the Base
Case of the Strong Andrica Conjecture.

Also note the system of congruences that generates the maximal gap of Pn, you’ll see that all of
the primes are bracketed and they they all share the same anchor.

However, before we move unto the Absentee Theorem, the type of anchor being discussed in the
paragraph of this definition will be known as a 1-0-1 anchor. This is because the even integer (k + 1)(s) is
the anchor of the bracketed prime s; and due to the length of this bracket, the even integer (k + 1)(s) is
also divisible by 3, and thus ultimately the anchor, (k + 1)(s) , is divisible by 6.

The reason this particular type of anchor is named a 1-0-1 anchor is due to the fact that these
odd multiples of s are considered to be Principal Dirichlet Characters in respect to the modulus 6, where
the outer (odd) multiples of s are coprime to 6 and the inner (even) multiple of s is divisible by both 2 and
3. Thus if we were inspecting an arithmetic sequence of integers in the form of y = x(s) , and converted
this sequence into Dirichlet Characters in respect to the modulus 6, then the bracketed sequence would
appear as [1,0,1].

The concept of a 1-0-1 anchor will not be needed for case 3c, but before the inductive case of the
Andrica Conjecture can be solved, Dirichlet Characters must be introduced, and 1-0-1 anchors and
1-0-0-0-1 anchors will follow as a result.

Also, on the topic of Dirichlet Characters, in order for the inductive case of the Strong Andrica
Conjecture to be proven, we will be forced to deliver an elementary proof related to Dirichlet’s Theorem
using the Surjection Theorem, since we must show that there exists a maximum distance between
totatives of Ω (Sn) in the form of a + (x)d , where a and d are coprime, and that maximal distance (given
appropriate bounds for a and d) falls within the quadratic interval of the Sieve of Eratosthenes.

Ultimately, once the inductive case of the Strong Andrica Conjecture has been proven (proving
Andrica’s Conjecture), we can revisit this elementary proof concerning arithmetic sequences and revise it
to deliver an elementary proof of Dirichlet's Theorem itself.

However, we have must now proceed to the proof Case 3c of the Surjection Theorem, yet one
last theorem must be presented before we can proceed to the proof of Case 3c.

Theorem 4.1.2.17; The Absentee Theorem:


Let s be a prime: For any sequence of (s + 1) consecutive integers, s must be either redundant or
semi-belligerent within that range of (s + 1) integers; for any sequence of (2s + 1) consecutive integers, s
must be either redundant or semi-belligerent or semi-bracketed within that range of (2s + 1) integers.

Any contradiction (absurdity) that demands the some prime, s, is absent within a sequence of
(s+1) integers is known as an Absentee Failure.

/
The Proof of Case 3c, The Perforated Case (General Case)
Case 3c: T = Pn ∩ Sn = { t1 , t2 , t3 ....tβ , }

Let L = Pn - T, |L| = α
Let M = Sn - T, |M| = α
|T| = β

Step A: Proving that m1 is absolutely belligerent , α = 1, β = n − 1


T = Pn ∩ Sn = { t1 , t2 , t3 ....tn−1 , }
M = { m1 }

If Sn could generate a gap greater than 2pn−1 and the prime number m1 was active, but not
absolutely belligerent in respect to μ , then m1 must be bracketed about some anchor within the gap.

Then the set T must sieve all of the integers between two consecutive odd multiples of m1 , which
is a span of 2m1 consecutive integers. However, the set T cannot sieve more than 2pn−1 consecutive
integers, because it has already been proven by brute force that Pn cannot sieve more than 2pn−1
consecutive integers and the intersection of Pn and Sn is T. which is proper subset of Pn, whose span is
limited to 2pn−1 consecutive integers. Therefore, in accordance with the Superset and Subset Span
Theorem, the maximum span of T is less than 2pn−1 , and therefore cannot span the distance between
between two consecutive odd multiples of m1 .

Therefore, since T cannot span a length greater than 2pn−1 consecutive integers, then m1 cannot
be bracketed in respect to μ (or partially bracketed) and, as a consequence, m1 must be absolutely
belligerent in respect to μ , therefore ⊖ (M) = 1.

Step B: Proving that Sn cannot span 2pn−1 , α = x, β = n − α , ⊖ (M) = α


Tβ = Pn ∩ Sn = { t1 , t2 , t3 ....tβ , }
M = { m1 , m2 ...mα }
L = Pn - Tβ.

If the primes in Sn could generate a gap greater than 2pn−1 , and each prime number in M was
absolutely belligerent in respect to μ , then each prime modulus in M could be exchanged with a prime
modulus in L for the system of congruences representing the gap, such that the primes in Pn would be
coerced to have generated a gap of the same size of greater, which is absurd, since it has already been
proven by brute force that the primes in Pn cannot convene to generate a gap greater than the limit
imposed by the Strong Andrica Conjecture.

Thus, since Step A showed that m1 must be absolutely belligerent, Step B shows that α ≠ 1 if the
primes in Sn have convened to generate a gap whose length exceeds 2pn−1 , thus α ≤ 2 .

Step C: Proving that m1 , m2 are absolutely belligerent , α = 2, β = n − 2


T = Pn ∩ Sn = { t1 , t2 , t3 ....tn−2 }

/
M = { m1 , m2 }
L = Pn- M.

Let M ordered such that m1 < m2

If Sn could generate a gap greater than 2pn−1 and either of the primes in M were active, but not
absolutely belligerent in respect to μ , then that prime must be partially bracketed about some anchor, Δ1 ,
within the gap. Let this prime be m1 , then m2 is either absent within the bracket of m1 or, since m2 > m1 ,
then m2 partially belligerent within the bracket of m1 .

Let G = T

If m2 is is partially belligerent within the bracket of m1 , then G must span the bracket of m1 ,
however, the prime modulus m2 in system of congruences for the primes in G in respect to the anchor
Δ(m1 ) , can be replaced with a prime modulus from L, and this system of congruences (using L) would be
a proper subset of Pn, we call name this subset P′n. Therefore if G can span the bracket of m1 , then P′n
can also span the bracket m1 .

However, since the superset Pn cannot span a range beyond 2pn−1 , then neither can its subset
P′n, and since the bracket of m1 spans 2m1 consecutive integers, and m1 > pn−1 , then if G spans the
bracket of m1 , there would exist an illegal surjection unto the subset P′n , thus G cannot span the bracket
of m1 . Therefore m1 cannot be bracketed. Now we must address the case of m2 being bracketed.

Let M ordered such that m1 < m2

If Sn could generate a gap greater than 2pn−1 and m2 was active, but not absolutely belligerent in
respect to μ , then m2 must be partially bracketed about some anchor, Δ2 , within the gap. The smaller
prime, m1 , is either absent within the bracket of m1 or, since m1 < m2 , then m1 partially belligerent within
the bracket of m2 or partially bracketed itself (nested) within the bracket of m2 .

If m1 is partially bracketed within the bracket of m2 , then let Δ1 be the anchor for the nested m1
bracket. It follows that the primes in T must span the bracket of m1 , since both m1 and m2 are absent
within the bracket of m1 ; however, T is a proper subset of Pn, and we know that Pn cannot span the
bracket of m1 , and therefore the subset T cannot span this bracket either, thus m1 cannot be nested
within m2 .

For the remainder of this paper, when a smaller prime, si , is partially bracketed within the bracket
of a larger prime, sj , we will say that si is nested within sj .

Then, if m2 is partially bracketed about the anchor Δ2 , then m1 must be partially belligerent or
absent within the bracket of m2 . If m1 is partially belligerent within the bracket of m2 ,then:

Let G = T

/
If m1 is is partially belligerent within the bracket of m2 , then G must span the bracket of m2 ,
however, the prime modulus m1 in system of congruences for the primes in G in respect to the anchor
Δ(m2 ) , can be replaced with a prime modulus from L, and this system of congruences (using L) would be
a proper subset of Pn, we call name this subset P’n. Therefore if G can span the bracket of m2 , then P’n
can also span the bracket m2 .

However, since the superset Pn cannot span a range beyond 2pn−1 , then neither can its subset
P’n, and since the bracket of m2 spans 2m2 consecutive integers, and m2 > pn−1 , then if G spans the
bracket of m2 , there would exist an illegal surjection unto the subset P’n , thus G cannot span the bracket
of m2 . Therefore m1 cannot be partially belligerent within the bracket of m2 .

Thus if m1 can neither be nested within m2 nor partially belligerent within the bracket of m2 , then
must be m1 absent within the bracket of m2 in order for Sn to generate a gap exceeding 2pn−1
consecutive integers; however, this is absurd since the bracket of m2 contains a sequence of 2 m2
consecutive integers, and in accordance with the Absentee Theorem, since m1 < m2 , m1 cannot be
absent within the bracket of m2 , thus an Absentee Failure has occurred.

Thus if m1 can neither be nested within m2 , nor partially belligerent within the bracket of m2 , nor
m1 absent within the bracket of m2 in order for Sn to generate a gap exceeding 2pn−1 consecutive
integers, the only remaining option is that m1 is redundant within the bracket of m2 , such that T was the
active set spanning the bracket of m2 , and we have already shown that T cannot span the bracket of m2 .

Thus, we have shown that when α = 2, that neither m1 nor m2 can be bracketed if Sn can
generate a gap exceeding 2pn−1 consecutive integers, the it follows that m1 and m2 are absolutely
belligerent for this supposed gap (if either is nugatory, then it’s not the maximal gap for Sn in accordance
with the Superset Span Theorem). Therefore ⊖ (M) = 2 , and therefore ⊖ (M) = α .

Thus, since Step C showed that m1 and m2 must be absolutely belligerent when α = 2, such that
⊖ (M) = 2 , and Step B shows that the primes in Sn cannot generate a gap whose length exceeds 2pn−1
when ⊖ (M) = α , then it follows that α ≠ 2 if Sn can generate such a gap; therefore, in accordance with
the previous steps, α ≤ 3 .

Step D: Proving that m1 , m2 ...mα are absolutely belligerent , α = x, β = n − x


Infinite Descent:
T = Pn ∩ Sn = { t1 , t2 , t3 ....tβ }
M = { m1 , m2 ...mα }
L = Pn- M

Let M ordered such that m1 < m2 < ...mα

If Sn could generate a gap greater than 2pn−1 and any prime in M, mi , was active, but not
absolutely belligerent in respect to μ , then mi must be partially bracketed about some anchor, Δi , within
the gap.

/
Any primes in M that are smaller than mi , must be redundant, partially belligerent or nested within
mi , however they cannot be absent within the bracket of mi due to the Absentee Theorem; conversely,
any primes in M that are larger than mi must be absent, redundant or partially belligerent within the
bracket of mi , however they cannot be nested within mi since the lengths of their brackets would exceed
the length of the bracket of mi .

Let the set of primes smaller than mi be denoted as F , and the set of primes larger than mi be
denoted as M′, and let the greatest prime in F that is nested within be f 1 (assuming there are any primes
in F nested within mi ) and finally let |F| = k.

Now let G = Sn - { mi , f 1 }, then G must span the bracket of f 1 (or some smaller subset of G).

Furthermore, if there exists another prime in F that is nested within f 1 , call this f 2 , or more
generally if there exists finite series of nests of the primes f i+1 within f i , from f 1 to f c , c ≤ k ≤ α − 1 ,
f i+1 < f i , and let the set of primes f 1 to f c be known as F′ and let the set F″ = F - F′, then let the subset
G′ = G - F′ , which must span the bracket of f c .

Since M′ is subset of G′, and every prime in M′ is greater than mi and f 1 , then if any prime in M′
is active within the bracket of f c , then those primes in M′ would be partially belligerent within that range,
since they cannot be nested within f c due to their size; furthermore if any of the primes in F″ are active
within the bracket of f c , then they must also be partially belligerent within the bracket of f c since by
definition they cannot be nested within f c , and F″ is a subset of G′.

Let E = M′ ∪ F″; let the the anchor of f c = Δc

Therefore the primes in G′ must span the bracket of f c . However, if this were possible, then we
could replace each prime modulus in E with any of the prime modulii in the subset L, since each prime in
E is partially belligerent within the bracket of f c , such that it would generate a system of congruences for
some subset of Pn in respect to Δc that also managed to span the bracket of f c .

However, such a system of congruences would be illegal since the span of the bracket of f c
contains 2f c consecutive integers, and the set Pn cannot sieve more than 2pn−1 consecutive integers,
and therefore no subset of Pn can sieve more than 2pn−1 consecutive integers, and since 2pn−1 < 2f c , then
a stretch failure has occurred, and therefore neither can E span the bracket of f c since it would induce a
stretch failure for Pn.

Thus it follows that neither mi nor any prime in F′ can be bracketed, and since we could have
made the greatest prime in M equal to mi , it also follows that no prime in M can be bracketed in respect
to μ , therefore all primes in M must be absolutely belligerent in respect to μ .

Thus, since every prime in M must be belligerent for all values of α , ⊖ (M) = α in all cases, and
therefore, in accordance with Step B, there exists no set Sn that can sieve more than 2pn−1 consecutive
integers, assuming that the Base Case of the Strong Andrica Conjecture has been proven for Pn .

QED Case 3c.

/
In order to complete the proof of the Surjection Theorem we must show that, with exception of
Case 3a (Lemma 2.2.3.4), that Sn can not span a length that exceeds 2pn−1 and furthermore, cannot
even span a length of 2pn−1 .

So far we have shown that the former, that Sn can not span a length that exceeds 2pn−1 . Showing
that latter statement, however, that the maximum span of Sn is less 2pn−1 (excluding Case 3a), will
require an extension of the logic that was already presented thus far.

We know that ⊖ (Pn) = 2 when the primes in Pn convene to generate the maximal gap of 2pn−1 .
We also know, through brute force, that maximal gap only occurs once (ignoring its additive inverse),
making it unique. Thus, if ⊖ (Pn) ≥ 3 for any system of congruences in respect to μ that generates a
gap, then, by the virtue of the unique form of the maximal gap, the length of this gap is less than 2pn−1 .
We will define this result as a theorem:

Theorem 4.1.2.18; The Uniquity Theorem:


Assume the the Strong Andrica Conjecture has been proven up to Pn through brute force, then if
⊖ (Pn) ≥ 3 for any gap that is generated by Pn, then the length of that gap must be less than 2pn−1 ,
since the system of congruences that generates the maximal gap of 2pn−1 is unique and ⊖ (Pn) = 2
when that system of congruences occurs in respect to μ .

In Case 3b and Case 3c, it was shown that when |M| > 3, that ⊖ (M) ≥ 3 and therefore that could
be mapped unto a subset of Pn such that ⊖ (L) ≥ 3. In these cases, it follows that Sn cannot span 2pn−1 ,
since not even Pn can span 2pn−1 when ⊖ (Pn) ≥ ⊖ (L) ≥ 3.

Of course, this leaves the case of |M| ≤ 2 so far unresolved for Case 3c; whereas if |M| ≤ 2 for
Case 3b then it would be Case 3a instead, where the length of the maximal gap was actually retained by
Sn.

If |M| ≤ 2 then there only exists one or two primes in Sn that do not intersect with Pn . We already
know that these primes absolutely belligerent, and we also know that either pn−1 or pn is in Sn , and
therefore within the intersectory set T (otherwise it would be Case 3a). For the sake of simplicity, we will
denote either pn−1 or pn as z, such that z ∈/ M, and therefore z ∈ T.

Step 1:
|M| = 2
z = pn
Since all primes in in Sn must be active when they convene to form their maximal gap, then z
must also be active, however z cannot be bracketed since the remaining primes in Sn would have to span
the bracket of z . This is impossible because the remaining primes in Sn are a subset of Sn, and the
primes in M must be absolutely belligerent, therefore the subset of Sn spanning the bracket of z can be
mapped unto system of congruences for some subset of Pn that also spans the bracket of z , which is
absurd since even the set Pn itself cannot span 2 pn consecutive integers, nevermind one of its subsets.

/
Step 2:
|M| = 2
z = pn−1
Since all primes in in Sn must be active when they convene to form their maximal gap, then z
must also be active, however z cannot be bracketed since the remaining primes in Sn would have to span
the bracket of z . This is impossible because the remaining primes in Sn are a subset of Sn and the
singleton prime in M must be absolutely belligerent, therefore the subset of Sn spanning the bracket of z
can be mapped unto system of congruences for the subset Pn that also spans the bracket of z , which is
absurd since the set Sn - { z } cannot span more than 2 pn−2 consecutive integers since this subset has a
size of (n − 1) .

Thus, in accordance with Steps 1 and 2, z must be absolutely belligerent in respect to μ , and
since |M| = 2 and z ∈/ M and ⊖ (M) =2 , then ⊖ (Sn) ≥ 3, therefore Sn cannot span a range of 2pn−1
consecutive integers since it would induce a stretch failure when projected unto a system of congruences
for Pn.

Step 3:
|M| = 1
z = pn
Since all primes in in Sn must be active when they convene to form their maximal gap, then z
must also be active, however z cannot be bracketed since the remaining primes in Sn would have to span
the bracket of z . This is impossible because the remaining primes in Sn are a subset of Sn, and the
singleton prime in M must be absolutely belligerent, therefore the subset of Sn spanning the bracket of z
can be mapped unto system of congruences for some subset of Pn that also spans the bracket of z ,
which is absurd since even the set Pn itself cannot span 2 pn consecutive integers, nevermind one of its
subsets. Therefore z cannot be bracketed.

Thus z must be absolutely belligerent in respect to μ , and since |M| = 1 and z ∈/ M and z =/ pn−1
and pn−1 must also be absolutely belligerent in respect to μ and ⊖ (M) = 1 , then ⊖ (Sn) ≥ 3, therefore
Sn cannot span a range of 2pn−1 consecutive integers since it would induce a stretch failure when
projected unto a system of congruences for Pn.

Step 4:
|M| = 1
z = pn−1
Since all primes in in Sn must be active when they convene to form their maximal gap, then z
must also be active, however z cannot be bracketed since the remaining primes in Sn would have to span
the bracket of z . This is impossible because the remaining primes in Sn are a subset of Sn and the
primes in M must be absolutely belligerent, therefore the subset of Sn spanning the bracket of z can be
mapped unto system of congruences for the subset Pn that also spans the bracket of z , which is absurd
since the set Sn - { z } cannot span more than 2 pn−2 consecutive integers since this subset has a size of
(n − 1) . Therefore z cannot be bracketed.

Thus z must be absolutely belligerent in respect to μ , and since |M| = 1 and z ∈/ M and z =/ pn
and pn must also be absolutely belligerent in respect to μ and ⊖ (M) = 1 , then ⊖ (Sn) ≥ 3, therefore Sn

/
cannot span a range of 2pn−1 consecutive integers since it would induce a stretch failure when projected
unto a system of congruences for Pn.

Thus we have shown that even when |M| ≤ 2, that Sn cannot span 2pn−1 consecutive integers
(excluding Case 3a).

Thus we can state the following:

i=n
The count of the gaps generated by Sn is given by the formula: |TotS| = ∏ (si − 1) ,
i=1
i=n
The count of the gaps generated by Pn is given by the formula: |TotP| = ∏ (pi − 1) ,
i=1

It follows that |TotS| > |TotP| ; however, since the length of any gap generated by Sn can be
mapped unto a gap generated by Pn of equal or greater length, then it follows that there always exists a
surjection from each gap generated by Sn unto the gaps of Pn.

However, only in exceptionally rare cases (Case 3a) does there exist an injection from all the
gaps generated by Pn into the some of the gaps generated by Sn, but otherwise there generally does not
exist such an injection.

QED
Surjection Theorem Complete

End of Chapter 1

/
Chapter 1, Local Appendices

Appendix A
Computer Script to Brute Force Verify
The Strong Andrica Conjecture.

Computer Code for Brute Force Check of Strong Andrica Conjecture

Let P be a linear array of consecutive primes of size n − 1 (starting from P[0] = 2).
Let G be an integer equal to the product of the integers in the array P.

Let A be an integer variable, initially equal to 0.


Let B be an integer variable, initially equal to 0.
Let D be an integer variable, initially equal to 0.
Let M be an integer variable, initially equal to 0. (max difference)

Let U be an integer array of size 4, all initially equal to 0. (midpoints of max difference)
Let Ubool be a boolean variable, initially equal to True (the Strong Andrica Conjecture
confirmation).
Let Ucheck be an integer variable, initially equal to 0.

Let V be a boolean variable, initially equal to True.


Let W be a boolean variable, initially equal to False. (Loop Switch)

Let K be an integer variable, initially equal to 0.


Let L be an integer variable, initially equal to 0.

Loop Alpha: For each integer from 1 to G.


Set K = K + 1
Set L = − 1
Set V equal to True
Loop Beta: For integer from 0 to n − 1
Set L = L + 1
If K ≡ 0 mod S[L],
Then
set V equal to False
Stop Loop Beta
Else
Do nothing
If V equals true
Then
If W equals true
Then
Set W equal to false.
Else
Set W equal to true.

If W equals true
Then
Set A = K.
Set D = A - B
Else
Set B = K.
Set D = B - A

If D > M
Then
Set M = D
Else
Do nothing.

If M ≥ 2P[ n − 2 ]
Then
If Ucheck < 2 and M = 2P[ n − 2 ]
Then
Set Ucheck = Ucheck + 1
Set U[Ucheck] = 12 A+B
Else
Set Ubool = false
Stop all loops.
Else
Do nothing.

Else
Do nothing.

If Ubool = true
Then
Display M, U[1] and U[2] and show that U[3] is empty (or zero depending on preference).
Else
Display text: Strong Andrica Conjecture Failed. The world is doomed! Display M, U[1], U[2], U[3].
End of Code

Remember that this code is using n − 1 as the final index, not n , since arrays use the index “0.”
This is why the code ultimately checks 2P[ n − 2 ] using n − 2 as the index instead of n − 1 . Use your own
method to solve the system of congruences for U[1] and U[2] and U[3] in respect to P.
Appendix B
Counting Gaps of Length k,
Class A-D Formulas

Theorem 1.10.0 Class A0 Formula, The Null Count.


Let Sn be a set of prime numbers, and n be the cardinality of S, such that |S| = n and
∀s ∈ S, s ≥ (n + 1) , then for all k , such that k > (n + 1) , |𝚳k| = 0.

Theorem 1.10.1 Class A1 Formula, The Factorial Count.


Let Sn be a set of prime numbers, and n be the cardinality of S, such that |S| = n and
∀s ∈ S, s ≥ (n + 1) , then for all k , such that k = (n + 1) , |𝚳k| = n! .

Theorem 1.10.2 Class A2 Formula, The Permutative Count.


Let Sn be a set of prime numbers, and n be the cardinality of S, such that |S| = n and
∀s ∈ S, s ≥ (n + 1) , then for all k , such that k < (n + 1) , |𝚳k| = c , such that:

( )
m= (k−1) i=n
c= ∑ (− 1)m (km−1) ∏ (si − 2 − m)
m=0 i=1

Definition 1.10.12 Class B1 Counting Formula, The Cyclic Preset Count

Given [TS, h = 21 Ω (T), let S - T = Y, such that |Y| = n − m = v = -1 + Φ ( Ω (T))


and ∀y ∈ Y, y > h if 2 ∉ S, OR, y > 2h if 2 ∈ S; then:

( )
a= (w−1) i=v
a
c= ∑ (− 1) (wa−1) ∏ (y i − 2 − m) ; w = σ [TS, h ] = -1 + Φ ( Ω (T)).
a=0 i=1

Definition 1.10.13 Class B2 Counting Formula, The Repeating Cyclic Preset Count

Given [TS, h = (g) 21 Ω (T), let S - T = Y, such that |Y| = n − m = v = -1 + (g)Φ ( Ω (T))
and ∀y ∈ Y, y > h if 2 ∉ S, OR, y > 2h if 2 ∈ S; then:

( )
a= (w−1) i=v
a
c= ∑ (− 1) (wa−1) ∏ (y i − 2 − m) ; w = σ [TS, h ] = -1 + (g)Φ ( Ω (T))
a=0 i=1
Definition 1.10.17 Class B3 Counting Formula, Augmented Senary Preset:

In general, if si > k , ∀i ≥ 3 , and 2, 3 ∈ S, and (n − 2) ≥ β , then:

Since 2(s3 ) ≢ 0 mod 3 , then

( )
m=β i=n
(s3 −1−m) m 2(s3 )
|𝚳 2(s3 )
|= ∑ (
(s3 −2−m) −
1) (mβ ) ∏ (si − 2 − m) , where β = Floor[ 3
]
m=0 i=3

Definition 1.10.13 Class C1 Counting Formula, Complete or Partial Senary Cyclic


Presets

Let T = {2, 3}, |T| = m = 2

Given [TS, h = z + (g) 21 Ω (T), let S - T = Y, such that:

|Y| = n − m = v ≥-1 + (g)Φ ( Ω (T)), if z ≡ 0 mod 6


|Y| = n − m = v ≥ (g)Φ ( Ω (T)), if z ≢ 0 mod 6

and ∀y ∈ Y, y > 2h if 2 ∈ S; then:

or, if h ≡ 0 mod 3 then

|𝚳h| = 2 ( m=β

m=0
( m
i=v
(− 1) (mβ ) ∏ (y i − 2 − m)
i=1
)) , where β = -1 + 2 g

or, if k ≢ 0 mod 3 then

( )
m=β i=v
m
|𝚳h| = ∑ (− 1) (mβ ) ∏ (y i − 2 − m) , where β = 2g
m=0 i=1
General Class C2 Counting Formula, Partial Cyclic Presets:
Gaps of h .

( )
f =h m= ⌹( f, 1 ) i=v
m
h
|𝚳 | = c = ∑ [(⌹( f , 2 )) ∑ (− 1) (⌹(mf , 1 ) ) ∏ (y i − 2 − m) ]
f =1 m=0 i=1

Class D1 Formula
Gaps of Ω (T)

|𝚳 Ω(T ) | = Φ ( Ω (T)) ( a= (w−1)



a=0
( a
i=v
(− 1) (wa−1) ∏ (y i − 2 − m)
i=1
)) ; w = σ [TS, h ] = -1 + Φ ( Ω (T)).
Expanded Senary Formulas

The cardinality of M k, for k = 2, 4, 6, 8, 10, 12, 14, 16, 18, 20 , and the general formula for any k
thereafter, assuming that both 2 and 3 are in S, where M k is the set of all gaps of length k generated by
S.

Let {2, 3} ⊂ S, let |S| = n and let S be ordered from least to greatest:

i=n
|M2| = ∏ (si − 2)
i=3
i=n
|M4| = ∏ (si − 2)
i=3

Note that the count of twin and cousin coprimes is equal (M2 and M4), alluding to the Hardy and Littlewood Conjecture.

(∏ )
i=n i=n
|M6| = 2 (si − 2) − ∏ (si − 3) , n≥3
i=3 i=3

i=n i=n i=n


|M8| = ∏ (si − 2) − 2 ∏ (si − 3) + ∏ (si − 4) , n ≥ 4
i=3 i=3 i=3

i=n i=n i=n


|M10| = ∏ (si − 2) − 2 ∏ (si − 3) + ∏ (si − 4) , n ≥ 4
i=3 i=3 i=3

or, if 5 ∈ S, then

i=n i=n i=n


|M10| = 43 ∏ (si − 2) − 2 3
2 ∏ (si − 3) + 21 ∏ (si − 4) , n ≥ 4
i=3 i=3 i=3

1
For the rest of these formulas, it is assumed that all primes in S are greater than 2k , excluding 2
and 3.
1
11
121
1331
14641
1 5 10 10 5 1
1 6 15 20 15 6 1

(∏ )
i=n i=n i=n i=n
|M12| = 2 (si − 2) − 3 ∏ (si − 3) + 3 ∏ (si − 4) − ∏ (si − 5) , n≥5
i=3 i=3 i=3 i=3

i=n i=n i=n i=n i=n


|M14| and |M16| = ∏ (si − 2) − 4 ∏ (si − 3) + 6 ∏ (si − 4) − 4 ∏ (si − 5) + ∏ (si − 6) ,
i=3 i=3 i=3 i=3 i=3
1,485 - 4,480 +4,914 - 2,304 + 385 = 0, {11, 13, 17}

(∏ )
i=n i=n i=n i=n i=n i=n
|M18| = 2 (si − 2) − 5 ∏ (si − 3) + 10 ∏ (si − 4) − 10 ∏ (si − 5) + 5 ∏ (si − 6) − ∏ (si − 7) ,
i=3 =3 i=3 i=3 i=3 i=3
n≥7

|M20|, |M22| =
i=n i=n i=n i=n i=n i=n i=n
∏ (si − 2) − 6 ∏ (si − 3) + 15 ∏ (si − 4) − 20 ∏ (si − 5) + 15 ∏ (si − 6) − 6 ∏ (si − 7) + ∏ (si − 8), n≥8
i=3 =3 i=3 i=3 i=3 i=3 i=3

In general, let m C β be the binomial coefficient, then, if si > 12 k, ∀i ≥ 3 , and 2, 3 ∈ S, then

if k ≡ 0 mod 3 then

|Mk| = 2
( m=β

m=0
( (− 1)
m
mC β
i=n
∏(si − (m + 2))
i=3
)) , where β = k
3

or, if k ≢ 0 mod 3 then

( )
m=β i=n
m
|𝚳k| = ∑ (− 1) mC β ∏(si − (m + 2)) , where β = Ceiling[ k3 ]
m=0 i=3
Expanded Trigesimal Formulas

The cardinality of M k, for k = 10, 12, 14, 16, 18, 20, 22, 24, 26, 28, 30 .
Let {2, 3 ,5} ⊂ S, let |S| = n let S be ordered from least to greatest, and it is assumed that no
prime in S is less than k besides 2,3 and 5, then let and let Y = S - {2,3,5}, then, |Y| = v = n − 3 , then:

( )
i=v i=v i=v
4
|𝚳10| = c = 3 ∏ (y i − 2) − ( 32 )2 ∏ (y i − 3) + 2
1 ∏ (y i − 4)
i=1 i=1 i=1

( )
i=v i=v i=v i=v
|𝚳12| = 2 + 1 ∏ (y i − 2) − 7 ∏ (y i − 3) + 5 ∏ (y i − 4) − 1 ∏ (y i − 5)
i=1 i=1 i=1 i=1

( )
i=v i=v i=v i=v
|𝚳14| = c = + 3 ∏ (y i − 2) − 8 ∏ (y i − 3) + 7 ∏ (y i − 4) − 2 ∏ (y i − 5)
i=1 i=1 i=1 i=1

( )
i=v i=v i=v i=v i=v
|𝚳16| = c = + 3 ∏ (y i − 2) − 10 ∏ (y i − 3) + 12 ∏ (y i − 4) − 6 ∏ (y i − 5) + ∏ (y i − 6)
i=1 i=1 i=1 i=1 i=1

( )
i=v i=v i=v i=v i=v
|𝚳18| = c = 2 + 3 ∏ (y i − 2) − 11 ∏ (y i − 3) + 15 ∏ (y i − 4) − 9 ∏ (y i − 5) + 2 ∏ (y i − 6)
i=1 i=1 i=1 i=1 i=1

( )
i=v i=v i=v i=v i=v i=v
|𝚳22| = c = + 3 ∏ (yi − 2) − 14 ∏ (yi − 3) + 26 ∏ (yi − 4) − 24 ∏ (yi − 5) + 11 ∏ (yi − 6) − 2 ∏ (yi − 7)
i=1 i=1 i=1 i=1 i=1 i=1

|𝚳24| = c =

( )
i=v i=v i=v i=v i=v i=v i=v
2 + 3 ∏ (y i − 2) − 16 ∏ (y i − 3) + 35 ∏ (y i − 4) − 40 ∏ (y i − 5) + 25 ∏ (y i − 6) − 8 ∏ (y i − 7) + 1 ∏ (y i − 8)
i=1 i=1 i=1 i=1 i=1 i=1 i=1

|𝚳26| = c =

( )
i=v i=v i=v i=v i=v i=v i=v
3 + 3 ∏ (y i − 2) − 6 ∏ (y i − 3) + 15 ∏ (y i − 4) − 20 ∏ (y i − 5) + 15 ∏ (y i − 6) − 6 ∏ (y i − 7) + 1 ∏ (y i − 8)
i=1 i=1 i=1 i=1 i=1 i=1 i=1
|𝚳28| = c =

( )
i=v i=v i=v i=v i=v i=v i=v
3 + 3 ∏ (y i − 2) − 6 ∏ (y i − 3) + 15 ∏ (y i − 4) − 20 ∏ (y i − 5) + 15 ∏ (y i − 6) − 6 ∏ (y i − 7) + 1 ∏ (y i − 8)
i=1 i=1 i=1 i=1 i=1 i=1 i=1

( )
i=v i=v i=v i=v i=v i=v
|𝚳20| = c = + 4 ∏ (yi − 2) − 18 ∏ (yi − 3) + 32 ∏ (yi − 4) − 28 ∏ (yi − 5) + 12 ∏ (yi − 6) − 2 ∏ (yi − 7)
i=1 i=1 i=1 i=1 i=1 i=1

(∏ )
i=v i=v i=v i=v i=v i=v i=v i=v
|𝚳30| = 8 (y i − 2) − 7 ∏ (y i − 3) + 21 ∏ (y i − 4) − 35 ∏ (y i − 5) + 35 ∏ (y i − 6) − 21 ∏ (y i − 7) + 7 ∏ (y i − 8) − ∏ (y i − 9) .
i=1 i=1 i=1 i=1 i=1 i=1 i=1 i=1
Appendix C
Counting Surjections (Maps)

Making a general formula that can count surjections and denials for any size gap, for any size of
two sets of large primes, is no light task, but our Class A2 formula provides the first step.

( )
m= (k−1) i=n
c= ∑ (− 1)m (km−1) ∏ (si − 2 − m) .
m=0 i=1

The class A2 formula already recognizes every permutation of the solution vectors in (k) -space,
where k is the length of the gap; however, it does not recognize the restrictions (non-trivial denials) of the
complimentary (n − k) -space in which the solution vectors are projected into.

This forces us to examine each of the permutations of each Preset in k -space, but in turn the
shape of the complimentary (n − k) -space becomes constant...yet, each new Preset (permutation) will
also generate a distinct array inducing a new Dirichlet-Kernel for each. The problem is therefore multifold,
but not unsolvable.

Before I simply “tell” you the solution to this problem, we’ll do an example of a gap of 8 (gap of 4
without 2 in the Set), which requires 3 primes to cover the unsieved integers. We’ll increase the size of
our sieving sets to 7, that way we will have a complimentary space of four dimensions, which will show us
exactly how the filters (and therefore kernels) are being applied in each permutation.
Let S = {2,7,67,71,73,79, 83,101} and let T = {2,11,13,19, 23, 29, 31, 103}, maximal gap equals
10. F is Nugatory Set.

S and T, Gap of 10, Principal Dirichlet Character on top, zeta function bottom, F = { bt , bw , bc , be , bh }

1 0 0 0 μ1 0 0 0 1

ξ (k) =∅ 2 bx 2 by 2 bz 2 ∅

Since we have seven belligerent primes and three unsieved integers, we have 210 vectors in
3-space that correspond the necessary remainders for each modulus that are required to complete the
length of this nucleus.

Let W be generalization of both S and T: W = {2, w1 , w2 , w3 , w4 , w5 , w6 , w7 }. We need at least


three primes in W to sieve the integers at μ − 2 , μ and μ + 2 : μ ≡ + 2 mod wi ≡ 0 mod wk ≡ − 2 mod wj .
n!
The permutation formula (n−k)!
tells us that there’s 210 different ordered combinations of three primes
from a set of seven primes, and thus 210 immediate solutions to the systems of congruences (solution
vectors).

We can now replace each w with their respective s and t moduli. Each of those 210 3-vectors
are solutions which must be projected through each cell of complimentary 5-space, respecting the filters
that are induced by each permutation.

First we examine all vectors from { w1 , w2 , w3 }. There are six mutually exclusive orderings of the
remainders {-2, 0,+2} to the { w1 , w2 , w3 } moduli; more generally, the number of orderings of the (k − 1)
remainders per set of (k − 1) moduli is simply (k − 1)! Since k = 4 , which is the length of our gap,
(k − 1)! = 6 .

Each of these six 3-vectors are now orthogonally projected through every cell of 5-space, obeying
the initial boundaries and filters of w4, w5, w6 , w7 , which sieve all integers +/- 4 mod ( t4, t5, t6 , t7 ) within the
boundaries of ± 12 (s4 − 1) , ± 12 (s5 − 1) , ± 12 (s6 − 1) and ± 12 (s7 − 1) We apply our earlier Denial formula
separately to the dimensions of w4, w5, w6 , w7 .

s
Number of Denials = (f )(F loor[ t i ]) + 2ε , where f is the number of filters.
i

1
a= (s
2 i − 1)

b = a − 12 (ti − 1) .

c = b mod ti ; 0 ≤ c ≤ ti

z = F loor( at ) .
i
Apply sieve over the range: z (ti ) + c ; Number of Denials, X , in the final partition is the number of
integers sieved in that range by the filters, ε = c − X
We will assign variables to these numbers for w4, w5, w6 , w7 : G 1,1,4 , G1, 1,5 , G 1,1,6 , G1,1,7
f = 2 in this first slave iteration. The indices tell us the following numbers: Master iteration, Slave
iteration and last index is the index of each w .

Since our first Slave iteration would appear as six columns being erected through an independent
5-space, none of them would intersect. Thus our count so far, for the first Slave iteration, would be given
by: 6( w4 − G1,1,4 )(w5 − G1,1,5 )(w5 − G1,1,6 )(w5 − G1,1,7 ) .

Before we continue, let’s discuss what a Master Iteration and a Slave Iteration are. We first
examine the Power Set of W (not including the prime number 2), W = { w1 , w2 , w3 , w4 , w5 , w6 , w7 }. From
the power set we want to know all sets of size four, since we want a list of all the complimentary spaces.

{1,2,3,4},{1,2,3,5},{1,2,3,6},{1,2,3,7},{1,2,4,5},{1,2,4,6},{1,2,4,7},{1,2,5,6},{1,2,5,7},{1,2,6,7},{1,3,4,
5},{1,3,4,6},{1,3,4,7},{1,3,5,6},{1,3,5,7},{1,3,6,7},{1,4,5,6},{1,4,5,7},{1,4,6,7},{1,5,6,7},{2,3,4,5},{2,3,4,6},{2,
3,4,7},{2,3,5,6},{2,3,5,7},{2,3,6,7},{2,4,5,6},{2,4,5,7},{2,4,6,7},{2,5,6,7},{3,4,5,6},{3,4,5,7},{3,4,6,7},{3,5,6,7}
,{4,5,6,7}.

We now find all of those sets that contain a 7 (and therefore end with a 7) and therefore we have
a list of all complimentary spaces utilizing the dimension of w7 . This is our first Master Iteration (I put the
complement of the complements side by side).

{4,5,6,7} {3,4,5,7} {1,2,3}{1,2,6}


{3,5,6,7} {2,4,5,7} {1,2,4}{1,3,6}
{2,5,6,7} {1,4,5,7} {1,3,4}{2,3,6}
{1,5,6,7} {2,3,5,7} {2,3,4}{1,4,6}
{3,4,6,7} {1,3,5,7} {1,2,5}{2,4,6}
{2,4,6,7} {1,2,5,7} {1,3,5}{3,4,6}
{1,4,6,7} {2,3,4,7} {2,3,5}{1,5,6}
{2,3,6,7} {1,3,4,7} {2,4,5}{2,5,6}
{1,3,6,7} {1,2,4,7} {2,4,5}{3,5,6}
{1,2,6,7} {1,2,3,7} {3,4,5}{4,5,6}

The reason we first find all complementary sets containing a 7 is because the 7th prime in this
scenario is also the greatest prime. As such, it will never be subject to a filter (beyond the original two
mandatory filters at μ ± 4 ).

This is because of the definition of the zeta function in this volume, ξ . It accepts the least prime
in W as the active divisor of some integer; hence, in order to prevent the recount of surjections throughout
the various counting iterations, we apply the filters only to primes in the complementary set that are less
than those that are in the solution vector set.
Let’s look at the first four iterations:

Iteration Solution Space, Complement Space, Complement Filters


Master# (k − 1) -space (n − k + 1) -space
Slave #

1,1 {w1 , w2 , w3 } {w4 , w5 , w6 , w7 } 2,2,2,2

1,2 {w1 , w2 , w4 } {w3 , w5 , w6 , w7 } 3,2,2,2

1,3 {w1 , w3 , w4 } {w2 , w5 , w6 , w7 } 4,2,2,2

1,4 {w2 , w3 , w4 } {w1 , w5 , w6 , w7 } 5,2,2,2

1,5 {w1 , w2 , w5 } {w3 , w4 , w6 , w7 } 3,3,2,2

Here we can see that in the first iteration, each prime in the complement space received 2 filters,
yet in the fifth iteration, the first two primes received 3 filters, and yet the last two retained their two filters.

The way this is determined is that there exists four Registers. They are determined as follows:

Definition 2.2.4, Surjection Register, the Filter Count, Filter Determinate π

Let W be a generalization of two sets of primes of the same size, S and T.


Let k be the length of the gap.

A solution space has (k − 1) primes, W′ , with (k − 1)! orderings. Let W′ be ordered from least to
greatest, let us rename W′ as Θ ; | Θ | = k − 1

Each set of primes in the complementary set, W′c, has (n − k + 1) primes. Let W′c be ordered from
least to greatest, let us rename W′c as Γ ; | Γ | = n − k + 1

Each prime in Γ in falls into one of the following relationships in respect to Θ :

Register 1: Γk < Θi , ∀i, i ≤ (k − 1) ; c = 1 , OR,

Register k : Γk > Θi , ∀i, i ≤ (k − 1) ; c = k , OR,

Register c : Γk > Θi , ∀i, i < c AND Γk < Θj , ∀i, j ≥ c ; 2 ≤ c ≤ (k − 1) .

The amount of filters applied to Γk equals (2 + (k − c)) .

Let π = (2 + (k − c)) , let this be called the Filter Determinate.


END OF DEFINITION 2.2.4
Using Definition 2.2.4 we can engineer a rather dramatic scenario and yet still know the amount
of filters that are being applied in each permutation. Let’s say we have a gap of 6 (gap of 12 with 2 in S
and T), then we need 5 primes to span the nucleus. Let |W| = 1000. We have 1000 primes from which we
must choose 5 primes. Since W is ordered from least to greatest, let us then take 5 random primes.

{w100 , w250 , w600 , w700 , w800 } . These primes would be sieving the integers from either terminal of
the nucleus, for gap of 12 (with 2 in S and T), the integers being sieved are μ + {− 4,− 2, 0,+ 2,+ 4} .

μ−6 μ−4 μ−2 μ μ+2 μ+4 μ+6

ξ ∅ 2 w600 2 w250 2 w700 2 w100 2 w800 2 ∅

Our complimentary set however, would contain 995 primes, yet, from Definition 2.2.4 we know
how to apply the filters. Since k = 6 , we will need six registers.

1st register: ∀wi , i < 100 , apply seven filters; 7 = 2 + 6 − 1


2nd register: ∀wi , 100 < i < 250 , apply six filters; 6 = 2 + 6 − 2 .
3rd register: ∀wi , 250 < i < 600 , apply five filters; 5 = 2 + 6 − 3
4th register: ∀wi , 600 < i < 700 , apply four filters; 4 = 2 + 6 − 4
5th register: ∀wi , 700 < i < 800 , apply three filters; 4 = 2 + 6 − 5
6th register: ∀wi , i > 800 , apply two filters; 4 = 2 + 6 − 6 .

No prime in the 1st register is permitted to divide any of the unsieved locations, since that would
reduce the value of ξ .

Any prime in 2nd register is only permitted to divide μ + 2 , since w100 already divides it.
Any prime in the 3rd register may divide μ + 2 or μ − 2 , since w100 or w250 already divides it.
Any prime in the 4th register may divide μ + 2 or μ − 2 or μ − 4 , since w100 or w250 or w600
already divides it.
Any prime in the 5th register may divide μ + 2 or μ − 2 or μ − 4 or μ , since w100 or w250 or w600
or w700 already divides it.
Any prime in the 6th register may divide μ + 2 or μ − 2 or μ − 4 or μ or μ + 4 , since w100 or w250
or w600 or w700 or w800 already divides it.

These registers do not recognize the specific ordering (permutation) of {w100 , w250 , w600 , w700 , w800 } ,
since the effect would be the same no matter where we place them in the nucleus. Thus we’ve reduced
our problem to combinations of W, rather than permutations, furthermore we’ve trivialized the fitler
dilemma.

Thus, the conundrum facing us is not how to count and apply filters, but rather how to order our
iterations!

Let us return to the more comprehensible example utilizing 7 primes and four registers and write
out all the slave iterations for the first Master Iteration, and then see if we can organize them better. For
computer programmers, this is a question of finding the optimal way in which to arrange a series of
nested loops to reduce computation to a minimum.

The first master iteration yields every sequence of filters ending in a 2. Within this Master Iteration
you can see that I set it up so that they end in 2,2, then 3,2, then 4,2, then 5,2.
Iteration Solution Space, Complement Space, π
Master# (k − 1) -space (n − k + 1) -space Filter Determinate
Slave #

1,1 {w1 , w2 , w3 } {w4 , w5 , w6 , w7 } 2,2,2,2

1,2 {w1 , w2 , w4 } {w3 , w5 , w6 , w7 } 3,2,2,2

1,3 {w1 , w3 , w4 } {w2 , w5 , w6 , w7 } 4,2,2,2

1,4 {w2 , w3 , w4 } {w1 , w5 , w6 , w7 } 5,2,2,2

1,5 {w1 , w2 , w5 } {w3 , w4 , w6 , w7 } 3,3,2,2

1,6 {w1 , w3 , w5 } {w2 , w4 , w6 , w7 } 4,3,2,2

1,7 {w2 , w3 , w5 } {w1 , w4 , w6 , w7 } 5,3,2,2

1,8 {w1 , w4 , w5 } {w2 , w3 , w6 , w7 } 4,4,2,2

1,9 {w2 , w4 , w5 } {w1 , w3 , w6 , w7 } 5,4,2,2

1,10 {w3 , w4 , w5 } {w1 , w2 , w6 , w7 } 5,5,2,2

1,11 {w1 , w2 , w6 } {w3 , w4 , w5 , w7 } 3,3,3,2

1,12 {w1 , w3 , w6 } {w2 , w4 , w5 , w7 } 4,3,3,2

1,13 {w2 , w3 , w6 } {w1 , w4 , w5 , w7 } 4,3,3,2

1,14 {w1 , w4 , w6 } {w2 , w3 , w5 , w7 } 4,4,3,2

1,15 {w2 , w4 , w6 } {w1 , w3 , w5 , w7 } 5,4,3,2

1,16 {w3 , w4 , w6 } {w1 , w2 , w5 , w7 } 5,5,3,2

1,17 {w1 , w5 , w6 } {w2 , w3 , w4 , w7 } 4,4,4,2

1,18 {w2 , w5 , w6 } {w1 , w3 , w4 , w7 } 5,4,4,2

1,19 {w3 , w5 , w6 } {w1 , w2 , w4 , w7 } 5,5,4,2

1,20 {w4 , w5 , w6 } {w1 , w2 , w3 , w7 } 5,5,5,2


Each order of the filter determinants is also unique. Whether or not we have 4 primes or 995
primes in our complementary space, the pattern (order) of determinants will never be repeated. Also, the
filter determinants cannot appear randomly, they appear in blocks; a determinate cannot be less than the
determinate that preceded it!

What then is the best way to arrange the combinations of W to reduce all computation to a bare
minimum? If we can find an answer to this, then we can write a generalized formula (with algorithmic
components) to count surjections that preserve the length of the nucleus.

Let’s increase the size of our complimentary space again. To keep things within the scope of
human observation, we’ll increase it to 13. Thus |W| = 16, since we need three primes from W to
complete the nucleus. Also, to keep the numbers small, we’re going to work in base-16 (hexadecimal),
except with a 16th digit instead of a 0.

1,2,3,4,5,6,7,8,9,A=10, B=11, C=12, D=13, E = 14, F= 15, G = 16

The first combination that comes to mind is always the simplest (call this Seed 1):

{1,2,3} vs {4,5,6,7,8,9,A,B,C,D,E,F,G} → {2,2,2,2,2,2,2,2,2,2,2,2,2}


{1,2,4} vs {3,5,6,7,8,9,A,B,C,D,E,F,G} → {3,2,2,2,2,2,2,2,2,2,2,2,2}
{1,2,5} vs {3,4,6,7,8,9,A,B,C,D,E,F,G} → {3,3,2,2,2,2,2,2,2,2,2,2,2}
{1,2,6} vs {3,4,5,7,8,9,A,B,C,D,E,F,G} → {3,3,3,2,2,2,2,2,2,2,2,2,2}

{1,2,F} vs {3,4,5,6,7,8,9,A,B,C,D,E,G} → {3,3,3,3,3,3,3,3,3,3,3,3,2}
{1,2,G} vs {3,4,5,6,7,8,9,A,B,C,D,E,F} → {3,3,3,3,3,3,3,3,3,3,3,3,3}

We generated Seed 1 by incrementing the last prime in the solution space.

That seems fairly straight forward so far. We’ve found every fitler determinant combination that
has either a 2 or 3, but not a 4 or a 5. Let’s see if we can find all those with a 3 or 4, but not a 2 or 5.

Call this Seed 2, we generate this seed by incrementing the second prime in the solution space.
{1,2,G} vs {3,4,5,6,7,8,9,A,B,C,D,E,F} → {3,3,3,3,3,3,3,3,3,3,3,3,3}
{1,3,G} vs {2,4,5,6,7,8,9,A,B,C,D,E,F} → {4,3,3,3,3,3,3,3,3,3,3,3,3}
{1,4,G} vs {2,3,5,6,7,8,9,A,B,C,D,E,F} → {4,4,3,3,3,3,3,3,3,3,3,3,3}
{1,5,G} vs {2,3,4,6,7,8,9,A,B,C,D,E,F} → {4,4,4,3,3,3,3,3,3,3,3,3,3}

{1,E,G} vs {2,3,4,5,6,7,8,9,A,B,C,D,F} → {4,4,4,4,4,4,4,4,4,4,4,4,3}
{1,F,G} vs {2,3,4,5,6,7,8,9,A,B,C,D,E} → {4,4,4,4,4,4,4,4,4,4,4,4,4}

We have now generated two “seeds” by incrementing the third prime and then the second prime,
let’s see what happens when we now increment our first prime, while starting from where we left off.
Call this Seed 3, we generate this seed by incrementing the first prime in the solution space.
{1,F,G} vs {2,3,4,5,6,7,8,9,A,B,C,D,E} → {4,4,4,4,4,4,4,4,4,4,4,4,4}
{2,F,G} vs {1,3,4,5,6,7,8,9,A,B,C,D,E} → {5,4,4,4,4,4,4,4,4,4,4,4,4}
{3,F,G} vs {1,2,4,5,6,7,8,9,A,B,C,D,E} → {5,5,4,4,4,4,4,4,4,4,4,4,4}
{4,F,G} vs {1,2,3,5,6,7,8,9,A,B,C,D,E} → {5,5,5,4,4,4,4,4,4,4,4,4,4}

{D,F,G} vs {1,2,3,4,5,6,7,8,9,A,B,C,E} → {5,5,5,5,5,5,5,5,5,5,5,5,4}
{E,F,G} vs {1,2,3,4,5,6,7,8,9,A,B,C,D} → {5,5,5,5,5,5,5,5,5,5,5,5,5}.

Thus our filter determinants are a result of the increments of each prime in the solution space, or
the “seed.” Let’s take this combination:

{5,9,C} vs {1,2,3,4,6,7,8,A,B,D,E,F,G} → {5,5,5,5,4,4,4,3,3,2,2,2,2}.

We result with four 5’s, three 4’s, two 3’s and four 2’s.

What is the difference between 5 and 1? 4


What is the difference between 9 and (5+1)? 3
What is the difference between C and (9+1)? 2
What is the difference between G and (C+1)? 4

Then our seed numbers show us the frequency of the filter determinants in each block (the
frequency of the primes in each Register):

{ x, y , z } → ( x − 1 ) primes in the 1st register; y − x + 1 primes in the second register, z − y + 1 in the


third register and G − z + 1 in the fourth register.
The only thing left to do is find an orderly fashion in which to increments the seeds. We want to
start with {1,2,3} and end with {E,F,G}. There are also well defined bounds on the values of each seed.

1 ≤ x ≤ E, 2 ≤ y ≤ F , 3 ≤ z ≤ G; x < y < z.

Again we start with the most basic combination:

{x, y , z } = {1, 2, 3} We will name this our first Master Iteration.

Now increment x , which forces y and z to also increment.


{x, y , z } = {2, 3, 4} , now we decrease x .
{x, y , z } = {1, 3, 4} , now we decrease y .
{x, y , z } = {1, 2, 4} . This is our second Master Iteration.

Now increment x twice, which forces y and z to also increment to their minimums.
{x, y , z } = {3, 4, 5} , now we decrease x .
{x, y , z } = {2, 4, 5} , now we decrease x .
{x, y , z } = {1, 4, 5} , now we decrease y , and reset x to its maximum.
{x, y , z } = {2, 3, 5} , now we decrease x .
{x, y , z } = {1, 3, 5} , now we decrease y , and reset x to its maximum..
{x, y , z } = {1, 2, 5} . This is our Third Master Iteration.

Now increment x thrice, which forces y and z to also increment to their minimums.
{x, y , z } = {4, 5, 6} , now we decrease x .
{x, y , z } = {3, 5, 6} , now we decrease x .
{x, y , z } = {2, 5, 6} , now we decrease x . (3 decreases)
{x, y , z } = {1, 5, 6} , now we decrease y , and reset x to its maximum.
{x, y , z } = {3, 4, 6} , now we decrease x .
{x, y , z } = {2, 4, 6} , now we decrease x . (2 decreases)
{x, y , z } = {1, 4, 6} , now we decrease y , and reset x to its maximum.
{x, y , z } = {2, 3, 6} , now we decrease x . (1 decrease)
{x, y , z } = {1, 3, 6} , now we decrease y , and reset x to its maximum.
{x, y , z } = {1, 2, 6} . Stop Loop. This is our Fourth Master Iteration.

The pattern should now be clear to an experienced computer programmer. This would continue
until the Fourteenth Master Iteration, at which point all combinations would be exhausted. Our Fifth
Master Iteration would start with 4 decreases, then 3 decreases, then 2 decreases, then one, then the
loop would stop. Our final Master Iteration (fourteenth) would start 13 decreases, then 12, then 11..etc.
So we’ve found the optimal way to increment three seeds. However, what if our number of seeds
were greater? In other words, what if we had a larger gap, and therefore a larger solution space? Let’s try
with 5 seeds. We’ll keep the size of W equal to 16 for the hexadecimal implementation.

1 ≤ x ≤ C , 2 ≤ y ≤ D , 3 ≤ z ≤ E , 4 ≤ a ≤ F , 5 ≤ b ≤ G; x < y < z < a < b.

Again we start with the most basic combination:

{x, y , z , a, b} = {1, 2, 3, 4, 5} We will name this our first Master Iteration. 1 step, 1 slave iteration, 1 step overall, 1= C (5, 5)

Now increment x , which forces y , z , a, b to also increment.


{x, y , z , a, b} = {2, 3, 4, 5, 6} , now we decrease x .
{x, y , z , a, b} = {1, 3, 4, 5, 6} , now we decrease y , and reset x to its maximum.
{x, y , z , a, b} = {1, 2, 4, 5, 6} , now we decrease z , and reset x, y to its maximum.
{x, y , z , a, b} = {1, 2, 3, 5, 6} , now we decrease a , and reset x, y , z to its maximum.
{x, y , z , a, b} = {1, 2, 3, 4, 6} . Stop All Loops. This is the Second Master Iteration (5 steps, 5 slave iterations, 6 steps
overall, 6 = C (6, 5) ).

Now increment x twice, which forces y , z , a, b to also increment to their minimums..


{x, y , z , a, b} = {3, 4, 5, 6, 7} , now we decrease x .
{x, y , z , a, b} = {2, 4, 5, 6, 7} , now we decrease x .
{x, y , z , a, b} = {1, 4, 5, 6, 7} , now we decrease y , and reset x to its maximum.
{x, y , z , a, b} = {2, 3, 5, 6, 7} , now we decrease x .
{x, y , z , a, b} = {1, 3, 5, 6, 7} , now we decrease y , and reset x to its maximum.
{x, y , z , a, b} = {1, 2, 5, 6, 7} , now we decrease z , and reset x, y to its maximum.
{x, y , z , a, b} = {2, 3, 4, 6, 7} , now we decrease x .
{x, y , z , a, b} = {1, 2, 4, 6, 7} , now we decrease z , and reset x, y to its maximum.
{x, y , z , a, b} = {1, 2, 3, 6, 7} , now we decrease a , and reset x, y , z to its maximum.
{x, y , z , a, b} = {2, 3, 4, 5, 7} , now we decrease x .
{x, y , z , a, b} = {1, 3, 4, 5, 7} , now we decrease y , and reset x to its maximum.
{x, y , z , a, b} = {1, 2, 4, 5, 7} , now we decrease z , and reset x, y to its maximum.
{x, y , z , a, b} = {1, 2, 3, 5, 7} ,now we decrease a , and reset x, y , z to its maximum.
{x, y , z , a, b} = {1, 2, 3, 4, 7} . Stop All Loops. This is the Third Master Iteration (15 steps. 15 slave iterations, 21 steps
overall, 21 = C (7, 5) )

Thus the number of nested loops being run is (k − 1) , where k is the length of the gap,
since we need (k − 1) primes to span a gap of length k , and therefore our number of seeds is
also the same. The number of overall iterations should be C (5 + M − 1, 5) , where M denotes the
Master Iteration Number.

Also, rather using separate local variables, use an array instead for the seeds:

Array Z = { z 1 , z 2 , z 3 ...z (k−1) }. Since there are C (n, k − 1) distinct seeds, we turn this Array into a
Matrix of height C (n, k − 1) .

Matrix Z( C (n, k − 1) , k ) = { z 1 , z 2 , z 3 ...z (k−1) } for each overall increment, h .


We now compile a list of filter determinants using the rank 3 tensor. Let’s take another look at
Definition 2.2.4:

Register 1: Γk < Θi , ∀i, i ≤ (k − 1) ; c = 1 , OR,


Register k : Γk > Θi , ∀i, i ≤ (k − 1) ; c = k , OR,
Register c : Γk > Θi , ∀i, i < c AND Γk < Θj , ∀i, j ≥ c ; 2 ≤ c ≤ (k − 1) .
The amount of filters applied to Γk equals (2 + (k − c)) .
Let π = (2 + (k − c)) , let this be called the Filter Determinate.

n!
From this we create a new matrix of length n and height (k −1)!(n−r−1)!
, which is C (n, k − 1) . Call this
Matrix Y.

For each h , denoting the height index of matrix Y, we retrieve the seed combination that
generated that index from Matrix Z, Array Z = { z 1 , z 2 , z 3 ...z (k−1) }. With these seeds for each iteration of h
we then proceed to divide every integer from 1 to n into k Registers, using Definition 2.2.4, we then
apply the appropriate π to each integer from 1 to n according to its register. For the integers from 1 to n
that are in the sequence { z 1 , z 2 , z 3 ...z (k−1) }, we apply a ∅ instead, since they delineate the registers.

We now turn Array Z into its actual meaning: S′ and T′ , and therefore S′c and T′c.

We are now ready to construct our General Formula to count surjections:

h=C(n,k −1) i=n−k+1


s
c= ∑ (k − 1)! ∏ (s′c i − π (F loor[ t i ] + 2ε) , S′c = S - S′, for each Array Z.
i
h=1 i =1

ε:
a= 1
(s
2 i − 1) ; b = a − 12 (ti − 1) ; c = b mod ti ; 0 ≤ c ≤ ti ; z = F loor( at ) .
i
Apply sieve over the range: z (ti ) + c ; Number of Denials, X , in the final partition is the number of integers
sieved in that range by the filters, ε = c − X .

We can use Matrix Y to remove most of the calculation by retrieving the appropriate data, but ε
will always have to be calculated not just for each combination of W, but also for each permutation, since
different remainders (unsieved integers) in the solution space will generate different ε . Thus we cannot
simply apply π over the sieve range for ε .

For an estimate, we can remove ε . The larger the primes in S (compared to T) and the larger the
size of S, the more irrelevant ε becomes.

h=C(n,k −1) i=n−k+1


s
c≈ ∑ (k − 1)! ∏ (s′c i − π (F loor[ t i ])
i
h=1 i =1

Thankfully counting surjections does not concern the Andrica Conjecture, to which we now
resume our attention. Contact me at [email protected] if you want to expand or simplify
otherwise optimize the counting methods I’ve so far introduced.
Appendix D

Rank Euler Polynomic Identities

Although the concept of a Rank Euler Polynomic Identity will not be used in this volume, it would
not do this section justice to conclude it without the final “polynomial to end all polynomials.”

Since we will not be using it in this volume, I will be brief with a quick numerical example.

3 2 4
Let's take the number 24,636,606,975 = 3 (5 )17 (19)23 . Since 4 is the largest exponent in the 
prime factorization, I'm going to term this a Rank-4 Euler Polynomial Identity. 
 
Each rank, r, is the set of distinct prime numbers in the factorization that has an exponent of r or greater. 

Rank 1: = {3, 5, 17, 19, 23} 

Rank 2: = {3, 5, 17} 

Rank 3: = {3, 17} 

Rank 4: = {17} 

For each rank, I'm going to count the numbers of gaps between each pair of consecutive totatives 
to the product of the Rank, and then multiply the counts by their respective lengths and sum them 
together to obtain the Primorial Sum for each rank. Then I will multiply the Primorial Sums of each rank 
together to return the original number. 

I know that the maximal gap of {3,5,17,19,23} is equal to 9, and that 5 is in the set and that n > 4, 
so the general trigesimal formulas will be used here (the formulas for radix 15 are the same for radix 30, 
just remember to halve the actual gap sizes). 

;  

Max Gap Between Totatives to 111,435 = 9 (gap would be 18 if 2 was in S). 

Count of gaps of length 1 = 16065 

Count of gaps of length 2 = 16065 

Count of gaps of length 3 = 14210 

Count of gaps of length 4 = 1850 


 

Count of gaps of length 5 = 1950 

Count of gaps of length 6 = 412

Count of gaps of length 7 = 112

Count of gaps of length 8 = 12

In this particular case of counting gaps of length 8, we did not have enough primes in the set for 
the quartic polynomial to activate (notice that the upper determinant was increased to 6). We would have 
needed at least one more prime (any prime except 2 or 7) in our set to activate the full form (the zero 
coefficient changes to 1, the 0 signifies that a null count occurred, we need a total of four extra primes to 
fill 4 unsieved locations. The same problem occurs for gaps of nine, where neither the quintic nor quartic 
parts of the polynomial can activate. 

{simplified}Count of gaps of length 8 = 12 

   
Counts of gaps length 9 = 12 (there are 6 systems of congruences that permit a gap of 9 for the 
set {3,5,17,19,23}, and each system has an additive inverse, making a total of 12. 

Now we add the gaps together (Euler Phi Function) to confirm the count is correct: 

16065 + 16065 +14210 +1850 + 1950 +412 +112 +12 + 12 = 50,688 = (2*4*16*18*22); Correct! 

Now we multiply each of the above counts by their corresponding lengths: 

(1)16065 + (2)16065 +(3)14210 +(4)1850 + (5)1950 +(6)412 +(7)112 +(8)12 + (9)12 = 111435, Correct! 

We have now completed the Rank 1 Euler Polynomial Identity! 

Rank 2 is much quicker. = {3,5,17} 

;  

Max Gap Between Totatives to 255 = 5  

Count of gaps of length 1 = 45 

Count of gaps of length 2 = 45 

Count of gaps of length 3 = 34 

Count of gaps of length 4 = 2 

Count of gaps of length 5 = 2 

The count of gaps length 5 is not altered by the 4/3, 3/2, 1/2 like it was in the Rank-1 calculations, since 

. In gaps of 5, the prime number 3 sieves two of the integers, and the other two primes (5 and 
17 in this case) must sieve the two in between, and thus there's only two ways to create this gap. 

Count of all gaps = 45 +45 +34 + 2 + 2 = 128 = (phi 255), correct. 

Counts of gaps multiplied by their respective lengths: 

(1)(45) + (2)(45)+(3)(34) + (4)(2) + 5(2) = 255 = 3*5*17, correct. 

 
Rank 3: = {3,17} 

;  

Max Gap Between Totatives to 51 = 2  

Count of gaps of length 1 = 15 

Count of gaps of length 2 = 15 

Count of gaps of length 3 = 2 

15+15+2 = 32 (phi 51) 

(1)(15)+(2)(15)+(3)(2) = 51 

Rank 4: = {17} 

  

Max Gap Between Totatives to 51 = 2  

{1}: Count of gaps of length 1 = 15 

Count of gaps of length 2 = 1 

(because 3 is no longer in the set, the formula has returned to the most basic Class A1 form) 

15 + 1 = 16 

(1)15 + (2)1 = 17 

Those are all four of the Rank Euler Polynomial Identities for 24,636,606,975. 

Now that we have our Rank Euler Identities, we can sum together all the coefficients for each
polynomial per rank (after multiplying through by their respective lengths) to get the Primorial Sum of
each rank, call this Rank-Identity Polynomial ζ r . Then we can multiply all the Rank-Identity Polynomials
together to get one massive distinct and unique polynomial that sums to the original number (it must be
unique since the prime factorization of every integer is unique).
Appendix E

Extended Treatment of Case 3a of the Surjection Theorem.

It is assumed that the reader has read all of Section IV before proceeding to this appendix.
Lemma 4.1.2.4: Maximal Gap Attained, The Exchange Case, Rigorous Proof
Case 3a:
T = Pn ∩ Sn = {2, … pn−j } 1 ≤ j ≤ 2

Since the sets Pn and Sn share all of the primes from 2 to pn−2 , then the only difference between the sets
is the existence of pn−1 and pn against sn−1 and sn .

Case 3a.x: j = 1
T = Pn ∩ Sn = {2, … pn−1 }; T = Pn-1
L = T - Pn = { ℓ1 } = { pn }
M = T - Sn = { m1 } = { sn }

Let E be an Exchange Set, which will be equal to the symmetric difference of Pn and Sn,
E = (Pn U Sn) - (T), or more formally E = Pn Δ Sn ∴ E = {ℓ1 , m1 }, |E| = 2 ≥ (n − (n − 1)) = 1

Step 1, proving pn is belligerent:


Since the set Pn cannot sieve more than 2 pn−1 consecutive integers, then whenever pn becomes
active in generating a gap, it must be belligerent, since it is not possible to have two consecutive odd
multiples of pn contained within the same gap, since that gap would have to be larger than 2pn .

Ψ(μ) ≤ 2pn−1 ; γ ≤ pn−1

k μ−γ ... ... ... ... μ+j ... μ+γ

ξ (k) ∅ ... ... ... ... pn ... ∅

Step 2, proving sn is belligerent:


Also, since the the set Pn-1 is a subset of Sn, then when the prime m1 , m1 = sn , is active, such
that if it sieves an integer within the nucleus, then it must be belligerent, since m1 cannot sieve two or
more integers within the nucleus, as the gap would be larger than 2m1 , and it is not possible for the
subset Pn-1 to sieve the (2m1 − 2) consecutive integers betwixt the two consecutive odd multiples of m1 .

Contradiction: Ψ(μ) > 2pn−1 ; γ > pn−1 ; (μ + j − m1 ) ≡ 0 mod 2 ≡ 0 mod 3 ≡ a3 mod 5... ≡ an−1 mod pn−1

k μ − γ ... μ + j − 2m1 μ + j − 2m1 + 1 ... μ + j − m1 ... μ + j -1 μ+j ... μ+γ

ξ (k) ∅ ... m1 ... ... 2 ... ... m1 ... ∅

Let ξ (μ + j ) = m1 ; ξ (μ + j − 2m1 ) = m1
Let c = (μ + j − m1 ) , then c ≡ 0 mod 2 ≡ 0 mod 3 ≡ a3 mod 5... ≡ an−1 mod pn−1 . Therefore, in accordance
with the Strong Andrica Conjecture, which has been verified up to Pn, this system of congruences cannot
sieve more than 2pn−2 consecutive integers, never mind 2m1 − 2 consecutive integers.
Step 3, the Exchange:

Since both pn and sn must be belligerent or nugatory in any gap generated by their shared
subset, T, then we use the exchange set, E, to state the following:

Let E = Pn Δ Sn = { pn , sn }
and let E′ ⊆ E, such that |E′| = n − (n − 1) = 1 , |E′| = τ = 1
|T| = n − 1

X = T U E′, and therefore |X| = m + τ = n , X is the exchanged set.

Since each element of E must be belligerent when placed into a system of congruences with the
primes in T then if it were possible for Sn to sieve more than 2pn−1 consecutive integers, then the set Pn
could also sieve more than 2pn−1 consecutive integers, which is absurd! It’s already been proven by brute
force that the set Pn cannot sieve more than 2pn−1 consecutive integers, and therefore neither can the set
Sn sieve more than 2pn−1 consecutive integers since the exchange set would map an illegal surjection
onto the nuclei (gaps) generated by Pn.

Step 4, Asserting this result for Pw, w ≤ n :

Given any w, w ≤ n, Pw-1 is a subset of Sw, then when the primes ℓ1 and m1 ( ℓ1 = pw ; m1 = sw ), are
active, such that if they sieve an integer within the nucleus, then they must be belligerent, since neither
prime can sieve two or more integers within the nucleus, as the gap would be larger than 2ℓ1, and it is not
possible for the subset Pw-1 to sieve the (2 ℓ1 − 2) consecutive integers betwixt the two consecutive odd
multiples of ℓ1, nor the ( 2m1 − 2) consecutive integers betwixt the two consecutive odd multiples of m1 .

Therefore δ (Sw) = 2pw−1 under the overall conditions of Case 3a.


QED
Case 3a.y: j = 2
T = Pn ∩ Sn = {2, … pn−2 }; T = Pn-2
L = T - Pn = { ℓ1 , ℓ2 } = { pn−1 , pn }
M = T - Sn = { m1 , m2 } = { sn−1 , sn }

Let E be an Exchange Set, which will be equal to the symmetric difference of Pn and Sn,
E = (Pn U Sn) - (T), or more formally E = Pn Δ Sn ∴ E = {ℓ1 , ℓ2 , m1 , m2 }, |E| = 4 ≥ (n − (n − 2)) = 2

Step 1, proving pn−1 , pn are belligerent:


Since the set Pn cannot sieve more than 2 pn−1 consecutive integers, then whenever pn−1 or pn
become active in generating a gap, they must be belligerent, since it is not possible to have two
consecutive odd multiples of pn−1 or pn contained within the same gap, since that gap would have to be
larger than 2pn−1 .

Ψ(μ) ≤ 2pn−1 ; γ ≤ pn−1

k μ−γ ... μ+i ... ... μ+j ... μ+γ

ξ (k) ∅ ... pn−1 ... ... pn ... ∅

Step 2, proving sn−1 and sn are belligerent:

We shall revert to Step 4 of the previous case to prove these:

Let Sw = (Pn-2 ⋃ { m2 }) ; w = n − 1 , then Sw-1 = Pn-2, therefore, in accordance with Step 4 of Case
3.a.x, Sw cannot span the bracket of m1 , since the maximal gap δ (Sw) = 2pw−1 < 2m1 .

Let Sw = (Pn-2 ⋃ { m1 }) ; w = n − 1 , then Sw-1 = Pn-2, therefore, in accordance with Step 4 of Case
3.a.x, Sw cannot span the bracket of m2 , since the maximal gap δ (Sw) = 2pw−1 < 2m2 .

Therefore, since the set Pn-2 is a subset of Sn, then when the primes m1 or m2 are active such
that if they sieve an integer within the nucleus, then they must be belligerent, since m1 or m2 cannot sieve
two or more integers within the nucleus, as the gap would be larger than 2m1 , and it is not possible for
the subset Pn-2 to sieve the (2m1 − 2) or (2m2 − 2) consecutive integers that exist within the brackets of
m1 or m2 (respectively), even if m2 or m1 are partially belligerent within the bracket of m1 or m2
(respectively).
Step 3, the Exchange:

Since both pn−1 , pn and sn−1 , sn must be belligerent or nugatory in any gap generated by their
shared subset, Pn-2, then we use the exchange set, E, to state the following:

Let E = Pn Δ Sn = { pn−1 , pn , sn−1 , sn } = {ℓ1 , ℓ2 , m1 , m2 }


and let E′ ⊆ E, such that |E′| = n − (n − 2) = 2 , |E′| = τ = 2
|T| = n − 2

X = T U E′, and therefore |X| = m + τ = n , X is the exchanged set.

Since each element of E must be belligerent when placed into a system of congruences with the
primes in T then if it were possible for Sn to sieve more than 2pn−1 consecutive integers, then the set Pn
could also sieve more than 2pn−1 consecutive integers, which is absurd! It’s already been proven by brute
force that the set Pn cannot sieve more than 2pn−1 consecutive integers, and therefore neither can the set
Sn sieve more than 2pn−1 consecutive integers since the exchange set would map an illegal surjection
onto the nuclei (gaps) generated by Pn.

Step 4, Asserting this result for Pw, w ≤ n :


Given any w, w ≤ n, Pw-2 is a subset of Sw, then when the primes ℓ1 , ℓ2 and m1 , m2 are active, such
that if they sieve an integer within the nucleus, then they must be belligerent, since none of those primes
can sieve two or more integers within the nucleus, as the gap would be larger than 2ℓ1, and it is not
possible for the subset Pw-2 to sieve the (2 ℓ1 − 2) consecutive integers betwixt the two consecutive odd
multiples of ℓ1, nor the ( 2m1 − 2) consecutive integers betwixt the two consecutive odd multiples of m1 .

Therefore δ (Sw) = 2pw−1 under the overall conditions of Case 3a.


QED
Case 3a.z is an offshoot of the general Case 3c.

Case 3a.z:
T = Pn ∩ Sn = {2, … pn−2 , pn }; T′ = Pn-2
L = T - Pn = { ℓ1 } = { pn−1 }
M = T - Sn = { m1 } = { sn }; pn = sn−1

Step 1, proving pn−1 , pn are belligerent:


Since the set Pn cannot sieve more than 2 pn−1 consecutive integers, then whenever pn−1
becomes active in generating a gap, it must be belligerent, since it is not possible to have two
consecutive odd multiples of pn−1 contained within the same gap, since that gap would have to be larger
than 2pn−1 ; likewise, neither can pn sieve more than one integer in the same gap, therefore it must also
be belligerent if it becomes active.

Step 2, proving sn−1 = pn is belligerent:


Since the set Pn-1 cannot sieve more than 2 pn−2 consecutive integers, then whenever sn−1 = pn
becomes active in generating a gap for the set Sn-1, it must be belligerent, since it is not possible to have
two consecutive odd multiples of sn−1 = pn contained within the same gap, since that gap would have to
be larger than 2pn . Therefore it follows from Step 4 of Case 3a.x that:

Let Sw = (Pn-2 ⋃ { sn−1 }) ; w = n − 1 , therefore δ (Sw) = 2pw−1 = 2pn−1 < 2pn = 2sn−1 .

Step 3, proving sn−1 is belligerent:


Since the set Pn-1 cannot sieve more than 2 pn−2 consecutive integers, then whenever sn
becomes active in generating a gap for the set Sn-1, it must be belligerent, since it is not possible to have
two consecutive odd multiples of sn contained within the same gap, since that gap would have to be
larger than 2sn . Therefore it follows from Step 4 of Case 3a.x that:

Let Sw = (Pn-2 ⋃ { sn }) ; w = n − 1 , therefore δ (Sw) = 2pw−1 = 2pn−1 < 2pn < 2sn .

Step 4, applying Case 3c to prove Case 3.a.z


If it were possible for Sn to generate a gap whose length exceeds 2pn−1 , then both sn−1 = pn and
sn must be belligerent, since if either is bracketed, there exists no subset of Sn that could span the bracket
of either.

Contradiction: Ψ(μ) > 2pn−1 ; γ > pn−1 ; (μ + j − m1 ) ≡ 0 mod 2 ≡ 0 mod 3 ≡ ...an−2 mod pn−2 ≡ an mod pn

k μ − γ ... μ + j − 2m1 μ + j − 2m1 + 1 ... μ + j − m1 ... μ + j -1 μ+j ... μ+γ

ξ (k) ∅ ... m1 ... ... 2 ... ... m1 ... ∅

Let ξ (μ + j ) = m1 ; ξ (μ + j − 2m1 ) = m1
Let c = (μ + j − m1 ) , then c ≡ 0 mod 2 ≡ 0 mod 3 ≡ ...an−2 mod pn−2 ≡ an mod pn . Therefore, in accordance
with the Strong Andrica Conjecture, which has been verified up to Pn, this system of congruences cannot
sieve more than 2pn−2 consecutive integers, never mind 2m1 − 2 consecutive integers.
QED

You might also like